Harrisons Principles of Internal Medicine Self-Assessment and Board Review 18th Ed.

SECTION IV. Infectious Diseases

ANSWERS

IV-1.   The answer is B. (Chap. 119) Deficiencies in the complement system predispose patients to a variety of infections. Most of these deficits are congenital. Patients with sickle cell disease have acquired functional defects in the alternative complement pathway. They are at risk of infection from Streptococcus pneumoniae and Salmonella spp. Patients with liver disease, nephrotic syndrome, and systemic lupus erythematosus may have defects in C3. They are at particular risk for infections with Staphylococcus aureus, S. pneumoniae, Pseudomonas spp., and Proteus spp. Patients with congenital or acquired (usually systemic lupus erythematosus) deficiencies in the terminal complement cascade (C5-8) are at particular risk of infection from Neisseria spp. such as N. meningitis or N. gonorrhoeae.

IV-2.   The answer is A. (Chap. 121) Choosing the appropriate empiric coverage for patients with severe sepsis is important for improving outcomes. This patient has undergone splenectomy in the distant past and is at 58 times greater risk for overwhelming bacterial sepsis than the normal population. The risk is greatest in the first 2 years after splenectomy but persists across the entire lifetime. The most common organisms causing severe sepsis in asplenic patients are encapsulated bacteria, especially Streptococcus pneumoniae, Neisseria meningitidis, and Haemophilus influenzaeS. pneumoniae causes 50% to 70% of severe sepsis in asplenic patients. The recommended empiric antibiotics in asplenic patients with sepsis are ceftriaxone 2 g IV every 12 hours and vancomycin 1 g IV every 12 hours.

IV-3.   The answer is C. (Chap. 121) Necrotizing fasciitis is a life-threatening infection that leads to extensive necrosis of the subcutaneous tissue and fascia. It is most commonly caused by group A streptococci and a mixed facultative and anaerobic flora. Recently, there have been an increasing number of cases of necrotizing fasciitis caused by community-acquired methicillin-resistant Staphylococcus aureus. Risk factors include diabetes mellitus, intravenous drug use, and peripheral vascular disease. The infection often arises at a site of minimal trauma, and the physical findings initially are minimal compared with the severity of pain and fever. The mortality rate for necrotizing fasciitis is between 15% and 34% but rises to as high as 70% if toxic shock syndrome is present. Wide surgical debridement of the affected tissue is necessary, and without surgery, the mortality rate is near 100%. A high index of clinical suspicion is important for selecting the appropriate antibiotic therapy and early consultation of surgery. The initial antibiotics should cover the typical organisms and include vancomycin 1 g IV every 12 hours, clindamycin 600 mg IV every 6–8 hours, and gentamicin 5 mg/kg/day intravenously.

IV-4.   The answer is B. (Chap. 121) Cat bites are the most likely animal bites to lead to celluli-tis because of deep inoculation and the frequent presence of Pasteurella multicoda. In an immunocompetent host, only cat bites warrant empirical antibiotics. Often the first dose is given parenterally. Ampicillin–sulbactam followed by oral amoxicillin–clavulanate is effective empirical therapy for cat bites. However, in an asplenic patient, a dog bite can lead to rapid overwhelming sepsis as a result of Capnocytophaga canimorsus bacteremia. These patients should be followed closely and given third-generation cephalosporins early in the course of infection. Empirical therapy should also be considered for dog bites in elderly adults, for deep bites, and for bites on the hand.

IV-5.   The answer is A. (Chap. 122) Immunization programs have the goals to control, eliminate, and eradicate disease. Disease control refers to decreases the impact of a specific illness on both health-related and societal outcomes. Examples of vaccinations that have lead to improved control of disease include the pneumococcal and influenza vaccines. Elimination can have two meanings. The first definition is to have zero cases in a defined geographic area. A second meaning is to reduce or eliminate the indigenous sustained transmission of an infection in a specific geographic area. In 2010, vaccine programs had eliminated measles, rubella, poliomyelitis, and diphtheria in the United States, although increasing numbers of cases of measles have been reported is some parts of the United States because of incomplete vaccination in children. Disease eradication is the most difficult goal to achieve. A disease can be considered eradicated when its elimination can be sustained without ongoing interventions. The only disease that has been globally eradicated at this point is smallpox. Poliomyelitis has been eradicated in most of the world although Afghanistan, Pakistan, India, and Nigeria continue to have ongoing transmission of the disease.

IV-6.   The answer is B. (Chap. 122) Pneumococcal vaccination has been recommended for all individuals at any age with a variety of chronic medical conditions, including chronic respiratory disease, chronic heart disease, chronic liver failure, diabetes mellitus, asplenia, and chronic kidney disease. Determining when to revaccinate individuals has been somewhat controversial. The current recommendations are to revaccinate individuals ages 19 to 64 5 years after the initial vaccine if they have chronic renal failure or nephrotic syndrome, asplenia, or other immunocompromising conditions. All other individuals should receive a one-time revaccination at age 65 years and older if they were vaccinated 5 or more years previously and younger than 65 years old at the time of original vaccination.

IV-7.   The answer is C. (Chap. 122) The varicella-zoster vaccine is a live virus vaccine that was recently introduced for prevention of shingles in older adults. The current recommendation is all adults older than 60 years be offered the zoster vaccine regardless of whether they report a childhood history of chickenpox. As this is a live virus vaccine, it cannot be administered to anyone who has severe immunodeficiency. Specific recommendations for whom the zoster vaccine is contraindicated include:

1. Pregnancy

2. Anyone younger than 60 years

3. Patients with leukemia, lymphoma, or other malignant neoplasms affecting the bone marrow. If a patient is in remission and has not received chemotherapy or radiation therapy within 3 months, the vaccine can be given.

4. Individuals with AIDS or HIV with a CD4+ count <200/μL or ≤15% peripheral lymphocytes

5. Individuals taking immunosuppressive therapy equivalent to prednisone ≥20 mg/day, methotrexate 0.4 mg/kg/wk, or azathioprine <3 mg/kg/day

6. Anyone with suspected cellular immunodeficiency (e.g., hypogammaglobulinemia)

7. Individuals receiving a hematopoietic stem cell transplant

8. Individuals receiving recombinant human immune mediators or modulators, especially antitumor necrosis factor agents

IV-8.   The answer is E. (Chap. 122) Live-attenuated viruses are generally contraindicated as vaccines for immunocompromised hosts for fear of vaccine-induced disease. The most cited example of this is smallpox vaccine resulting in disseminated vaccinia infection. However, yellow fever vaccine is another example of a live virus vaccine. The other examples listed in this example are inactivated organisms (rabies, IM typhoid) or polysaccharide (meningococcal) and are therefore noninfectious. Oral typhoid vaccine is a live-attenuated strain, so the IM form is likely preferable in this host. Malaria prophylaxis currently involves chemoprophylaxis rather than vaccination. Although safe from an infectious standpoint, potential interactions with cyclosporine should be monitored.

IV-9.   The answer is E. (Chap. 122) In recent years, rabies virus has been most frequently transmitted by bats in the United States. Usually a bite is noted but not always. Therefore, patients who have unexpected, unmonitored (i.e., while they are asleep) close contact with bats should be told to seek medical attention and likely vaccination. A bite is a clear indication for the most effective immunization strategy involving both active (inactivated virus vaccine) and passive (human rabies immunoglobulins) immune activation unless the offending bat is captured and found to be rabies negative with further testing. The vaccination schedule for nonimmunes is intensive, with doses at 0, 3, 7, 14, and 28 days. Although there has been at least one report of successful antiviral treatment of rabies, there is no indication for prophylactic antiviral therapy.

IV-10.  The answer is E. (Chap. 123) When traveling abroad, it is important to plan ahead and consider the potential infectious agents to which one might be exposed. The Centers for Disease Control and Prevention and the World Health Organization publish guidelines for recommended vaccinations before travel to countries around the world. Before travel, it is certainly recommended that an individual be up to date on all routine vaccinations, including measles, diphtheria, and polio. Influenza is perhaps the most common preventable illness in travelers, and the influenza vaccine should be administered per routine guidelines. There are, however, very few required vaccinations in most countries. Yellow fever is one exception, and proof of vaccination is required by many countries in sub-Saharan Africa and equatorial South America. This is especially important for individuals traveling from areas where yellow fever is endemic or epidemic. The only other required vaccinations are meningococcal meningitis and influenza vaccination to travel in Saudi Arabia during the Hajj.

IV-11.  The answer is E. (Chap. 123http://wwwnc.cdc.gov/travel/destinations/haiti.htm. Malaria remains endemic in many parts of the world, and an estimated 30,000 travelers from the United States and Europe are infected with malaria during travel yearly. The areas of highest risk are in sub-Saharan Africa and Oceania with the lowest risk in South and Central America, including Haiti and the Dominican Republic. Chloroquine resistance is growing throughout the world and is especially notable in parts of South America, Africa, and Southeast Asia. However, in Haiti, the incidence of chloroquine resistant malaria is low. For a traveler to Haiti, the Centers for Disease Control and Prevention states that travelers have a choice of chloroquine, doxycycline, atovaquone–proguanil, or mefloquine. In addition, travelers should be cautioned to use appropriate techniques for malarial prevention, including protective clothing, DEET-containing insect repellants, permethrin-impregnated bednets, and screened sleeping accommodations, if possible.

IV-12.  The answer is E. (Chap. 123) Individuals with HIV are generally considered at high risk of infectious complications when traveling abroad. However, individuals who have no symptoms and a CD4+ count greater than 500/μL appear to be at no greater risk than individuals without HIV infection. Before travel, it is important to research the travel requirements for the specific country of travel. Many countries routinely deny entry for HIV-positive individuals for prolonged stays, and proof of HIV testing is required in many countries for stays longer than 3 months. Consular offices should be contacted before travel to determine if any special documentation is required. HIV-infected travelers should have all routine immunizations before travel, including influenza and pneumococcal vaccinations. The response rate to influenza in an asymptomatic HIV-positive person is greater than 80%. Generally, live-attenuated viruses are not given to HIV-infected individuals. However, because measles can be lethal in those with HIV, this vaccine is recommended unless the CD4+ count is less than 200/μL, and the expected response rate would be between 50% and 100%. In contrast, the live yellow fever vaccine is not given to HIV-infected travelers, and individuals with CD4+ counts below 200/μL should be discouraged from traveling to countries with endemic yellow fever. Some countries in sub-Saharan Africa require yellow fever vaccination. However, because this patient is traveling from a low-risk area, a medical waiver would likely be issued.

IV-13.  The answer is E. (Chap. 124) The etiologic agents of infective endocarditis vary by host (see Figure IV-13). Community-acquired native valve endocarditis remains an important clinical problem, particularly in elderly people. In those patients, streptococci (Viridans spp., S. gallolyticus, other non–group A and other group streptococci, and Abiotrophia spp.) account for approximately 40% of cases. Staphylococcus aureus (30%) is next most common. Enterococci, HACEK group, coagulase-negative, and culture-negative cases each account for less than 10% of community-acquired native valve cases. In health care–associated, injection drug use–associated, and greater than 12-month-old prosthetic valve endocarditis, S. aureus is most common. Coagulase-negative staphylococcus is the most common organism in prosthetic valve endocarditis less than 12 months. Enterococci cause endocarditis in approximately 10% to 15% of cases in health care–associated, 2- to 12-month prosthetic valve, and injection drug use cases. Culture-negative endocarditis accounts for 5% to 10% of cases in all of the aforementioned clinical scenarios.

image

FIGURE IV-13 (see Color Atlas)

IV-14.  The answer is B. (Chap. 124) The Duke criteria for diagnosis of infective endocarditis are a set of major and minor clinical, laboratory, and echocardiographic criteria that are highly sensitive and specific. The presence of two major criteria, one major criterion and three minor criteria, or five minor criteria allows a clinical diagnosis of definite endocarditis (see Table IV-14). Evidence of echocardiographic involvement as evidenced by an oscillating mass (vegetation) on a valve, supporting structure, or implanted material; an intracardiac abscess or partial dehiscence of a prosthetic valve; or a new valvular regurgitation are major criteria in the Duke classification. An increase or change in preexisting murmur by clinical examination is not sufficient. Transthoracic echocardiography is specific for infective endocarditis but only finds vegetations in about 65% of patients with definite endocarditis. It is not adequate for evaluation of prosthetic valves or for intracardiac complications. Transesophageal echocardiography is more sensitive, detecting abnormalities in more than 90% of cases of definite endocarditis.

TABLE IV-14 The Duke Criteria for the Clinical Diagnosis of Infective Endocarditisa


Major Criteria

1. Positive blood culture

Typical microorganism for infective endocarditis from two separate blood cultures

Viridans streptococci, Streptococcus gallolyticus, HACEK group, Staphylococcus aureusor Community-acquired enterococci in the absence of a primary focus, or



Persistently positive blood culture, defined as recovery of a microorganism consistent with infective endocarditis from:

Blood cultures drawn >12 h apart; or

All of 3 or a majority of ≥4 separate blood cultures, with first and last drawn at least 1 h apart



Single positive blood culture for Coxiella burnetii or phase I IgG antibody titer of >1:800

2. Evidence of endocardial involvement

Positive echocardiogramb

Oscillating intracardiac mass on valve or supporting structures or in the path of regurgitant jets or in implanted material in the absence of an alternative anatomic explanation, or

Abscess, or

New partial dehiscence of prosthetic valve, or



New valvular regurgitation (increase or change in preexisting murmur not sufficient)

Minor Criteria

1. Predisposition: predisposing heart condition or injection drug use

2. Fever ≥38.0°C (≥100.4°F)

3. Vascular phenomena: major arterial emboli, septic pulmonary infarcts, mycotic aneurysm, intracranial hemorrhage, conjunctival hemorrhages, Janeway lesions

4. Immunologic phenomena: glomerulonephritis, Osler’s nodes, Roth’s spots, rheumatoid factor

5. Microbiologic evidence: positive blood culture but not meeting major criterion as noted previouslyc or serologic evidence of active infection with organism consistent with infective endocarditis


aDefinite endocarditis is defined by documentation of two major criteria, of one major criterion and three minor criteria, or of five minor criteria. See text for further details.

bTransesophageal echocardiography is recommended for assessing possible prosthetic valve endocarditis or complicated endocarditis.

cExcluding single positive cultures for coagulase-negative staphylococci and diphtheroids, which are common culture contaminants, and organisms that do not cause endocarditis frequently, such as gram-negative bacilli.

Abbreviation: HACEK, Haemophilus spp., Aggregatibacter actinomycetemcomitansCardiobacterium hominisEikenella corrodensKingella spp.

Source: Adapted from Li JS et al: Proposed modifications to the Duke criteria for the diagnosis of infective endocarditis. Clin Infect Dis 30:633, 2000, with permission from the University of Chicago Press.

IV-15.  The answer is C. (Chap. 124) The recommendations for prophylaxis to prevent infective endocarditis have undergone change recently with a change to recommending it for fewer patients. The most recent American Heart Association guidelines (Circulation 116:1736, 2007) reverse many of the former recommendations based on indirect evidence suggesting that benefit is minimal and is not supported by cost-benefit or cost-effectiveness studies. Current recommendations advise prophylactic antibiotics only for those at highest risk for severe morbidity or mortality from endocarditis undergoing manipulation of gingival tissue or periapical region of the teeth, perforation of the oral mucosa, or a procedure on an infected site. Prophylaxis is not advised for routine gastrointestinal or genitourinary procedures. High-risk patients include those with prior endocarditis, prosthetic heart valves, unrepaired cyanotic congenital heart disease lesions, recently (<6 months) repaired congenital heart lesions, incompletely repaired congenital heart disease lesions, and valvulopathy after cardiac transplant. The British Society for Antimicrobial Chemotherapy does recommend prophylaxis for at-risk patients undergoing selected gastrointestinal or genitourinary procedures; however, the National Institute for Health and Clinical Excellence in the United Kingdom advised discontinuation of the practice (http://www.nice.org.uk/guidance/cg64).

IV-16.  The answer is A. (Chap. 124) This patient has culture-negative endocarditis, a rare entity defined as clinical evidence of infectious endocarditis in the absence of positive blood cultures. In this case, evidence for subacute bacterial endocarditis includes valvular regurgitation; an aortic valve vegetation; and embolic phenomena on the extremities, spleen, and kidneys. A common reason for negative blood cultures is prior antibiotics. In the absence of this, the two most common pathogens (both of which are technically difficult to isolate in blood culture bottles) are Q fever, Coxiella burnetii (typically associated with close contact with livestock), and Bartonella spp. In this case, the patient’s homelessness and body louse infestation are clues for Bartonella quintana infection. Diagnosis is made by blood culture about 25% of the time. Otherwise, direct polymerase chain reaction of valvular tissue, if available, or acute and convalescent serologies are diagnostic options. Empirical therapy for culture-negative endocarditis usually includes ceftriaxone and gentamicin with or without doxycycline. For confirmed Bartonella endocarditis, optimal therapy is gentamicin plus doxycycline. EBV and HIV do not cause endocarditis. A peripheral blood smear would not be diagnostic.

IV-17.  The answer is D. (Chap. 124) Although any valvular vegetation can embolize, vegetations located on the mitral valve and vegetations larger than 10 mm are greatest risk of embolizing. Of the answer choices, C, D, and E are large enough to increase the risk of embolization. However, only choice D demonstrates the risks of both size and location. Hematogenously seeded infection from an embolized vegetation may involve any organ but particularly affects those organs with the highest blood flow. They are seen in up to 50% of patients with endocarditis. Tricuspid lesions lead to pulmonary septic emboli, which are common in injection drug users. Mitral and aortic lesions can lead to embolic infections in the skin, spleen, kidneys, meninges, and skeletal system. A dreaded neurologic complication is mycotic aneurysm, focal dilations of arteries at points in the arterial wall that have been weakened by infection in the vasa vasorum or septic emboli, leading to hemorrhage.

IV-18.  The answer is A. (Chap. 124) Patients with infective endocarditis on antibiotic therapy can be expected to demonstrate clinical improvement within 5 to 7 days. Blood cultures frequently remain positive for 3 to 5 days for Staphylococcus aureus treated with β-lactam antibiotics and 7 to 9 days with vancomycin. Neither rifampin nor gentamicin has been shown to provide clinical benefit in the scenario described in this question. Vancomycin peak and trough levels have not been shown to improve drug efficacy in infective endocarditis. It is too early in therapy to consider this case representative of vancomycin failure. The efficacy of daptomycin or linezolid because an alternative to vancomycin for left-sided MRSA endocarditis has not been established.

IV-19.  The answer is B. (Chap. 125) Bullae (Latin for bubbles) are skin lesions that are greater than 5 mm and fluid filled. They may be regular or irregularly shaped and filled with serous or seropurulent fluid. Clostridium spp., including perfringens, may cause bullae through myonecrosis. Staphylococcus causes scalded skin syndrome through elaboration of the exfoliatin toxin from phage group II, particularly in neonates. Streptococcus pyogenes, the causative agent of impetigo, may cause bullae initially that progress to crusted lesions. MRSA may also cause impetigo. The halophilic Vibrio, including V. vulnificus, may cause an aggressive fasciitis with bullae formation. Patients with cirrhosis exposed to Gulf of Mexico or Atlantic waters (or ingestion of raw seafood from those waters) are at greatest risk. Infection with the dimorphic fungus, Sporothrix schenckii, presents with discrete crusted lesions resembling ringworm. Lesions may progress to ulcerate. Patients often have a history of working with soil or roses.

IV-20.  The answer is D. (Chap. 125) This patient has necrotizing fasciitis and myonecrosis. His computed tomography scan shows edema and inflammation of the left chest wall. Necrotizing fasciitis and myonecrosis may also be caused by infection with mixed aerobes and anaerobes, Staphylococcus aureus including, methicillin-resistant S. aureus, and Clostridium spp. Treatment involves prompt surgical evaluation and empiric therapy for the causative agents. Mycobacterium tuberculosis would most commonly cause cavitary lung lesions. Coxsackie virus causes vesicular lesions during acute infection. There may be myalgias and elevated muscle enzymes but not frank myonecrosis. Rickettsia akari is the causative agent of rickettsialpox. It occurs after a mite bite with a papule with central vesicle that evolves to form a painless black eschar. Rickettsialpox has been recently described in Ohio, Arizona, and Utah. Varicella-zoster virus causes chickenpox with acute infection and zoster with reactivation. The lesions are crusting vesicles, not fasciitis or myonecrosis.

IV-21.  The answer is C. (Chap. 126) Although Staphylococcus aureus (methicillin-resistant Saureus and methicillin-sensitive S. aureus) is the most common bacteria causing osteomyelitis, the infection may also be caused by gram-negative organisms (Pseudomonas aeruginosaEscherichia coli), coagulase-negative staphylococci, enterococci, and propionibacteria. Mycobacterium tuberculosis is an important cause of vertebral osteomyelitis in countries with fewer medical resources and high prevalence (along with brucellosis). Prosthetic joint implants and stabilization devices are commonly sources of osteomyelitis, often seeded from bacteremia or after trauma. MRSA osteomyelitis is a growing problem in hospitals throughout the developed world, particularly after surgery. The reason for the higher morbidity and cost may be related to virulence factors or less effective or timely treatment. Patients with diabetes are at very high risk of osteomyelitis of the foot and require a high index of suspicion.

IV-22.  The answer is A. (Chap. 126) The therapy for osteomyelitis is challenging because of the multiplicity of potential causative organisms, the diagnostic difficulty, and the prolonged necessary therapy. Early surgical intervention may be beneficial diagnostically and therapeutically. In this case, the Gram stain is polymicrobial, and the putrid smell is very specific for anaerobic organisms. The diagnosis of acute osteomyelitis is also very likely based on the positive probe to bone test and wide ulcer. Broad-spectrum antibiotics are indicated. Vancomycin and linezolid cover methicillin-resistant Staphylococcus aureus (MRSA) and streptococcal isolates but would miss gram-negative rods and anaerobic bacteria. Metronidazole covers only anaerobes, missing gram-positive organisms that are key in the initiation of diabetic foot infections. Clindamycin covers gram-positive organisms and anaerobes but misses gram-negative rods. Ampicillin–sulbactam is broad-spectrum antibiotic and covers all three classes of organism except MRSA. If the patient has a history of MRSA or MRSA risk factors, then the addition of vancomycin or linezolid is a strong consideration. Recent studies have also suggested that daptomycin may be a promising therapy for MRSA osteomyelitis.

IV-23.  The answer is A. (Chap. 127) Primary (spontaneous) bacterial peritonitis (PBP) occurs when the peritoneal cavity becomes infected without an apparent source of contamination. PBP occurs most often in patients with cirrhosis, usually with preexisting ascites. The bacterial likely invade the peritoneal fluid because of poor hepatic filtration in cirrhosis. Although fever is present in up to 80% of cases, abdominal pain, acute onset, and peritoneal signs are often absent. Patients may present with nonspecific findings such as malaise or worsening encephalopathy. A neutrophil count in peritoneal fluid of greater than 250/μL is diagnostic; there is no % neutrophil differential threshold. Diagnosis is often difficult because peritoneal culture findings are often negative. Blood cultures may reveal the causative organism. The most common organisms are enteric gram-negative bacilli, but gram-positive cocci are often found. Anaerobes are not common (in contrast to secondary bacterial peritonitis), and empiric antibiotics targeting them are not necessary if PBP is suspected. Third-generation cephalosporins or piperacillin–tazobactam are reasonable initial empiric therapy. Diagnosis requires exclusion of a primary intraabdominal source of peritonitis.

IV-24.  The answer is D. (Chap. 127) This patient has continuous ambulatory peritoneal dialysis (CAPD)–associated peritonitis. Unlike primary or secondary bacterial peritonitis, this infection is usually caused by skin organisms, most commonly Staphylococcus spp. The organisms migrate into the peritoneal fluid via the device. There may not be a tunnel or exit-site infection. Peritonitis is the most common reason for discontinuing CAPD. Y-connectors and diligent technique decrease the risk of CAPD. In contrast to PBP and similar to spontaneous bacterial peritonitis (SBP), the onset of symptoms is usually acute with diffuse pain and peritoneal signs. The dialysate will be cloudy with greater than 100 WBC/μL and greater than 50% neutrophils. Dialysate should be placed in blood culture media and often is often positive with one organism. Finding more than one organism in culture should prompt an evaluation for SBP. Empirical intraperitoneal coverage for CAPD peritonitis should be directed against staphylococcal species based on local epidemiology. If the patient is severely ill, intravenous antibiotics should be added. If the patient does not respond within 4 days, catheter removal should be considered.

IV-25.  The answer is D. (Chap. 127) The computed tomography scan shows a large complex liver abscess in the right lobe. Liver abscesses may arise from hematogenous spread, biliary disease (most common currently), pylephlebitis, or contiguous infection in the peritoneal cavity. Fever is the only common physical finding in liver abscess. Up to 50% of patients may not have symptoms or signs to direct attention to the liver. Nonspecific symptoms are common, and liver abscess is an important cause of fever of unexplained origin in elderly patients. The only reliably abnormal serum studies are elevated alkaline phosphatase or WBC in 70% of patients. Liver abscess may be suggested by an elevated hemidiaphragm on chest radiograph. The most common causative organisms in presumed biliary disease are gram-negative bacilli. Anaerobes are not common unless pelvic or other enteric sources are suspected. Fungal liver abscesses occur after fungemia in immunocompromised patients receiving chemotherapy, often presenting symptomatically with neutrophil reconstitution. Drainage, usually percutaneous, is the mainstay of therapy and is useful initially diagnostically (Figure IV-25B).

image

FIGURE IV-25B

IV-26.  The answer is C. (Chap. 127) It is important to distinguish between primary (spontaneous) and secondary peritonitis. Primary peritonitis is a result of long-standing ascites, usually as a result of cirrhosis. The pathogenesis is poorly understood but may involve bacteremic spread or translocation across the gut wall of usually only a single species of pathogenic bacteria. Secondary peritonitis is caused by rupture of a hollow viscous or irritation of the peritoneum caused by a contiguous abscess or pyogenic infection. It typically presents with peritoneal signs and in most cases represents a surgical emergency. Secondary peritonitis in a patient with cirrhosis is difficult to distinguish on clinical grounds from primary (spontaneous) peritonitis. It is often overlooked because classic peritoneal signs are almost always lacking, and it is uniformly fatal in the absence of surgery. Suspicion for this diagnosis should occur when ascites shows a protein greater than 1 g/dL, lactate dehydrogenase (LDH) greater than serum LDH, glucose level below 50 mg/dL, or a polymicrobial Gram stain. When this diagnosis is suspected, abdominal radiography is indicated to rule out free air, and prompt surgical consultation is warranted. Unlike with primary (spontaneous) bacterial peritonitis, in cases of secondary peritonitis, antibiotics should include anaerobic coverage and often antifungal agents. This patient requires intravenous fluid because he has hypotension and tachycardia caused by sepsis. Drotrecogin alfa has been shown to reduce mortality in patients with sepsis, but it is not indicated in patients with thrombocytopenia, cirrhosis, and ascites.

IV-27.  The answer is B. (Chap. 128) Acute infectious diarrhea remains a leading cause of death worldwide, especially among children younger than 5 years of age. The major categories of acute diarrheal illness include noninflammatory, inflammatory, and penetrating diarrhea. Vibrio cholerae causes diarrhea through production of an enterotoxin, which is characteristic of noninflammatory diarrhea. After ingestion of a large volume (105–106) of organisms, V. cholerae attaches to the brush border of the small intestinal enterocytes and produces cholera toxin. The primary clinical characteristic of diarrheal illness caused by toxin production is profuse watery diarrhea that is not bloody. Fecal leukocytes are typically not present in noninflammatory diarrhea. However, a mild increase in fecal lactoferrin can be seen because this test is more sensitive for the presence of mild inflammation. Other pathogens that are common causes of noninflammatory diarrhea are enterotoxigenic Escherichia coliBacillus cereusStaphylococcus aureus, and viral diarrhea, among others.

The site of inflammation in inflammatory diarrhea is typically the colon or distal small bowel. In inflammatory diarrhea, there is invasion of leukocytes into the wall of the intestines. The prototypical pathogen of inflammatory diarrhea is Shigella dysenteriae. Bloody stools are common, and the stool contains large quantities of fecal leukocytes and fecal lactoferrin. Other pathogens that cause inflammatory diarrhea are most Salmonella species, Campylobacter jejuni, eneterohemorrhagic Escherichia coli, and Clostridium difficile.

Penetrating diarrhea is caused by either Salmonella typhi or Yersinia enterocolitica. The site of inflammation in penetrating diarrhea is the distal small bowel. In penetrating diarrhea, these organisms penetrate the intestinal wall and multiply within Peyer’s patches and intestinal lymph nodes before disseminating into the bloodstream. Clinically, penetrating diarrhea presents as enteric fever with fever, relative bradycardia, abdominal pain, leukopenia, and splenomegaly.

IV-28 and IV-29. The answers are B and D, respectively. (Chap. 128) Traveler’s diarrhea is common among individuals traveling to Asia, Africa, and Central and South America, affecting 25% to 50% of travelers to these areas. Most traveler’s diarrhea begins within 3 to 5 days after arrival and is self-limited, lasting 1 to 5 days. Most individuals acquire traveler’s diarrhea after consuming contaminated food or water. Although some organisms have a geographic association, enterotoxigenic and enteroaggregative Escherichia coli are found worldwide and are the most common causes of traveler’s diarrhea. In Asia, Campylobacter jejuni is also common. This presentation would be uncommon for Shigella spp. because it most frequently causes bloody diarrhea. Norovirus is associated with a more profuse diarrhea. It has been the causative organism in large outbreaks on cruise ships. Giardia lamblia is a parasite that is responsible for 5% or less of traveler’s diarrhea.

The approach to treatment of traveler’s diarrhea should be tailored to the severity of the patient’s symptoms. In general, most cases are self-limited. As long as an individual is able to maintain adequate fluid intake, no specific therapy may be required if there are no more than one or two unformed stools daily without distressing abdominal symptoms, bloody stools, or fever. In this scenario, the patient is not having a large number of stools, but in the presence of distressing abdominal symptoms, use of bismuth subsalicylate or loperamide is recommended. If loperamide is used, an initial dose of 4 mg is given followed by 2 mg after passage of each unformed stool. Antibacterial therapy is only recommended if there is evidence of inflammatory diarrhea (bloody stools or fever) or there are more than two unformed stools daily. The antibacterial agent of choice is usually a fluoroquinolone. Ciprofloxacin given as a single dose of 750 mg or 500 mg three times daily for 3 days is typically effective. In Thailand, Campylobacter jejuni is a common agent and has a high degree of fluoroquinolone resistance. For travelers to Thailand who require antibiotics, azithromycin is recommended with an initial dose of 10 mg/kg on the first day followed by 5 mg/kg on days 2 and 3 if diarrhea persists.

IV-30.  The answer is C. (Chap. 128) Acute bacterial food poisoning occurring 1 to 6 hours after ingestion of contaminated food is most commonly caused by infection with Staphylococcus aureus or Bacillus cereusS. aureus is associated with ingestion of ham, poultry, potato or egg salad, mayonnaise, or cream pastries that have been allowed to remain at room temperature after cooking. B. cereus is classically associated with contaminated fried rice. The symptoms of bacterial food poisoning begin abruptly with nausea, vomiting, abdominal cramping, and diarrhea. However, fever is not a common finding and should cause one to consider other etiologies of vomiting and diarrhea.

IV-31.  The answer is C. (Chap. 128) The patient most likely has food poisoning caused by contamination of the fried rice with Bacillus cereus. This toxin-mediated disease occurs when heat-resistant spores germinate after boiling. Frying before serving may not destroy the preformed toxin. The emetic form of illness occurs within 6 hours of eating and is self-limited. No therapy is necessary unless the patient develops severe dehydration. This patient currently has no symptoms consistent with volume depletion; therefore, she does not need intravenous fluids at present. Sarcoidosis does not predispose patients to infectious diseases.

IV-32.  The answer is E. (Chap. 129) Although frequent nonbloody diarrheal illness is commonly associated with Clostridium difficile infection, other presentations are well described, including fever in 28% of cases, abdominal pain, and leukocytosis. Adynamic ileus is often seen with C. difficile infection, and leukocytosis in this condition should be a clue that C. difficile is at play. Recurrent infection after therapy has been described in 15% to 30% of cases.

IV-33.  The answer is D. (Chap. 129) Clostridium difficile infection is diagnosed by the following means: diarrhea of three or more stools per day for 2 or more days with no other cause plus (1) demonstration of toxin A or B in the stool, (2) polymerase chain reaction for toxin-producing C. difficile of the stool, or (3) demonstration of pseudomembranes on colonoscopy. Although many tests are available, none has adequate sensitivity to definitively rule out C. difficile infection. Thus, empiric therapy is appropriate in a patient (such as patient C) with a high likelihood of C. difficile infection.

IV-34.  The answer is C. (Chap. 129) The patient has evidence of recurrent Clostridium difficile infection, which occurs in up to 30% of treated patients. Because there is no evidence that she has severe infection and this is her first recurrence, the recommended therapy is to retreat with oral metronidazole. Vancomycin is reserved for patients with severe infection either initially or with recurrence. Fecal transplantation, intravenous immunoglobulin, and oral nitazoxanide are all potential therapies for patients with multiple recurrences.

IV-35.  The answer is E. (Chap. 129) Clindamycin, ampicillin, and cephalosporins (including ceftriaxone) were the first antibiotics associated with Clostridium difficile–associated disease and still are. More recently, broad-spectrum fluoroquinolones, including moxifloxacin and ciprofloxacin, have been associated with outbreaks of C. difficile, including outbreaks in some locations of a more virulent strain that has caused severe disease among elderly outpatients. For unclear reasons, β-lactams other than the later generation cephalosporins appear to carry a lesser risk of disease. Penicillin–β-lactamase combination antibiotics appear to have lower risk of C. difficile–associated disease than the other agents mentioned. Cases have even been reported associated with metronidazole and vancomycin administration. Nevertheless, all patients initiating antibiotics should be warned to seek care if they develop diarrhea that is severe or persists for more than 1 day because all antibiotics carry some risk for C. difficile–associated disease.

IV-36.  The answer is A. (Chap. 130) Common causes of urethral discomfort and discharge in men include Chlamydia trachomatis, Neisseria gonorrhoeae, Mycoplasma genitalium, Ureaplasma urealyticum, Trichomonas vaginalis, and herpes simplex virus. Gardnerella spp. is the usual cause of bacterial vaginosis in women and is not a pathogen in men.

IV-37.  The answer is C. (Chap. 130) The patient has symptoms consistent with the urethral syndrome characterized by “internal” dysuria with urgency and frequency and pyuria but no uropathogens at counts of 102/mL or greater in urine. This is most commonly caused by infection with Chlamydia trachomatis or Neisseria gonorrhoeae and can be readily confirmed by nucleic acid amplification testing for these pathogens in the urine. “External” dysuria includes pain in the vulva during urination, often without frequency or urgency. This is found in vulvovaginal candidiasis and herpes simplex infection, which can be visualized on physical examination. Cervical culture would not be useful with her urinary symptoms. Elevated vaginal pH above 5.0 is commonly present in trichomonal vaginitis. Clue cells on vaginal secretion microscopy suggest bacterial vaginosis.

IV-38.  The answer is B. (Chap. 130) Bacterial vaginosis is associated with Gardnerella vaginalis and various anaerobic or noncultured bacteria. It generally has malodorous discharge that is white or gray. There is no external irritation, and pH of vaginal fluid is usually above 4.5; a fishy odor is present with 10% KOH preparation; and microscopy shows clue cells, few leukocytes, and many mixed microbiota. Normal vaginal findings are described in patient D with pH below 4.5 and lactobacilli seen on microscopic examination. A high pH above 5 with external irritation is often found in vulvovaginal candidiasis, but the presence of motile trichomonads is diagnostic for trichomonal vaginitis.

IV-39.  The answer is E. (Chap. 130) In a study of patients with mucopurulent cervicitis seen at a sexually transmitted disease clinic in the 1980s, more than one-third of cervical samples failed to reveal any etiology. In a recent similar study in Baltimore using nucleic acid amplification testing, more than half of the cases were not microbiologically identified. Chlamydia trachomatis is the most frequently diagnosed organism followed by Neisseria gonorrhoeae. Because of the difficulty in making a microbiologic diagnosis, empiric therapy for C. trachomatis and, in areas were N. gonorrhoeae is highly endemic, gonococcus is indicated.

IV-40.  The answer is C. (Chap. 130) The presence of right upper quadrant tenderness in conjunction with classic findings of pelvic inflammatory disease is highly suggestive of Fitz-Hugh-Curtis syndrome or perihepatitis caused by inflammation of the liver capsule caused by either Neisseria gonorrhoeae or Chlamydia trachomatis infection. Although this condition may be easily visualized by laproscopic examination, the resolution of right upper quadrant symptoms with therapy of pelvic inflammatory disease is the more common proof of the diagnosis. The presence of normal liver function testing is reassuring that hepatitis is not present, making hepatitis C virus infection unlikely.

IV-41.  The answer is A. (Chap. 130) The most common causes of genital ulceration are herpes simplex virus, syphilis, and chancroid. Gonorrhea typically manifests as a urethritis, not genital ulcers. Syphilitic ulcers (primary chancre) are firm, shallow single ulcers that are not pustular and are generally not painful. Despite these usual findings, rapid plasma reagin testing is indicated in all cases of genital ulceration given the disparate presentations of Treponema pallidum. Herpes simplex virus ulcers are quite painful but are vesicular rather than pustular. In primary infection, they may be bilateral, but with reactivation, they are generally unilateral. Haemophilus ducreyi, the agent responsible for chancroid, causes multiple ulcers, often starting as pustules, that are soft, friable, and exquisitely tender, as present in this case. Primary infection with HIV usually causes an acute febrile illness, not focal ulcers. The presence of genital ulcers increases the likelihood of acquisition and transmission of HIV.

IV-42.  The answer is F. (Chap. 130) HIV is the leading cause of death in some developing countries. Efforts to decrease transmission include screening and treatment of sexually associated infections. All of the listed conditions have been linked with higher acquisition of HIV based on epidemiologic studies and high biologic plausibility. Up to 50% of women of reproductive age in developing countries have bacterial vaginosis. All of the bacterial infections are curable, and treatment can decrease the frequency of genital herpes recurrences. This highlights an additional reason that primary care doctors should screen for each of these infections in female patients with detailed historic questions, genitourinary and rectal examinations, and evidence-based routine screening for these infections based on age and risk category.

IV-43.  The answer is B. (Chap. 131) Nosocomial infections have reservoirs and sources just as do community-acquired pathogens. In hospitalized patients, cross-contamination (i.e., indirect spread of organisms from one patient to the next) accounts for many nosocomial infections. Although hand hygiene is uniformly recommended for health care practitioners, adherence to hand washing is low often because of time pressure, inconvenience, and skin damage. Because of improved adherence, alcohol-based hand rubs are now recommended for all heath care workers except when hands are visibly soiled or after care of a patient with Clostridium difficile infection, whose spores may not be killed by alcohol and thus require thorough hand wash with soap and water.

IV-44.  The answer is E. (Chap. 132) Ultimately, solid organ transplant patients are at highest risk for infection because of T-cell immunodeficiency from antirejection medicines. As a result, they are also at risk for reactivation of many of the viruses from the herpes virus family, most notably cytomegalovirus, varicella-zoster virus, and Epstein-Barr virus. However, immediately after transplant, these deficits have not yet developed in full. Neutropenia is not common after solid organ transplantation as in bone marrow transplantation. In fact, patients are most at risk of infections typical for all hospitalized patients, including wound infections, urinary tract infection, pneumonia, Clostridium difficile infection, and line-associated infection. Therefore, a standard evaluation of a febrile patient in the first weeks after a solid organ transplant should include a detailed physical examination, blood and urine cultures, urinalysis, chest radiography, and C. difficile stool antigen or toxin studies if warranted, in addition to a transplant-specific evaluation.

IV-45.  The answer is E. (Chap. 132) The patient presents with symptoms suggestive of infection in the middle period after transplantation (1–4 months). In patients with prior cytomegalovirus (CMV) exposure or receipt of CMV-positive organ transplant, this is a period of time when CMV infection is most common. The patient presented here has classic signs of CMV disease with generalized symptoms in addition to dysfunction of her transplanted organ (kidney). Often bone marrow suppression is present, demonstrated here by lymphopenia. Because CMV infection is linked with graft dysfunction and rejection, prophylaxis is frequently used, including valganciclovir. Trimethoprim–sulfamethoxazole is used for Pneumocystis jiroveci prophylaxis, acyclovir generally is used for varicellazoster virus prophylaxis, itraconazole may be considered in patients considered at risk for histoplasmosis reactivation, and isoniazid is used for individuals with recent purified protein derivative conversion or positive chest imaging and no prior treatment.

IV-46.  The answer is E. (Chap. 132) Toxoplasma gondii commonly achieves latency in cysts during acute infection. Reactivation in the central nervous system in AIDS patients is well known. However, Toxoplasma cysts also reside in the heart. Thus, transplanting a Toxoplasma-positive heart into a Toxoplasma-negative recipient may cause reactivation in the months after transplant. Serologic screening of cardiac donors and recipients for T. gondii is important. To account for this possibility, prophylactic doses of trimethoprim–sulfamethoxazole, which is also effective prophylaxis against Pneumocystis and Nocardia spp., is standard after cardiac transplantation. Cardiac transplant recipients, similar to all other solid organ transplant recipients, are at risk of developing infections related to impaired cellular immunity, particularly more than 1 month to 1 year post-transplant. Wound infections or mediastinitis from skin organisms may complicate the early transplant (<1 month) period.

IV-47.  The answer is C. (Chaps. 132 and 204) During the first week after hematopoietic stem cell transplantation, the highest risk of infection comes from aerobic nosocomially acquired bacteria. However, after about 7 days, the risk of fungal infection rises, particularly with prolonged neutropenia. The patient presented here presents with symptoms and signs of a respiratory illness after prolonged neutropenia; fungal infection is high on the differential diagnosis list. The computed tomography scan with nodules and associated halo sign is suggestive of Aspergillus infection. The halo sign often occurs in Aspergillusinfection in the context of an increasing neutrophil count after a prolonged nadir. The serum galactomannan antigen test detects galactomannan, a major component of the Aspergillus cell wall that is released during growth of hyphae. The presence of this compound suggests invasion and growth of the mold. This noninvasive test is receiving wider acceptance in the diagnosis of invasive Aspergillus spp. in immunocompromised hosts. Additionally, galactomannan assays in bronchoalveolar lavage fluid may aid diagnosis of invasive Aspergillus in immunocompromised hosts (American Journal of Respiratory and Critical Care Medicine 177: 27-34, 2008). In the absence of purulent sputum, sputum cultures are unlikely to be helpful. Aspergillus is seldom cultured from the sputum in cases of invasive aspergillosis. Examination of buffy coat is useful for the diagnosis of histoplasmosis, but the focal nodules with halo sign and absence of other systemic symptoms makes histoplasmosis less likely. Legionella spp. and cytomegalovirus pneumonia are generally not associated with nodules and have either lobar infiltrates or diffuse infiltrates.

IV-48.  The answer is E. (Chap. 133) All bacteria, both gram negative and gram positive, have rigid cell walls that protect bacterial intracellular hyperosmolarity from the host environment. Peptidoglycan is the present in both gram-negative and gram-positive bacteria, but only gram-negative bacteria have an additional outer membrane external to peptidoglycan. Many antibiotics target cell wall synthesis and thus lead to inhibition of growth or cell death. These antibiotics include bacitracin, glycopeptides such as vancomycin, and β-lactam antibiotics. Macrolides such as azithromycin, lincosamides (clindamycin), linezolid, chloramphenicol, aminoglycosides such as tobramycin, mupirocin, and tetracycline all inhibit protein synthesis. Sulfonamides and trimethoprim interrupt cell metabolism. Rifampin and metronidazole alter nucleic acid synthesis. The quinolones, such as ciprofloxacin, and novobiocin inhibit DNA synthesis. Finally, polymixins, gramicid, and daptomycin disrupt the cellular membrane.

IV-49.  The answer is B. (Chap. 133) The patient presents with evidence of methicillin-resistant Staphylococcus aureus–associated soft tissue infection that has failed therapy with clindamycin. Linezolid is an appropriate choice for antibiotic coverage in this situation. Subsequent development of neurologic symptoms, including agitated delirium, evidence of autonomic instability coupled with tremor, muscular rigidity, hyperreflexia, and clonus, suggests serotonin syndrome. Because linezolid is a monoamine oxidase inhibitor, it interacts with selective serotonin reuptake inhibitors and can cause serotonin syndrome. Other potential triggers include tyramine-rich foods and sympathomimetics such as phenylpropanolamine. The other drug–drug combinations in the answer choices are not described to be associated with serotonin syndrome.

IV-50.  The answer is A. (Chap. 134) Pneumococcal infections, particularly pneumonia, remain a worldwide public health problem. Intermittent colonization of the nasopharynx by pneumococcus transmitted by respiratory droplet is common and is the likely reservoir for invasive disease. Infants and elderly adults are at greatest risk of developing invasive pneumococcal disease (IPD) and death. In the developed world, children are the most common source of pneumococcal transmission. By 1 year of age, 50% of children have had at least one episode of colonization. Prevalence studies show carriage rates of 20% to 50% in children up to 5 years old and up to 15% for adults. These numbers approach 90% for children and 40% for adults in the developing world. Pneumococcal vaccination has dramatically impacted the epidemiology with reduced IPD in the United States attributable to reductions in serotypes included in the vaccine. Similar reductions have been observed in other countries implementing routine childhood vaccinations; however, in certain populations (Alaska native populations and United Kingdom), the reduction in vaccine covered serotype cases has been offset by increases in nonvaccine serotypes. Case fatality rates caused by pneumococcal pneumonia vary by host factors, age, and access to care. Interestingly, there appears to be no reduction in case fatality during the first 24 hours of hospitalization since the introduction of antibiotics. This is likely because of the development of severe multiorgan failure as a result of severe infection. Appropriate care in an intensive setting can reduce case fatality rate for severe infection. Outbreaks of disease are well recognized in crowded settings with susceptible individuals, such as infant daycare facilities, military barracks, and nursing homes. Furthermore, there is a clear association between preceding viral respiratory disease (especially but not exclusively influenza) and risk of secondary pneumococcal infections. The significant role of pneumococcal pneumonia in the morbidity and mortality associated with seasonal and pandemic influenza is increasingly recognized.

IV-51.  The answer is C. (Chap. 134) This elderly man presents with a typical story of pneumococcal pneumonia. His age, chronic conditions, and nursing home residence put him a high risk of invasive disease acquisition. Outbreaks commonly occur in crowded environments or nursing homes often after preceding upper respiratory or influenza viral infections and are spread through respiratory droplets. Although the differential diagnosis includes viral pathogens, mycoplasmas, Haemophilus influenzaeKlebsiella pneumoniaeStaphylococcus aureus, and Legionella spp., pneumococcal disease remains most common in this demographic. Blood cultures, even in severe disease are positive in fewer than 30% of cases. Diagnosis relies on a positive culture from blood or sputum or a positive urinary antigen test result. Urinary antigen testing has a high positive predictive value in adults because the intermittent colonization rate is low. The urinary antigen test is less specific for invasive disease in children who may be colonized. The radiograph in this case is typical for consolidation (with air bronchograms) of the right lower lobe. The clear right heart border and confinement below the major fissure suggests unilobar disease. With time and hydration, further radiologic extension may be apparent. Parapneumonic (noninfected) pleural effusions are common. The most common focal complication of pneumococcal disease is empyema, occurring in approximately 5% of cases. It should be suspected in cases of new or enlarging pleural effusion or persistent fever particularly after initiation of therapy. Meningitis may occur from hematogenous spread in conjunction with pneumonia or may be the sole presenting syndrome of pneumococcal infection. Pneumococcal resistance to penicillin has increased dramatically since the 1990s, and it is not recommended for empiric therapy of acute pneumonia. However, in culture-proven cases with minimal inhibitory concentration below 2 μg/mL, penicillin may remain an appropriate therapeutic choice for severe disease or meningitis.

IV-52 and IV-53. The answers are E and A, respectively. (Chaps. 134 and 143) In a previously healthy student, particularly one living in a dormitory, Streptococcus pneumoniae and Neisseria meningitidesare the pathogens most likely to be causing community-acquired bacterial meningitis. As a result of the increasing prevalence of penicillin- and cephalosporin-resistant streptococci, initial empirical therapy should include a third- or fourth-generation cephalosporin plus vancomycin. Dexamethasone has been shown in children and adults to decrease meningeal inflammation and unfavorable outcomes in acute bacterial meningitis. In a recent study of adults, the effect on outcome was most notable in patients with S. pneumoniae infection. The first dose (10 mg IV) should be administered 15 to 20 minutes before or with the first dose of antibiotics and is unlikely to be of benefit unless it is begun 6 hours after the initiation of antibiotics. Dexamethasone may decrease the penetration of vancomycin into the cerebrospinal fluid.

IV-54.  The answer is B. (Chap. 135) Although new genetic diagnostic kits for distinguishing microbes are becoming more common, basic biochemical characterization of bacterial pathogens is still widely used in microbiology laboratories. Clinicians should be familiar with the most common of these techniques when interpreting laboratory results. Whereas all staphylococci are catalase positive, streptococci are catalase negative. Whereas S. aureus are coagulase positive, Staphylococcus epidermidis (as well as S. hominis, S. saprophyticus, and others) are coagulase negative. This is the initial result that can make this important clinical distinction. Lactose fermentation is used to distinguish many gram-negative bacteria. Salmonella, Proteus, and Shigella spp. and Pseudomonas aeruginosa are unable to ferment lactose. The oxidase test is commonly used to identify P. aeruginosa. The urease test is used to identify Proteus spp., Helicobacter spp., and other gram-negative organisms.

IV-55.  The answer is D. (Chap. 135) The major clinical concern in this patient is epidural abscess or vertebral osteomyelitis, as well as line infection caused by Staphylococcus aureus. These concerns plus her significant likelihood of clinical deterioration necessitate close inpatient monitoring. Empiric therapy for methicillin-resistant S. aureus and gram-negative bacteria is warranted after obtaining blood cultures pending further evaluation. Metastatic seeding during S. aureus bacteremia has been estimated to occur as often as 30% of the time. The bones, joints, kidneys, and lungs are the most common sites. Metastatic infection to the spine should be evaluated in an emergent fashion with magnetic resonance imaging. The dialysis catheter should be removed because it is infected based on clinical examination. Infective endocarditis is a major concern. This diagnosis is based on positive blood culture results and either a vegetation on echocardiogram, new pathologic murmur, or evidence of septic embolization on physical examination. A transthoracic echocardiogram is warranted in the evaluation for endocarditis (a disease that this patient is at risk for). However, it need not be ordered emergently because it will not impact management during the initial phase of hospitalization. Moreover, because the diagnosis can only be established in the presence of positive blood cultures (or in rare cases serology of a difficult-to-culture organism), a rational approach is to await positive blood cultures before ordering an echocardiogram.

IV-56.  The answer is C. (Chap. 135) In the past 10 years, numerous outbreaks of community-based infection caused by methicillin-resistant Staphylococcus aureus (MRSA) in individuals with no prior medical exposure have been reported. These outbreaks have taken place in both rural and urban settings in widely separated regions throughout the world. The reports document a dramatic change in the epidemiology of MRSA infections. The outbreaks have occurred among such diverse groups as children, prisoners, athletes, Native Americans, and drug users. Risk factors common to these outbreaks include poor hygienic conditions, close contact, contaminated material, and damaged skin. The community-associated infections have been caused by a limited number of MRSA strains. In the United States, strain USA300 (defined by pulsed-field gel electrophoresis) has been the predominant clone. Although the majority of infections caused by this community-based clone of MRSA have involved the skin and soft tissue, 5% to 10% have been invasive, including severe necrotizing lung infections, necrotizing fasciitis, infectious pyomyositis, endocarditis, and osteomyelitis. The most feared complication is a necrotizing pneumonia that often follows influenza upper respiratory infection and can affect previously healthy people. This pathogen produces the Panton-Valentine leukocidin protein that forms holes in the membranes of neutrophils as they arrive at the site of infection and serves as marker for this pathogen. An easy way to identify this strain of MRSA is its sensitivity profile. Unlike MRSA isolates of the past, which were sensitive only to vancomycin, daptomycin, quinupristin–dalfopristin, and linezolid, CA-MRSA are almost uniformly susceptible to trimethoprim–sulfamethoxazole and doxycycline as well. The organism is also usually sensitive to clindamycin. The term community acquired has probably outlived its usefulness because this isolate has become the most common S. aureus isolate causing infection in many hospitals around the world.

IV-57.  The answer is A. (Chap. 135) Vancomycin remains the drug of choice for methicillin-resistant Staphylococcus aureus (MRSA). New patterns of staphylococcal resistance are developing including a strain (VISA, first reported in Japan) that shows vancomycin intermediate resistance. Of the drugs listed all have activity against MRSA. Telavancin is a derivative of vancomycin that is approved by the U.S. Food and Drug Administration for complicated skin and soft tissue infections. VISA strains appear to be susceptible. Linezolid is bacteriostatic against staphylococci and has oral and parenteral formulations. Quinupristin–dalfopristin is bactericidal against all staphylococcal strains, including VISA. It has been used in severe MRSA infections. Daptomycin is not effective for respiratory infections. It can be used for bacteremia and right-sided endocarditis.

IV-58.  The answer is E. (Chap. 135) Probably because of its ubiquity and ability to stick to foreign surfaces, Staphylococcus epidermidis is the most common cause of infections of central nervous system shunts as well as an important cause of infections on artificial heart valves and orthopedic prostheses. Corynebacterium spp. (diphtheroids), similar to S. epidermidis, colonize the skin. When these organisms are isolated from cultures of shunts, it is often difficult to be sure if they are the cause of disease or simply contaminants. Leukocytosis in cerebrospinal fluid, consistent isolation of the same organism, and the character of a patient’s symptoms are all helpful in deciding whether treatment for infection is indicated.

IV-59.  The answer is D. (Chap. 135) This patient has infectious pyomyositis, a disease of the tropics and of immunocompromised hosts such as patients with poorly controlled diabetes mellitus or AIDS. The pathogen is usually Staphylococcus aureus. Management includes aggressive debridement, antibiotics, and attempts to reverse the patient’s immunocompromised status. Clostridium perfringens may cause gas gangrene, particularly in devitalized tissues. Streptococcal infections may cause cellulitis or an aggressive fasciitis, but the presence of abscesses in a patient with poorly controlled diabetes makes staphylococcal infection more likely. Polymicrobial infections are common in diabetic ulcers, but in this case, the imaging and physical examination show intramuscular abscesses.

IV-60.  The answer is D. (Chap. 136) Recurrent episodes of rheumatic fever are most common in the first 5 years after the initial diagnosis. Penicillin prophylaxis is recommended for at least this period. After the first 5 years, secondary prophylaxis is determined on an individual basis. Ongoing prophylaxis is currently recommended for patients who have had recurrent disease, have rheumatic heart disease, or work in occupations that have a high risk for reexposure to group A streptococcal infection. Prophylactic regimens are penicillin V, PO 250 mg bid; benzathine penicillin, 1.2 million units IM every 4 weeks; and sulfadiazine, 1 g PO daily. Polyvalent pneumococcal vaccine has no cross-reactivity with group A streptococcus.

IV-61.  The answer is E. (Chap. 136) Necrotizing fasciitis involves the superficial or deep fascia (or both) investing the muscles of an extremity or the trunk. The source of the infection is either the skin, with organisms introduced into tissue through trauma (sometimes trivial), or the bowel flora, with organisms released during abdominal surgery or from an occult enteric source, such as a diverticular or appendiceal abscess. The inoculation site may be unapparent and is often some distance from the site of clinical involvement; for example, the introduction of organisms via minor trauma to the hand may be associated with clinical infection of the tissues overlying the shoulder or chest. Cases originating from the skin are most commonly caused by infection with Streptococcus pyogenes (group A streptococcus), sometimes with Staphylococcus aureus coinfection. In this case, the presence of fasciitis without myositis (which is more commonly caused by staphylococci) makes S. pyogenes the most likely organism. The onset of disease is often acute and the course fulminant. Although pain and tenderness may be severe, physical findings may be subtle initially. Local anesthesia (caused by cutaneous nerve infarction) and skin mottling are late findings. Cases associated with the bowel flora are usually polymicrobial, involving a mixture of anaerobic bacteria (e.g., Bacteroides fragilis or anaerobic streptococci) and facultative organisms (usually gram-negative bacilli). Necrotizing fasciitis is a surgical emergency with extensive debridement potentially life saving. At surgery, the extent of disease is typically more extensive than clinically or radiologically indicated. Antibiotic therapy is adjunctive. Patients with necrotizing fasciitis may develop streptococcal toxic shock syndrome. Streptococcus pneumoniae and Staphylococcus epidermidis are not causes of necrotizing fasciitis. Clostridium difficile causes antibiotic-associated colitis.

IV-62.  The answer is D. (Chap. 136) Streptococcus agalactiae is the only species of group B streptococci (GBS) and is a major cause of sepsis and meningitis in neonates. The infection in neonates is acquired by passage through a maternally colonized birth canal. Although 40% to 50% of neonates born of colonized mothers will themselves become colonized, only 1% to 2% develop infection. GBS is also a cause of peripartum fever and can cause significant endometritis or chorioamnionitis. Vaginal swab culture results are typically positive. Risk factors for infection of the mother and child include premature labor and prolonged rupture of membranes. Treatment is with penicillin. The Centers for Disease Control and Prevention recommends screening pregnant women for anogenital colonization at 35 to 37 weeks of pregnancy by a swab culture of the lower vagina and anorectum. Intrapartum chemoprophylaxis is recommended for culture-positive women.

IV-63.  The answer is B. (Chap. 137) Enterococci are the second most common organisms (after staphylococci) isolated from hospital-associated infections in the United States. Although Enterococcus faecalis remains the predominant species recovered from nosocomial infections, the isolation of E. faecium has increased substantially in the past 10 to 15 years. More than 80% of E. faecium isolates recovered in U.S. hospitals are resistant to vancomycin and more than 90% are resistant to ampicillin. The most important factors associated with vancomycin-resistant enterococci (VRE) colonization and persistence in the gut include prolonged hospitalization; long courses of antibiotic therapy; hospitalization in long-term-care facilities, surgical units, or intensive care units; organ transplantation; renal failure (particularly in patients undergoing hemodialysis) or diabetes; high APACHE scores; and physical proximity to patients infected or colonized with VRE or to these patients’ rooms. VRE infection increases the risk of death, independent of the patient’s clinical status, over that among individuals infected with a glycopeptide-susceptible enterococcal strain.

IV-64.  The answer is B. (Chap. 137) This patient has enterococcal endocarditis, which often occurs in patients with underlying gastrointestinal or genitourinary pathology. Enterococcus faecalis is a more common causative organism than E. faecium in community-acquired endocarditis. Patients tend to more commonly be men with underlying chronic disease. The typical presentation is one of subacute bacterial endocarditis and with involvement of the mitral or aortic valves. Prolonged therapy beyond 4 to 6 weeks is often necessary for organisms with drug resistance. Complications requiring valve replacement are common. Enterococci are intrinsically resistant or tolerant to several antimicrobial agents (with tolerance defined as lack of killing by drug concentrations 16 times higher than the minimal inhibitory concentration). Monotherapy for endocarditis with a β-lactam antibiotic (to which many enterococci are tolerant) has produced disappointing results with low cure rates at the end of therapy. However, the addition of an aminoglycoside to a cell wall–active agent (a β-lactam or a glycopeptide) increases cure rates and eradicates the organisms; moreover, this combination is synergistic and bactericidal in vitro. Therefore, combination therapy with a cell wall–active agent and an aminoglycoside is the standard of care for endovascular infections caused by enterococci. This synergistic effect can be explained, at least in part, by the increased penetration of the aminoglycoside into the bacterial cell, presumably as a result of cell wall alterations attributable to the β-lactam or glycopeptide.

IV-65.  The answer is D. (Chap. 137) Resistance to ampicillin and vancomycin is far more common in strains of Enterococcus faecium than E. faecalis. Linezolid and quinupristin–dalfopristin are approved by the U.S. Food and Drug Administration for the treatment of some vancomycin-resistant enterococci (VRE) infections. Linezolid is not bactericidal, and its use in severe endovascular infections has produced mixed results; therefore, it is recommended only as an alternative to other agents. Quinupristin–dalfopristin is not active against most E. faecalis isolates. Resistance to VRE strains of E. faecium is also emerging with increasing usage. Cephalosporins are generally inactive against enterococcal infections.

IV-66.  The answer is B. (Chap. 138) Rhodococcus spp., including R. equi, are phylogenetically related to the corynebacteria. They predominantly cause necrotizing lung infections in immunocompromised hosts. The differential diagnosis of the cavitating lung lesions includes tuberculosis, Nocardia infection, and septic emboli. The organisms can initially be mistaken for corynebacteria, but they should not be misconstrued as skin contaminants. The organism is routinely susceptible to vancomycin, which is considered the drug of choice. Infection caused by R. equi has also been treated successfully with antibiotics that penetrate intracellularly, including macrolides, clindamycin, trimethoprim–sulfamethoxazole, rifampin, tigecycline, and linezolid. β-Lactam antibiotics are not effective.

IV-67.  The answer is D. (Chap. 139) Listeria meningitis typically affects elderly and the chronically ill individuals. It is frequently a more subacute (developing over days) illness than other etiologies of bacterial meningitis. It may be mistaken for aseptic meningitis. Meningeal signs, including nuchal rigidity, are less common, as is photophobia, than in other more acute causes of bacterial meningitis. Typically, white blood cell (WBC) counts in the cerebrospinal fluid range from 100 to 5000/μL with a less pronounced neutrophilia. About 75% of patients will have a WBC count below 1000/μL. Gram stain is only positive in 30% to 40% of cases. Case fatality rates are approximately 20%.

IV-68.  The answer is A. (Chap. 139) Listeria monocytogenes causes gastrointestinal (GI) illness via ingestion of food that has been contaminated with high concentrations of bacteria. The bacteria may survive and multiply at refrigeration temperatures; therefore, deli meats, soft cheeses, hot dogs, and milk are common sources. The attack rate is very high, with close to 100% of exposed patients experiencing symptoms. Symptoms develop within 48 hours of exposure, and there is no prolonged asymptomatic carrier state. Person-to-person spread (other than vertically from mother to fetus) does not appear to occur during outbreaks. Although the bacteria have several virulence factors that lead to clinical symptoms, the organism, and not a specific toxin, mediates infection. A large inoculum is necessary to produce symptoms. Surveillance studies show that fewer than 5% of asymptomatic adults have positive stool cultures, and fecal–oral spread is not common. Typical symptoms, including fever, are as described in the case above. Patients with isolated GI illness do not require antibiotics.

IV-69.  The answer is B. (Chap. 139) Listeria bacteremia in pregnancy is a relatively rare but serious infection both for the mother and fetus. Vertical transmission may occur, with 70% to 90% of fetuses developing infection from their mothers. Preterm labor is common. Prepartum treatment of the mother increases the chances of a healthy delivery. Mortality among fetuses approaches 50% and is much lower in neonates receiving appropriate antibiotics. First-line therapy is with ampicillin, with gentamicin often added for synergy. This recommendation is the same for the mother and child. In patients with true penicillin allergy, the therapy of choice is trimethoprim–sulfamethoxazole. There are case reports of successful therapy with vancomycin, imipenem, linezolid, and macrolides, but there is not enough clinical evidence, and there have been some reports of failure that maintain ampicillin as recommended therapy.

IV-70.  The answer is D. (Chap. 140) Tetanus is an acute disease manifested by skeletal muscle spasm and autonomic nervous system disturbance. It is caused by a powerful neurotoxin produced by the bacterium Clostridium tetaniand is now a rare disease because of widespread vaccination. There were fewer than 50 cases reported recently in the United States, but there is a rising frequency in drug users. Older patients may be at higher risk because of waning immunity. The differential diagnosis of a patient presenting with tetanus includes strychnine poisoning and drug-related dystonic reactions. The diagnosis is clinical. Cardiovascular instability is common because of autonomic dysfunction and is manifest by rapid fluctuation in heart rate and blood pressure. Wound culture results are positive in approximately 20% of cases. Metronidazole or penicillin should be administered to clear infection. Tetanus immune globulin is recommended over equine antiserum because of a lower risk of anaphylactic reactions. Recent evidence suggests that intrathecal administration is efficacious in inhibiting disease progression and improving outcomes. Muscle spasms may be treated with sedative drugs. With effective supportive care and often respiratory support, muscle function recovers after clearing the toxin with no residual damage.

IV-71.  The answer is B. (Chap. 141) This patient most likely has wound botulism. The use of “black-tar” heroin has been identified as a risk factor for this form of botulism. Typically, the wound appears benign, and unlike in other forms of botulism, gastrointestinal symptoms are absent. Symmetric descending paralysis suggests botulism, as does cranial nerve involvement. This patient’s ptosis, diplopia, dysarthria, dysphagia, lack of fevers, normal reflexes, and lack of sensory deficits are all suggestive. Botulism can be easily confused with Guillain-Barré syndrome (GBS), which is often characterized by an antecedent infection and rapid, symmetric ascending paralysis and treated with plasmapheresis. The Miller Fischer variant of GBS is known for cranial nerve involvement with ophthalmoplegia, ataxia, and areflexia being the most prominent features. Elevated protein in the cerebrospinal fluid also favors GBS over botulism. Both botulism and GBS can progress to respiratory failure, so making a diagnosis by physical examination is critical. Other diagnostic modalities that may be helpful are wound culture, serum assay for toxin, and examination for decreased compound muscle action potentials on routine nerve stimulation studies. Patients with botulism are at risk of respiratory failure caused by respiratory muscle weakness or aspiration. They should be followed closely with oxygen saturation monitoring and serial measurement of forced vital capacity.

IV-72.  The answer is E. (Chap. 142) Clostridia are gram-positive, spore-forming obligate anaerobes that reside normally in the gastrointestinal (GI) tract. Several clostridial species can cause severe disease. Clostridium perfringens, which is the second most common clostridial species to normally colonize the GI tract, is associated with food poisoning, gas gangrene, and myonecrosis. C. septicum is seen often in conjunction with GI tumors. C. sordellii is associated with septic abortions. All can cause a fulminant overwhelming bacteremia, but this condition is rare. The fact that this patient is well several days after his acute complaints rules out this fulminant course. A more common scenario is transient, self-limited bacteremia caused by transient gut translocation during an episode of gastroenteritis. There is no need to treat when this occurs, and no further workup is necessary. Clostridium spp. sepsis rarely causes endocarditis because overwhelming disseminated intravascular coagulation and death occur so rapidly. Screening for GI tumor is warranted when C. septicum is cultured from the blood or a deep wound infection.

IV-73.  The answer is A. (Chap. 143) Neisseria meningitidis is an effective colonizer of the human nasopharynx, with asymptomatic infection rates of greater than 25% described in some series of adolescents and young adults and among residents of crowded communities. Despite the high rates of carriage among adolescents and young adults, only 10% of adults carry meningococci, and colonization is very rare in early childhood. Colonization should be considered the normal state of meningococcal infection. Meningeal pharyngitis rarely occurs. Meningococcal disease occurs when a virulent form of the organism invades a susceptible host. The most important bacterial virulence factor relates to the presence of the capsule. Unencapsulated forms of N. meningitides rarely cause disease. A nonblanching petechial or purpuric rash occurs in more than 80% of cases of meningococcal disease. Of patients with meningococcal disease, 30% to 50% present with meningitis, approximately 40% with meningitis plus septicemia, and 20% with septicemia alone. Patients with complement deficiency, who are at highest risk of developing meningococcal disease, may develop chronic meningitis.

IV-74.  The answer is B. (Chap. 143) Close contacts of individuals with meningococcal disease are at increased risk of developing secondary disease with reports of secondary cases in up to 3% of primary cases. The rate of secondary cases is highest during the week after presentation of the index case with most cases presenting within 6 weeks. Increased risk remains for up to 1 year. Prophylaxis is recommended for persons who are intimate or household contacts of the index case and health care workers who have been directly exposed to respiratory secretions. Mass prophylaxis is not usually offered. The aim of prophylaxis is to eradicate colonization of close contacts with the strain that has caused invasive disease. Prophylaxis should be given as soon as possible to all contacts at the same time to avoid recolonization. Waiting for culture is not recommended. Ceftriaxone as a single dose is currently the most effective option in reducing carriage. Rifampin is no longer the optimal agent because it requires multiple doses and fails to eliminate carriage in up to 20% of cases. In some countries, ciprofloxacin or ofloxacin is used, but resistance has been reported in some areas. Current conjugated vaccines do not include N. meningitides serotype B. Most sporadic cases in the United States are now caused by this serotype. Vaccination should be offered in cases of meningococcal disease caused by documented infection by a serotype included in the current vaccine.

IV-75.  The answer is D. (Chap. 144) Because of emerging resistance, treatment recommendations for gonorrhea require frequent updating. Fluoroquinolones and penicillin are no longer generally recommended in the United States because of resistance. Current effective therapies use single-dose therapies to maximize adherence. Oral cefixime or intramuscular ceftriaxone are effective for urethritis, cervicitis, and proctitis. Azithromycin is no longer effective for gonorrhea because of resistance, but it should be administered because of the presumption of chlamydial co-infection. Doxycycline also an option for co-treatment in nonpregnant women. Patients with uncomplicated infection who receive therapy do not require a test of cure. Patients should be instructed to contact sexual partners for screening and therapy. Recent studies have demonstrated that the provision of medications or prescriptions to treat gonorrhea and chlamydia in sexual partners diminishes the risk of reinfection in the affected patient.

IV-76.  The answer is D. (Chap. 145) Generally thought of as a disease of children, epiglottitis is also a disease of adults since the wide use of Haemophilus influenzae type B vaccination. Epiglottitis can cause life-threatening airway obstruction caused by cellulitis of the epiglottis and supraglottic tissues, classically caused by H. influenzae type B infection. However, other organisms are also common causes, including nontypeable H. influenzae, Streptococcus pneumoniae, H. parainfluenzaeStaphylococcus aureus, and viral infection. The initial evaluation and treatment for epiglottitis in adults includes airway management and intravenous antibiotics. The patient presented here is demonstrating signs of impending airway obstruction with stridor, inability to swallow secretions, and use of accessory muscles of inspiration. A lateral neck radiograph shows the typical thumb sign indicative of a swollen epiglottis. In addition, the patient has evidence of hypoventilation with carbon dioxide retention. Thus, in addition to antibiotics, this patient should also be intubated and mechanically ventilated electively under a controlled setting because he is at high risk for mechanical airway obstruction. Antibiotic therapy should cover the typical organisms outlined above and include coverage for oral anaerobes.

In adults presenting without overt impending airway obstruction, laryngoscopy would be indicated to assess airway patency. Endotracheal intubation would be recommended for those with more than 50% airway obstruction. In children, endotracheal intubation is often recommended because laryngoscopy in children has provoked airway obstruction to a much greater degree than adults, and increased risk of mortality has been demonstrated in some series in children when the airway is managed expectantly.

IV-77.  The answer is A. (Chap. 145) Moraxella catarrhalis is an unencapsulated gram-negative diplococcus that causes upper respiratory tract disease in children and adults. Some studies suggest that the widespread implementation of pneumococcal vaccination has increased the prevalence of M. catarrhalis and related organisms as a cause of disease. M. catarrhalis causes approximately 10% to 20% of cases of otitis media in children, often after a preceding viral infection. It is the second most common proven bacterial cause of chronic obstructive pulmonary disease (COPD) exacerbations after Haemophilus influenzae. Clinical features do not distinguish among the various bacterial and viral causes of COPD exacerbations. In most cases, a proven cause is not found. Currently, most strains of M. catarrhalisdemonstrate β-lactamase activity. Recommended therapy includes agents effective for upper respiratory, sinus, and otic infections presumed to be caused by M. catarrhalis, H. influenzae, and S. pneumoniae, including amoxicillin–clavulanic acid, extended-spectrum cephalosporins, azithromycin, clarithromycin, and flouroquinolones.

IV-78.  The answer is C. (Chap. 146) This patient has subacute bacterial endocarditis caused by infection with one of the HACEK organisms. The HACEK organisms (Haemophilus, Actinobacillus, Cardiobacterium, Eikenella, and Kingella spp.) are gram-negative rods that reside in the oral cavity. They are responsible for about 3% of cases of infective endocarditis in most series. They are the most common cause of gram-negative endocarditis in nondrug abusers. Most patients have a history of poor dentition or a recent dental procedure. Often, patients are initially diagnosed with culture-negative endocarditis because these organisms may be slow growing and fastidious. Cultures must be specified for prolonged culture of fastidious organisms. HACEK endocarditis is typically subacute, and the risk of embolic phenomena to the bone, skin, kidneys, and vasculature is high. Vegetations are seen on approximately 85% of transthoracic echocardiograms. Cure rates are excellent with antibiotics alone; native valves require 4 weeks, and prosthetic valves require 6 weeks of treatment. Ceftriaxone is the treatment of choice, with ampicillin–gentamicin as an alternative. Sensitivities may be delayed because of the organisms’ slow growth.

IV-79.  The answer is B. (Chap. 146) Capnocytophaga canimorsus is the most likely organism to have caused fulminant disease in this patient with alcoholism after a dog bite. Patients with a history of alcoholism, asplenia, and glucocorticoid therapy are at risk of developing disseminated infection, sepsis, and disseminated intravascular coagulation. Because of increasing β-lactamase expression, recommended treatment is with ampicillin–sulbactam or clindamycin. One of these therapies should be administered to asplenic patients with a dog bite. Other species of Capnocytophaga cause oropharyngeal disease and can cause sepsis in neutropenic patients, particularly in the presence of oral ulcers. Eikenella and Haemophilus spp. are common mouth flora in humans but not in dogs. Staphylococcus spp. can cause sepsis but is less likely in this scenario.

IV-80.  The answer is B. (Chap. 147) Despite antibiotic treatment, pneumonia from all causes remains a major source of mortality in the United States. Mortality from Legionella pneumonia varies from 0% to 11% in treated immunocompetent patients to about 30% if not treated effectively. Because Legionella spp. is an intracellular pathogen, antibiotics that reach intracellular MICs are most likely to be effective. Newer macrolides and quinolones are antibiotics of choice and are effective as monotherapy. Doxycycline and tigecycline are active in vitro. Anecdotal reports have described successes and failures with trimethoprim–sulfamethoxazole and clindamycin. Aztreonam, most β-lactams, and cephalosporins cannot be considered effective therapy for Legionella pneumonia. For severe cases, rifampin may be initially added to azithromycin or a fluoroquinolone.

IV-81.  The answer is C. (Chap. 147) Legionella is an intracellular pathogen that enters the body through aspiration or direct inhalation. Numerous prospective studies have found it is one of the four most common causes of community-acquired pneumonia with Streptococcus pneumoniaeHaemophilus influenzae, and Chlamydia pneumoniae accounting for 2% to 9% of cases. Postoperative patients are at risk because of an increased risk of aspiration. Cell-mediated immunity is the primary host defense against Legionella spp., and patients with HIV or those who take glucocorticoids are at risk based on their depressed cell-mediated immune function. Alveolar macrophages phagocytose Legionella spp. Smokers and those with chronic lung disease are at risk given their poor local immune responses and decreased ability for widespread phagocytosis. Neutrophils play a comparatively small role in the host defense against Legionella spp., and those with neutropenia are not predisposed to Legionella infection.

IV-82.  The answer is D. (Chap. 147) Legionella urine antigen is detectable within 3 days of symptoms and will remain positive for 2 months. It is not affected by antibiotic use. The urinary antigen test is formulated to detect only L. pneumophila (which causes 80% of Legionella infections) but cross-reactivity with other Legionella spp. has been reported. The urinary test result is sensitive and highly specific. Typically, Gram staining of specimens from sterile sites such as pleural fluid show numerous white blood cells but no organisms. However, Legionella spp. may appear as faint, pleomorphic gram-negative bacilli. Legionella spp. may be cultured from sputum even when epithelial cells are present. Cultures, grown on selective media, take 3 to 5 days to show visible growth. Antibody detection using acute and convalescent serum is an accurate means of diagnosis. A fourfold rise is diagnostic, but this takes up to 12 weeks, so it is most useful for epidemiologic investigation. Legionella polymerase chain reaction has not been shown to be adequately sensitive and specific for clinical use. It is used for environmental sampling.

IV-83.  The answer is C. (Chap. 148) Pertussis caused by the gram-negative bacteria Bordetella pertussis, is an upper respiratory infection characterized by a violent cough. Its prevalence has been dramatically reduced, but not eliminated, by widespread infant vaccination. It causes an extremely morbid and often mortal disease in infants younger than 6 months old, particularly in the developing world. The prevalence appears to be increasing in young adults and adolescents because of waning immunity. Some are recommending booster vaccination after 10 years. B. pertussis is also a growing pathogen in patients with chronic obstructive pulmonary disease. The clinical manifestations typically include a persistent, episodic cough developing a few days after a cold-like upper respiratory infection. The cough may become persistent. It often wakes the patient from sleep and results in posttussive vomiting. An audible whoop is only present in fewer than half of cases. Diagnosis is with nasopharyngeal culture or DNA probe testing. There is no urinary antigen testing available. The goal of antibiotic therapy is to eradicate the organism from the nasopharynx. It does not alter the clinical course. Macrolide antibiotics are the treatment of choice. Pneumonia is uncommon with B. pertussis. Cold agglutinins may be positive in infection with Mycoplasma pneumoniae, which is on the differential diagnosis of B. pertussis.

IV-84.  The answer is C. (Chap. 149) Enterotoxigenic Escherichia coli is the most common cause of traveler’s diarrhea, accounting for 50% of cases in Latin America and 15% in Asia. Enterotoxigenic and enteroaggregative E. coliare the most common isolates from persons with classic secretory traveler’s diarrhea. Bloody stools, fecal leukocytes, and fever are typically absent. Symptoms typically last less than 3 days. The spectrum of disease can range from mild to severe with life-threatening volume loss. Treatment of frequent watery stools caused by presumed E. coli infection with ciprofloxacin, or because of concerns regarding increasing ciprofloxacin resistance, azithromycin may shorten the duration of symptoms. Entamoeba histolytica and Vibrio cholerae account for smaller percentages of traveler’s diarrhea in Mexico. Campylobacter infection is more common in Asia and during the winter in subtropical areas. Giardia infection is associated with contaminated water supplies and in campers who drink from freshwater streams.

IV-85.  The answer is D. (Chap. 149) β-lactamases are a major source of antibiotic resistance in gram-negative bacilli. Many gram-negative bacteria produce broad-spectrum β-lactamases that confer resistance to penicillins and first-generation cephalosporins. The addition of clavulanate, a β-lactamase inhibitor, to an antibiotic regimen is often enough to overcome this resistance. Extended-spectrum β-lactamases (ESBLs), however, lead to resistance to all β-lactam drugs, including third- and fourth-generation cephalosporins. ESBL-producing genes can be acquired by gram-negative bacteria via plasmids and are becoming increasingly prevalent in hospitals worldwide. Klebsiella and Escherichia coli are the most common bacteria that acquire ESBLs, although it can be seen in many other gram-negatives, including Serratia, ProteusEnterobacter, and Citrobacter spp. The most common scenario for the development of ESBL-gram negative organisms in the hospital is prevalent use of third-generation cephalosporins. Carbapenems should be considered first-line antibiotics for these bacteria. Macrolides and quinolones have different mechanisms of action than β-lactam antibiotics and do not apply selective pressure to generate ESBL-producing bacteria.

IV-86.  The answer is B. (Chap. 149) Ecoli is the etiologic agent in 85% to 95% of uncomplicated urinary tract infections (UTIs) that occur in premenopausal women. Uncomplicated cystitis is the most common UTI syndrome. About 20% of women will develop a recurrence in 1 year after their initial UTI. Pregnant women are at high risk of cystitis developing into pyelonephritis. Proteus infection represents only 1% to 2% of uncomplicated UTIs. Proteusinfection causes 20% to 45% of UTIs in patients with long-term bladder catheterization. Klebsiella spp. also accounts for only 1% to 2% of uncomplicated UTIs; however, it is responsible for 5% to 17% of complicated UTIs. Enterobacter spp. is a rare cause of infection outside of the hospital. Candida spp. is most often a genitourinary colonizer in healthy patients and is rarely the cause of infection.

IV-87.  The answer is A. (Chap. 149) Shiga toxic and enterohemorrhagic strains of Escherichia coli (STEC/EHEC) cause hemorrhagic colitis and hemolytic uremic syndrome (HUS). Several large outbreaks resulting from the consumption of fresh produce (e.g., lettuce, spinach, sprouts) and of undercooked ground beef have received significant attention in the media. O157:H7 is the most prominent serotype, but others have been reported to cause similar disease. The ability of STEC/EHEC to produce Shiga toxin (Stx2 and/or Stx1) or related toxins is a critical factor in the expression of clinical disease. Manure from domesticated ruminant animals in industrialized countries serves as the major reservoir for STEC/EHEC. Ground beef—the most common food source of STEC/EHEC strains—is often contaminated during processing. Low bacterial numbers can transmit disease in humans, accounting for widespread infection from environmental sources and person-to-person spread. O157:H7 strains are the fourth most commonly reported cause of bacterial diarrhea in the United States (after CampylobacterSalmonella, and Shigellaspp.). STEC/EHEC characteristically causes grossly bloody diarrhea in more than 90% of cases. Significant abdominal pain and fecal leukocytes are common (70% of cases), but fever is not; absence of fever can incorrectly lead to consideration of noninfectious conditions (e.g., intussusception and inflammatory or ischemic bowel disease). STEC/EHEC disease is usually self-limited, lasting 5 to 10 days. HUS may develop in very young or elderly patients within 2 weeks of diarrhea. It is estimated that it occurs in 2% to 8% of cases of STEC/EHEC and that more than 50% of all cases of HUS in the United States and 90% of cases in children are caused by STEC/EHEC. Antibiotic therapy of STEC/EHEC cases of diarrhea should be avoided because antibiotics may increase the likelihood of developing HUS.

IV-88.  The answer is E. (Chap. 150) Infections with Acinetobacter spp. are a growing cause of hospital-acquired infections worldwide. Surveillance data from Australia and Asia suggest that infections are common, and there are reports of community-acquired Acinetobacter infection. They typically infect patients receiving long-term care in intensive care units by causing ventilator-associated pneumonia, bloodstream infections, or urinary tract infections. They are particularly of concern because of their propensity to develop multidrug (or pan-drug) resistance and their ability to colonize units because of health care worker transmission. A. baumannii is the most common isolate and develops drug resistance avidly. Many strains are currently resistant to carbapenems (imipenem, meropenem). Last-line agents such as colistin, polymixin A, and tigecycline are often the only available therapeutic options. Tigecycline has been used for pneumonia caused by carbapenem-resistant strains but is not thought to be efficacious in bloodstream infection because usual dosing does not achieve therapeutic levels against Acinetobacter spp.

IV-89.  The answer is B. (Chap. 151) Helicobacter pylori is thought to colonize about 50% (30% in developed countries and >80% in developing countries) of the world’s population. The organism induces a direct tissue response in the stomach, with evidence of mononuclear and polymorphonuclear infiltrates in all of those with colonization regardless of whether or not symptoms are present. Gastric ulceration and adenocarcinoma of the stomach arise in association with this gastritis. MALT is specific to H. pylori infection and because of prolonged B-cell activation in the stomach. Although H. pylori does not directly infect the intestine, it does diminish somatostatin production, indirectly contributing to the development of duodenal ulcers. Gastroesophageal reflux disease is not caused by H. pylori colonization. Recent studies have demonstrated that colonization by some strains of H. pylori may be protective for the development of adeno-carcinoma of the esophagus and premalignant lesions such as Barrett’s esophagus (odds ratio, 0.2–0.6).

IV-90.  The answer is E. (Chap. 151) It is impossible to know whether the patient’s continued dyspepsia is attributable to persistent Helicobacter pylori as a result of treatment failure or to some other cause. A quick noninvasive test to look for the presence of H. pylori is a urea breath test. This test can be done as an outpatient and gives a rapid, accurate response. Patients should not have received any proton pump inhibitors or antimicrobials in the meantime. Stool antigen test is another good option if urea breath testing is not available. If the urea breath test is positive more than 1 month after completion of first-line therapy, second-line therapy with a proton pump inhibitor, bismuth subsalicylate, tetracycline, and metronidazole may be indicated. If the urea breath test result is negative, the remaining symptoms are unlikely attributable to persistent H. pylori infection. Serology is useful only for diagnosing infection initially, but it can remain positive and therefore misleading in those who have cleared H. pylori. Endoscopy is a consideration to rule out ulcer or upper gastrointestinal malignancy but is generally preferred after two failed attempts to eradicate H. pyloriFigure IV-90 outlines the algorithm for management of H. pylori infection.

image

FIGURE IV-90

IV-91.  The answer is A. (Chap. 151) Helicobacter pylori is a disease of overcrowding. Transmission has therefore decreased in the United States as the standard of living has increased. It is predicated that the percentage of duodenal ulcers caused by factors other than H. pylori (e.g., use of nonsteroidal anti-inflammatory drugs) will increase over the upcoming decades. Controversial but increasing evidence suggests that H. pylori colonization may provide some protection from recent emerging gastrointestinal disorders, such as gastroesophageal reflux disease (and its complication, esophageal carcinoma). Therefore, the health implications of H. pylori eradication may not be simple.

IV-92.  The answer is A. (Chap. 151) In vitro, Helicobacter pylori is susceptible to a wide variety of antibiotics. However, monotherapy is no longer recommended because of inadequate antibiotic delivery to the colonization niche and the development of resistance. All current regimens include a proton pump inhibitor (omeprazole or equivalent), H2 blocker (ranitidine or equivalent), and/or bismuth. Regimens including quinolones may not be advisable because of common resistance and the risk of developing Clostridium difficile colitis. Current regimens have an eradication rate of 75% to 80%. (See Table IV-92.)

TABLE IV-92 Recommended Treatment Regimens for Helicobacter pylori Infection

image

image

IV-93.  The answer is A. (Chap. 152) Burkholderia cepacia is an opportunistic pathogen that has been responsible for nosocomial outbreaks. It also colonizes and infects the lower respiratory tract of patients with cystic fibrosis, chronic granulomatous disease, and sickle cell disease. In patients with cystic fibrosis, it portends a rapid decline in pulmonary function and a poor clinical prognosis. It also may cause a resistant necrotizing pneumonia. B. cepacia is often intrinsically resistant to a variety of antimicrobials, including many β-lactams and aminoglycosides. Trimethoprim–sulfamethoxazole (TMP/SMX) is usually the first-line treatment. Pseudomonas aeruginosa and Staphylococcus aureus are common colonizers and pathogens in patients with cystic fibrosis. Stenotrophomonas maltophilia is an opportunistic pathogen, particularly in patients with cancer, transplants, and critical illness. S. maltophilia is a cause of pneumonia, urinary tract infection, wound infection, and bacteremia. TMP/SMX is usually the treatment of choice for Stenotrophomonas infections.

IV-94.  The answer is A. (Chap. 152) Ecthyma gangrenosum is a disseminated collection of geographic, painful, reddish, maculopapular lesions that rapidly progress from pink to purple and finally to a black, dry necrosis. They are teeming with causative bacteria. In reviews on ecthyma, Pseudomonas aeruginosa is the most common isolate from blood and skin lesions. However, many organisms can cause this foreboding rash. Neutropenic patients and AIDS patients are at highest risk, but diabetics and intensive care unit (ICU) patients are also affected. Pseudomonal sepsis is severe with a high mortality rate. Its presentation is otherwise difficult to discern from other severe sepsis syndromes, with hypothermia, fever, hypotension, organ damage, encephalopathy, bandemia, and shock being common findings. Although antibiotic use, severe burns, and long ICU stays increase the risk for Pseudomonas infection, these exposures are also risk factors for other bacterial infections, many of which also carry daunting resistant profiles. Because of P. aeruginosa’s propensity for multidrug resistance, two agents (usually an antipseudomonal β-lactam plus an aminoglycoside or ciprofloxacin) are warranted until culture data return confirming sensitivity to one or both agents. At this point, the choice to narrow to one antibiotic or not is still debated and is largely physician preference.

IV-95.  The answer is A. (Chap. 152) Antibiotic therapy against Pseudomonas aeruginosa is often controversial, and the practitioner must interpret recommendations in light of local resistance patterns. Traditionally, many recommended two-drug therapy for Pseudomonas bacteremia because of synergy between aminoglycosides and β-lactam agents. Since the introduction of newer antipseudomonal antibiotics, most studies conclude that monotherapy with one of those drugs to which the isolate is sensitive is as effective as combination therapy. That conclusion holds for bacteremic patients with or without neutropenia. However, monotherapy with an aminoglycoside for P. aeruginosa bacteremia is not recommended. Many recommend adding an aminoglycoside to an antipseudomonal drug in cases of shock or where the prevalence of resistance to the primary drug is high. The Infectious Diseases Society of America recommends adding an aminoglycoside or ciprofloxacin for Pseudomonas pneumonia.

IV-96.  The answer is E. (Chap. 153) Salmonella enteritidis is one of the causes of nontyphoidal salmonellosis (NTS) along with Salmonella typhimurium and other strains. Enteric (typhoid) fever is caused by Salmonella typhi or Salmonella paratyphi. Recent cases of gastroenteritis caused by NTS have been associated with undercooked or raw eggs. In contrast to S. typhi and S. paratyphi, which only have human reservoirs, the NTS can colonize livestock accounting for outbreaks related to contaminated water (fresh produce, undercooked ground meat, dairy products). The gastroenteritis caused by NTS is indistinguishable clinically for other enteric pathogens. The diarrhea is nonbloody and may be copious. The disease is typically self-limited in healthy hosts, and antibiotic therapy is not recommended because it does not change the course of disease and promotes resistance. Therapy may be necessary for neonates or debilitated elderly patients who are more likely to develop bacteremia. Bacteremia occurs in fewer than 10% of cases. Metastatic infections of bone, joint, and endovascular devices may occur. There is no vaccine for NTS. Oral and parenteral vaccines for S. typhi are available.

IV-97.  The answer is D. (Chap. 154) Shigellosis remains a cause of dysentery in the developing world and sporadic cases caused by fecal–oral contamination occur in the developing and developed world. The human intestinal tract is the most prevalent reservoir for the bacteria. Clinical illness from Shigella infection can be caused by a very small inoculum. Shigellosis typically evolves through four phases: incubation, watery diarrhea, dysentery, and the postinfectious phase. The incubation period is usually 1 to 4 days, and the dysentery follows within hours to days. The dysentery syndrome is indistinguishable from other invasive enteropathogens (including Campylobacter spp.), and inflammatory bowel disease is also in the differential diagnosis. Because the organism is enteroinvasive, antibiotic therapy is indicated. Ciprofloxacin is generally recommended unless there is no or proven resistance. Ceftriaxone, azithromycin, pivmecillinam, and some recent quinolones are also effective. Shigella infection typically does not cause life-threatening dehydration. Antimotility agents are not recommended because they are thought to prolong the systemic symptoms and may increase the risk of toxic megacolon and hemolytic uremic syndrome. There is currently no commercially available vaccine for Shigella infection.

IV-98.  The answer is A. (Chap. 155) Campylobacter spp. are motile, curved gram-negative rods. The principal diarrheal pathogen is Campylobacter jejuni. This organism is found in the gastrointestinal tract of many animals used for food production and is usually transmitted to humans in raw or undercooked food products or through direct contact with infected animals. More than half of cases are caused by insufficiently cooked contaminated poultry. Campylobacter infection a common cause of diarrheal disease in the United States. The illness usually occurs within 2 to 4 days after exposure to the organism in food or water. Biopsy of an affected patient’s jejunum, ileum, or colon reveals findings indistinguishable from those of Crohn’s disease and ulcerative colitis. Although the diarrheal illness is usually self-limited, it may be associated with constitutional symptoms, lasts more than 1 week, and recurs in 5% to 10% of untreated patients. Complications include pancreatitis, cystitis, arthritis, meningitis, and Guillain-Barré syndrome. The symptoms of Campylobacter enteritis are similar to those resulting from infection with Salmonella typhi, Shigella spp., and Yersinia spp.; all of these agents cause fever and the presence of fecal leukocytes. The diagnosis is made by isolating Campylobacter organisms from the stool, which requires selective media. Escherichia coli(enterotoxigenic), Norwalk agent, and rotavirus are generally not associated with the finding of fecal leukocytes. About 5% to 10% of untreated patients with Campylobacter enteritis develop recurrences that may be clinically and pathologically confused with inflammatory bowel disease.

IV-99.  The answer is A. (Chap. 155) As is true with all acute diarrheal diseases, adequate volume resuscitation is central to treatment. Many patients with mild Campylobacter enteritis will resolve spontaneously, and not all patients clearly benefit from therapy. In the presence of high or persistent fever, bloody diarrhea, severe diarrhea, worsening symptoms, or symptoms persisting for more than 1 week, antibiotics are recommended. A 5- to 7-day course of erythromycin, azithromycin (and other macrolides), or ciprofloxacin is effective. Drug resistance to flouroquinolones and tetracycline is increasing. Antimotility agents are not recommended because they have been associated with the development of serious complications, including toxic megacolon and hemolytic uremic syndrome. Tinidazole and metronidazole are used to treat a variety of nonbacterial diarrhea syndromes, including giardiasis and amoebiasis. Metronidazole is also used for Clostridium difficile–associated colitis.

IV-100. The answer is B. (Chap. 156) Cholera remains a worldwide problem with sporadic cases usually related to contact with fecally contaminated water or seafood. Humans are the only known reservoir of Vibrio cholera. Most cases are reported in Africa or Asia. After a century, cholera returned to Haiti after recent natural disasters and breakdown of public health measures. The watery diarrhea of cholera is mediated by a specific cholera toxin that binds to small intestine epithelium to cause profuse fluid secretion. The diarrhea of cholera is painless, nonbloody, and watery with mucus and few inflammatory cells. The term “rice-water” diarrhea refers to the appearance of water after soaking rice. Morbidity and mortality from cholera are from profound volume depletion. Rehydration is essential to therapy. Major improvements in care came from the development of oral rehydration solutions that take advantage of glucose–sodium co-transport in the small intestine. These solutions allowed effective rehydration in resource limited settings where intravenous rehydration was not practical. Diagnosis is by culture or point-of-care antigen detection dipstick assay. Antibiotics are not necessary for cure, but they diminish the duration and volume of fluid loss and hasten the clearance of the organism from stool. A single dose of doxycycline is effective in adults in areas where there is not resistance. Ciprofloxacin or azithromycin may be alternatives.

IV-101. The answer is E. (Chap. 158) The most likely infecting organism in this patient is Francisella tularensis. Gentamicin is the antibiotic of choice for the treatment of tularemia. Fluoroquinolones have shown in vitro activity against F. tularensis and have successfully been used in a few cases of tularemia. Currently, however, it cannot be recommended as first-line therapy because data are limited regarding its efficacy relative to gentamicin, but it can be considered if an individual is unable to tolerate gentamicin. To date, there have been no clinical trials of fluoroquinolones to definitively demonstrate equivalency with gentamicin. Third-generation cephalosporins have in vitro activity against F. tularensis. However, use of ceftriaxone in children with tularemia resulted in almost universal failure. Likewise, tetracycline and chloramphenicol also have limited usefulness with a higher relapse rate (up to 20%) compared with gentamicin. F. tularensis is a small gram-negative, pleomorphic bacillus that is found both intra- and extracellularly. It is found in mud, water, and decaying animal carcasses, and ticks and wild rabbits are the sources for most human infections in the southeast United States and Rocky Mountains. In western states, tabanid flies are the most common vectors. The organisms usually enter the skin through the bite of a tick or through an abrasion. On further questioning, the patient above reported that during the camping trip, he was primarily responsible for skinning the animals and preparing dinner. He did sustain a small cut on his right hand at the site where the ulceration is apparent. The most common clinical manifestations of F. tularensis are ulceroglandular and glandular disease, accounting for 75% to 85% of cases. The ulcer appears at the site of entry of the bacteria and lasts for 1 to 3 weeks and may develop a black eschar at the base. The draining lymph nodes become enlarged and fluctuant. They may drain spontaneously. In a small percentage of patients, the disease becomes systemically spread, as is apparent in this case, with pneumonia, fevers, and sepsis syndrome. When this occurs, the mortality rate approaches 30% if untreated. However, with appropriate antibiotic therapy, the prognosis is very good. Diagnosis requires a high clinical suspicion because demonstration of the organism is difficult. It rarely seen on Gram stain because the organisms stain weakly and are so small that they are difficult to distinguish from background material. On polychromatically stained tissue, they may be seen both intra- and extracellularly, singly or in clumps. Moreover, F. tularensis is a difficult organism to culture and requires cysteine-glucose–blood agar. However, most laboratories do not attempt to culture the organism because of the risk of infection in laboratory workers, requiring biosafety level 2 practices. Usually the diagnosis is confirmed by agglutination testing with titers above 1:160 confirming diagnosis.

IV-102 and IV-103. The answers are E and B, respectively. (Chap. 159) This patient has a classic presentation of bubonic plague caused by Yersinia pestis. Plague is transmitted to humans from rodents via flea bites. The clinical manifestations include bubonic (most common, 80%–95% of cases), septicemic (bacteremia without a bubo), primary pneumonic, and secondary pneumonic. The untreated mortality is up to 20% with higher rates for septicemic and pneumonic presentations. Most cases in the United States occur in the Four Corners region or in the border zone of northern California, southern Oregon, and western Nevada. The presenting bubo in bubonic plague is usually near the inciting flea bite. The differential diagnosis includes streptococcal or staphylococcal infection, tularemia, cat-scratch disease, tick typhus, infectious mononucleosis, and lymphatic filariasis. These infections do not progress as rapidly as plague, are not as painful, and are associated with visible cellulitis or ascending lymphangitis, which are absent in plague. The organisms may be visualized on bubo aspirate. The gram-negative coccobacillus Y. pestis is characteristically bipolar on Wright’s stain. Bartonella and rickettsia are generally not visible on Gram staining. Traditionally, streptomycin was the first-line treatment, but because of fewer side effects, gentamicin is currently recommended. Fluoroquinolones have in vitro activity and have been reported effective in case reports. They would likely be administered in the event of pneumonic plague as a bioterrorism event.

IV-104. The answer is A. (Chap. 160) This patient has bacillary angiomatosis caused by cutaneous infection with Bartonella quintana or Bartonella henselae. Kittens are the likely source of the infection in this case. Bacillary angiomatosis occurs in HIV-infected patients with CD4+ T-cell counts below 100/μL. The cutaneous lesions of bacillary angiomatosis are typically painless cutaneous lesions but may appear as subcutaneous nodules, ulcerated plaques, or verrucous growths. They may be single or multiple. The differential diagnosis includes Kaposi’s sarcoma, pyogenic granuloma, and tumors. Biopsy findings are as described in this case, and the diagnosis is best made with histology. Treatment is with azithromycin or doxycycline. Oxacillin or vancomycin is the treatment for staphylococcal or streptococcal skin infections. Treatment with antiretrovirals to restore CD4+ T-cell count will prevent further episodes.

IV-105. The answer is A. (Chap. 160) This patient has culture-negative endocarditis, a rare entity defined as clinical evidence of infectious endocarditis in the absence of positive blood cultures. In this case, evidence for subacute bacterial endocarditis includes valvular regurgitation; an aortic valve vegetation; and embolic phenomena on the extremities, spleen, and kidneys. A common reason for negative blood culture results is prior antibiotics. In the absence of this, the two most common pathogens (both of which are technically difficult to isolate in blood culture bottles) are Q fever, or Coxiella burnetii (typically associated with close contact with livestock), and Bartonella spp. In this case, the patient’s homelessness and body louse infestation are clues for Bartonella quintana infection. Diagnosis is made by blood culture about 25% of the time. Otherwise, direct polymerase chain reaction of valvular tissue, if available, or acute and convalescent serologies are diagnostic options. Empirical therapy for culture-negative endocarditis usually includes ceftriaxone and gentamicin with or without doxycycline. For confirmed Bartonellaendocarditis, optimal therapy is gentamicin plus doxycycline. Epstein-Barr virus and HIV do not cause endocarditis. A peripheral blood smear would not be diagnostic.

IV-106. The answer is C. (Chap. 160) Although the patient’s gardening puts her at risk for Sporothrix infection, this infection typically causes a more localized streaking nodular lymphadenitis affecting the forearm. The differential diagnosis for nodular adenitis includes Sporothrix schenckii, Nocardia brasiliensis, Mycobacterium marinumLeishmania braziliensis, and Francisella tularensis and is based on direct inoculation of organism caused by contact with soil, a marine environment, or an insect or animal bite. This patient has regional lymphadenitis involving larger lymph nodes that drain the site of inoculation. Most likely in her case is cat-scratch disease caused by infection with Bartonella henselae, based on the kittens in her home, but lymphoma and staphylococcal infection must also be considered, and often a lymph node biopsy is required to make this distinction. Most cases of cat-scratch disease resolve without therapy. In immuno-competent patients, antibiotic therapy has minimal benefit but may expedite resolution of lymphadenopathy. Antimicrobial therapy, usually with azithromycin, is indicated in immunosuppressed patients.

IV-107. The answer is A. (Chap. 161) Donovanosis is caused by the intracellular organism Calymmatobacterium granulomatis and most often presents as a painless erythematous genital ulceration after a 1- to 4-week incubation period. However, incubation periods can be as long as 1 year. The infection is predominantly sexually transmitted, and autoinoculation can lead to formation of new lesions by contact with adjacent infected skin. Typically, the lesion is painless but bleeds easily. Complications include phimosis in men and pseudo-elephantiasis of the labia in women. If the infection is untreated, it can lead to progressive destruction of the penis or other organs. Diagnosis is made by demonstration of Donovan bodies within large mononuclear cells on smears from the lesion. Donovan bodies refers to the appearance of multiple intracellular organisms within the cytoplasm of mononuclear cells. These organisms are bipolar and have an appearance similar to a safety pin. On histologic examination, there is an increase in the number of plasma cells with few neutrophils; additionally, epithelial hyperplasia is present and can resemble neoplasia. A variety of antibiotics can be used to treat donovanosis, including macrolides, tetracyclines, trimethoprim–sulfamethoxazole, and chloramphenicol. Treatment should be continued until the lesion has healed, often requiring 5 or more weeks of treatment. All of the choices listed in the question are in the differential diagnosis of penile ulcerations. Lymphogranuloma venereum is endemic in the Caribbean. The ulcer of primary infection heals spontaneously, and the second phase of the infection results in markedly enlarged inguinal lymphadenopathy, which may drain spontaneously. Haemophilus ducreyi results in painful genital ulcerations, and the organism can be cultured from the lesion. The painless ulcerations of cutaneous leishmaniasis can appear similarly to those of donovanosis but usually occur on exposed skin. Histologic determination of intracellular parasites can distinguish leishmaniasis definitively from donovanosis. Finally, it is unlikely that the patient has syphilis in the setting of a negative rapid plasma reagin test result, and the histology is inconsistent with this diagnosis.

IV-108. The answer is D. (Chaps. 16 and 155) This patient is chronically immunosuppressed from his antirejection prophylactic regimen, which includes both glucocorticoids and azathioprine. However, the finding of a cavitary lesion on chest radiography considerably narrows the possibilities and increases the likelihood of nocardial infection. The other clinical findings, including production of profuse thick sputum, fever, and constitutional symptoms, are also quite common in patients who have pulmonary nocardiosis. The Gram stain, which demonstrates filamentous branching gram-positive organisms, is characteristic. Most species of Nocardia are acid fast if a weak acid is used for decolorization (e.g., modified Kinyoun method). These organisms can also be visualized by silver staining. They grow slowly in culture, and the laboratory must be alerted to the possibility of their presence on submitted specimens. After the diagnosis, which may require an invasive approach, is made, sulfonamides are the drugs of choice. Sulfadiazine or sulfisoxazole from 6 to 8 g/day in four divided doses generally is administered, but doses up to 12 g/day have been given. The combination of sulfamethoxazole and trimethoprim has also been used, as have the oral alternatives minocycline and ampicillin and intravenous amikacin. There is little experience with the newer β-lactam antibiotics, including the third-generation cephalosporins and imipenem. Erythromycin alone is not effective, although it has been given successfully along with ampicillin. In addition to appropriate antibiotic therapy, the possibility of disseminated nocardiosis must be considered; common sites include brain, skin, kidneys, bone, and muscle.

IV-109. The answer is D. (Chap. 163) The patient presents with symptoms suggestive of osteonecrosis of her jaw possibly caused by bisphosphonate use. Additionally, her jaw pain has progressed and now appears to be infected. Actinomyces is a classic oral organism with a propensity to infect the jaw, particularly when the bone is abnormal, usually because of radiation or osteonecrosis. Osteonecrosis of the jaw caused by bisphosphonates is an increasingly recognized risk factor for actinomyces infection. Frequently, the soft tissue swelling is confused for either parotitis or a cancerous lesion. Actinomyces spp. frequently form fistulous tracts, which provide an opportunity to examine the secretions and identify either the organism itself (less common) or sulfur granules. Sulfur granules are an in vivo concretion of Actinomyces bacteria, calcium phosphate, and host material. Gram stain of Actinomyces infection shows intensely positive staining at the center with branching rods at the periphery. Auer rods are found in acute promyelocytic leukemia. Although head and neck cancer is in the differential diagnosis, the acuity of the presentation and fever make this less likely. Weakly acid-fast branching filaments are found in nocardial infection, which is unlikely to involve the head and neck, although both organisms frequently cause pulmonary infiltrates. Although parotitis with obstruction caused by sialolith is possible, the symptoms are in the jaw and diffuse, not specifically involving the parotid gland, thus making sialolith less likely.

IV-110. The answer is C. (Chap. 163) Therapy for actinomyces requires a long course of antibiotics even though the organism is very sensitive to penicillin therapy. This is presumed to be attributable to the difficulty of using antibiotics to penetrate the thick-walled masses and sulfur granules. Current recommendations are for penicillin IV for 2 to 6 weeks followed by oral therapy for a total of 6 to 12 months. Surgery should be reserved for patients who are not responsive to medical therapy.

IV-111. The answer is E. (Chap. 164) The patient presents with symptoms suggestive of lung infection, his demographics suggest that an abscess might be present, and the foul-smelling breath supports this diagnosis. His chest radiograph shows a cavity with air-fluid level, thus confirming the diagnosis of lung abscess. Lung abscess generally presents in the dependent lobes, as it did in this patient, and often involves oral anaerobic bacteria that are normally found in the crevices of teeth. Although infections are frequently polymicrobial, clindamycin is usually adequate to treat the infection. In this case, if there is no response to therapy targeted at lung abscess for several weeks, then further testing would be warranted, generally including bronchoscopy to evaluate possible malignancy. The negative purified protein derivative result makes tuberculosis very unlikely, and chest radiography did not show upper lobe infiltrates or cavities.

IV-112. The answer is B. (Chap. 164) The major reservoirs in the human body for anaerobic bacteria are the mouth, lower gastrointestinal tract, skin, and female genital tract. Generally, anaerobic infections occur proximal to these sites after the normal barrier (i.e., skin or mucous membrane) is disrupted. Thus, common infections resulting from these organisms are abdominal or lung abscess, periodontal infection, gynecologic infections such as bacterial vaginosis, and deep tissue infection. Properly obtained cultures in these circumstances generally grow a mixed population of anaerobes typical of the microenvironment of the original reservoir.

IV-113. The answer is C. (Chap. 165) Tuberculosis is most commonly transmitted from person to person by airborne droplets. Factors that affect the likelihood of developing tuberculosis infection include the probability of contact with an infectious person, the intimacy and duration of contact, the degree of infectiousness of the contact, and the environment in which the contact takes place. The most infectious patients are those with cavitary pulmonary or laryngeal tuberculosis with about 105 to 107 tuberculous bacteria per milliliter of sputum. Individuals who have a negative acid-fast bacillus smear with a positive culture for tuberculosis are less infectious but may transmit the disease. However, individuals with only extrapulmonary (e.g., renal, skeletal) tuberculosis are considered noninfectious.

IV-114. The answer is D. (Chap. 165) Aging, chronic disease, and suppression of cellular immunity are risk factors for developing active tuberculosis in patients with latent infection. (See Table IV-114.) The greatest absolute risk factor for development of active tuberculosis is HIV positivity. The risk of developing active infection is greatest in those with the lowest CD4 counts; however, having a CD4 count above a threshold value does not negate the risk of developing an active infection. The reported incidence of developing active tuberculosis in HIV-positive individuals with a positive purified protein derivative result is 10% per year compared with a lifetime risk of 10% in immunocompetent individuals. Whereas malnutrition and severe underweight confer a twofold greater risk of developing active tuberculosis, IV drug use increases the risk 10 to 30 times. Silicosis also increases the risk of developing active tuberculosis 30 times. Although the risk of developing active tuberculosis is greatest in the first year after exposure, the risk also increases in elderly adults. Coal mining has not been associated with increased risk independent of other factors, such as tobacco smoking.

TABLE IV-114 Risk Factors for Active Tuberculosis Among Persons Who Have Been Infected With Tubercle Bacilli

image

IV-115. The answer is A. (Chap. 165) The chest radiograph shows a right upper lobe infiltrate with a large cavitary lesion. In this man from an endemic area for tuberculosis, this finding should be treated as active pulmonary tuberculosis until proven otherwise. In addition, this patient’s symptoms suggest a chronic illness with low-grade fevers, weight loss, and temporal wasting that would be consistent with active pulmonary tuberculosis. If a patient is suspected of having active pulmonary tuberculosis, the initial management should include documentation of disease while protecting health care workers and the population in general. This patient should be hospitalized in a negative-pressure room on airborne isolation until three expectorated sputum samples have been demonstrated to be negative. The samples should preferably be collected in the early morning because the burden of organisms is expected to be higher on a more concentrated sputum. The sensitivity of a single sputum for the detection of tuberculosis in confirmed cases is only 40% to 60%. Thus, a single sputum sample is inadequate to determine infectivity and the presence of active pulmonary tuberculosis. Skin testing with a purified protein derivative of the tuberculosis mycobacterium is used to detect latent infection with tuberculosis and has no role in determining whether active disease is present. The cavitary lung lesion shown on the chest imaging could represent malignancy or a bacterial lung abscess, but because the patient is from a high-risk area for tuberculosis, tuberculosis would be considered the most likely diagnosis until ruled out by sputum testing.

IV-116. The answer is A. (Chap. 165) Initial treatment of active tuberculosis associated with HIV disease does not differ from that of a non-HIV-infected person. The standard treatment regimen includes four drugs: isoniazid, rifampin, pyrazinamide, and ethambutol (RIPE). These drugs are given for a total of 2 months in combination with pyridoxine (vitamin B6) to prevent neurotoxicity from isoniazid. After the initial 2 months, patients continue on isoniazid and rifampin to complete a total of 6 months of therapy. These recommendations are the same as those of non–HIV-infected individuals. If the sputum culture remains positive for tuberculosis after 2 months, the total course of antimycobacterial therapy is increased from 6 to 9 months. If an individual is already on antiretroviral therapy (ART) at the time of diagnosis of tuberculosis, it may be continued, but often rifabutin is substituted for rifampin because of drug interactions between rifampin and protease inhibitors. In individuals not on ART at the time of diagnosis of tuberculosis, it is not recommended to start ART concurrently because of the risk of immune reconstitution inflammatory syndrome (IRIS) and an increased risk of medication side effects. IRIS occurs as the immune system improves with ART and causes an intense inflammatory reaction directed against the infecting organism(s). There have been fatal cases of IRIS in association with tuberculosis and initiation of ART. In addition, both ART and antituberculosis drugs have many side effects. It can be difficult for a clinician to decide which medication is the cause of the side effects and may lead unnecessarily to alterations in the antituberculosis regimen. ART should be initiated as soon as possible and preferably within 2 months. Three-drug regimens are associated with a higher relapse rate if used as a standard 6-month course of therapy and, if used, require a total of 9 months of therapy. Situations in which three-drug therapy may be used are pregnancy, intolerance to a specific drug, and in the setting of resistance. A five-drug regimen using Rifampin, isoniazide, pyrazinamide, ethambutol (RIPE) plus streptomycin is recommended as the standard retreatment regimen. Streptomycin and pyrazinamide are discontinued after 2 months if susceptibility testing is unavailable. If susceptibility testing is available, the treatment should be based on the susceptibility pattern. In no instance is it appropriate to withhold treatment in the setting of active tuberculosis to await susceptibility testing.

IV-117. The answer is B. (Chap. 165) The aim of treatment of latent tuberculosis is to prevent development of active disease, and the tuberculin skin test (purified protein derivative [PPD]) is the most common means of identifying cases of latent tuberculosis in high-risk groups. To perform a tuberculin skin test, 5 tuberculin units of PPD are placed subcutaneously in the forearm. The degree of induration is determined after 48 to 72 hours. Erythema only does not count as a positive reaction to the PPD. The size of the reaction to the tuberculin skin test determines whether individuals should receive treatment for latent tuberculosis. In general, individuals in low-risk groups should not be tested. However, if tested, a reaction larger than 15 mm is required to be considered as positive. School teachers are considered low-risk individuals. Thus, the reaction of 7 mm is not a positive result, and treatment is not required. A size of 10 mm or larger is considered positive in individuals who have been infected within 2 years or those with high-risk medical conditions. The individual working in an area where tuberculosis is endemic has tested newly positive by skin testing and should be treated as a newly infected individual. High-risk medical conditions for which treatment of latent tuberculosis is recommended include diabetes mellitus, injection drug use, end-stage renal disease, rapid weight loss, and hematologic disorders. PPD reactions 5 mm or larger are considered positive for latent tuberculosis in individuals with fibrotic lesions on chest radiographs, those with close contact with an infected person, and those with HIV or who are otherwise immunosuppressed. There are two situations in which treatment for latent tuberculosis is recommended regardless of the results on skin testing. First, infants and children who have had close contact with an actively infected person should be treated. After 2 months of therapy, a skin test should be performed. Treatment can be discontinued if the skin test result remains negative at that time. Also, individuals who are HIV positive and have had close contact with an infected person should be treated regardless of their skin test results.

IV-118. The answer is C. (Chap. 165) T-lymphocyte release of interferon-gamma in response to highly specific tuberculosis antigen stimulation is the basis for the commercially available interferon-gamma release assays (IGRAs). IGRAs are more specific than tuberculin skin testing caused by less cross-reactivity with non-mTB organisms, including Bacillus Calmette-Guérin and nontuberculous mycobacteria. The absolute sensitivity of IGRAs is not clearly known because of the difficulty in establishing a gold standard, but most studies demonstrate superior performance in detecting latent tuberculosis in low-incidence settings. They also are more user friendly because there is no administration expertise, interpretation is less subjective, and results do not require a second visit. The results are far less clear in settings of high tuberculosis or HIV burden. The tuberculin skin testing booster phenomenon, a spurious conversion caused by serial testing, does not occur with IGRAs; however, a tuberculin skin test may cause a false-positive IGRA result. In the United States, IGRA is preferred for most persons older than 5 years of age being screened for latent tuberculosis.

IV-119. The answer is E. (Chap. 165) Bacillus Calmette-Guérin (BCG) is derived from an attenuated strain of Mycobacterium bovis. It has been available since 1921. Many vaccines are available, but they vary in efficacy from 0% to 80% in clinical trials. The vaccine protects infants and young children from serous forms of tuberculosis, including meningitis and miliary disease. Side effects from the vaccine are rare, but BCG dissemination (BCGitis) may occur in patients with severe combined immunodeficiency or advanced HIV-induced immune suppression. BCG cross-reacts with tuberculin skin testing, but the size of the response wanes with time. BCG vaccination is currently recommended in countries with a high TB prevalence. It is not recommended in the United States because of the low prevalence of disease and cross-reactivity with tuberculin skin testing. Infants with unknown HIV infection status, infants of mothers with known HIV infection, and HIV-infected individuals should not receive BCG vaccination.

IV-120. The answer is B. (Chap. 167) The chest computed tomography shows a “tree-in-bud” pattern in the peripheral right lung and bilateral bronchiectasis. This pattern is consistent with bronchiolar inflammation and is typical of nontuberculous mycobacterial infection. Nontuberculous mycobacteria, such as Mycobacterium avium complex (MAC), may cause chronic pulmonary infections in normal hosts and those with underlying pulmonary disease immunosuppression. In normal hosts, bronchiectasis is the most common underlying condition. In immunocompetent patients without underlying disease, treatment of pulmonary infection with MAC is considered on an individual basis based on symptoms, radiographic findings, and bacteriology. Treatment should be initiated in the presence of progressive pulmonary disease or symptoms. In patients without any prior lung disease, no structural lung disease, and who do not demonstrate progressive clinical decline, M. avium pulmonary infection can be managed conservatively. Patients with underlying lung disease, such as chronic obstructive pulmonary disease, bronchiectasis, or cystic fibrosis, or those with a history of pulmonary tuberculosis should receive antibiotics. This patient has both clinical and historic reasons for antibiotic treatment. The appropriate regimen in this case is clarithromycin (or azithromycin), ethambutol, and rifampin (or rifabutin) for 12 months after culture sterilization (typically 18 months). The combination of pyrazinamide, isoniazid, rifampin, and ethambutol is effective treatment for M. tuberculosis infection, which is not present here. Other drugs with activity against MAC include intravenous and aerosolized aminoglycosides, fluoroquinolones, and clofazimine.

IV-121. The answer is C. (Chap. 168) Pyrazinamide (PZA) is the first-line treatment for Mycobacterium tuberculosis. Addition of PZA for 2 months to isoniazid and rifampin allows the total duration of treatment to be shortened from 9 months to 6 months. PZA has no utility in the treatment of nontuberculous mycobacteria. Ethambutol has no serious drug interactions, but patients must be closely monitored for optic neuritis, which may manifest with decreased visual acuity, central scotoma, or difficulty seeing green (or red). All patients initiating therapy with ethambutol should have a visual and ophthalmologic examination at baseline. In the United States overall, isoniazid resistance remains uncommon. Primary isoniazid resistance is more common in patients with tuberculosis born outside the United States. Rifampin is a potent inducer of cytochrome P450 system and has numerous drug interactions. The Centers for Disease Control and Prevention has guidelines for managing antituberculosis drug interactions including rifampin. Rifabutin is a less potent inducer of hepatic cytochromes. Rifabutin is recommended for HIV-infected patients who are on antiretroviral therapy with protease inhibitors or non-nucleoside reverse transcriptase inhibitors (particularly nevirapine) in place of rifampin.

IV-122. The answer is E. (Chap. 169) Neurosyphilis has generally been thought to be a late complication of syphilis infection, but this is now known to be inaccurate. Within weeks after infection, the central nervous system is invaded with treponemal organisms. The vast majority of cases are asymptomatic. Abnormal protein levels within the cerebrospinal fluid (CSF) or positive CSF Venereal Disease Research Laboratory (VDRL) test can be seen in up to 40% of individuals with primary or secondary syphilis and 25% of cases of latent syphilis. In symptomatic cases, neurosyphilis can have a variety of manifestations that are typically considered in three broad categories: meningeal, meningovascular, and parenchymal syphilis. Meningeal syphilis is dominated by headache, neck stiffness, and cranial nerve abnormalities. Meningovascular syphilis has signs of meningitis but also can include a vasculitis complicated by stroke. Parenchymal syphilis does indeed represent a late manifestation of disease. Changes in personality, dementia, Argyll Robertson pupils, and paresis are typical findings.

It can be difficult to determine which patients with syphilis required a lumbar puncture to assess for central nervous system involvement of the disease. However, it is quite important because the treatment of neurosyphilis requires 14 days of treatment with intravenous penicillin. Clearly, any patient with a positive test result for syphilis with concerning neurologic symptoms should undergo a lumbar puncture. Some experts also recommend lumbar puncture in all patients with syphilis who are HIV positive. However, this is controversial with others recommending lumbar puncture only if the CD4 count is less than 350/μL. Other instances in which a lumbar puncture is recommended is in the setting of a very high titer rapid plasma reagin (RPR) or VDRL (>1:32) or failure of the RPR or VDRL to fall by a factor of 4 after appropriate treatment. Thus, all of the patients presented would be recommended to undergo a lumbar puncture.

IV-123. The answer is D. (Chap. 169) The patient’s clinical examination is consistent with primary syphilis and he should receive appropriate therapy. In primary syphilis, 25% of patients will have negative nontreponemal tests for syphilis (rapid plasma reagin or Venereal Disease Research Laboratory). A single dose of long-acting benzathine penicillin is the recommended treatment for primary, secondary, and early latent syphilis. Ceftriaxone is the treatment of choice for gonorrhea, but this lesion is not consistent with that diagnosis. Ceftriaxone given daily for 7 to 10 days is an alternative treatment for primary and secondary syphilis. Acyclovir is the drug of choice for genital herpes. Herpetic lesions are classically multiple and painful. Observation is not an option because the chancre will resolve spontaneously without treatment and the patient will remain infected and infectious. Given the appearance and clinical history, there is no indication for tissue biopsy or surgical resection.

IV-124. The answer is E. (Chap. 171) The patient has Weil’s syndrome caused by infection with Leptospira interrogans. L. interrogans is a spirochete that is acquired through contact with an infected animal. Species that commonly transmit the organism to human include rats, dogs, cattle, and pigs. The organism is excreted in urine and can survive in water for months. For a human to become infected, susceptible individuals typically have indirect contact with infected animal urine through contaminated water sources and other wet environments. Tropical human environments, rodent infestations, and large populations of infected dogs are also important for transmission. Leptospirosis occurs only sporadically in the United States with most cases occurring in Hawaii.

Clinically, leptospirosis can take many manifestations, including subclinical infection, a self-limited febrile illness, and Weil’s disease. Leptospirosis is classically a biphasic disease. After the acute exposure, fevers last for 3 to 10 days. During this time, the patient will complain of malaise and myalgias. Conjunctival suffusion (dilated conjunctival blood vessels without drainage) is common as are pharyngeal edema, muscle tenderness, and crackles on lung examination. Weil’s disease is the most serious form of leptospirosis and occurs during the immune phase of the disease. Clinically, severe jaundice in the absence of hepatocellular injury is a striking feature of the disease. In addition, acute kidney injury, hypotension, and diffuse hemorrhage are common. The lungs are the most common site of hemorrhage, but the gastrointestinal tract, retroperitoneum, pericardium, and brain can also be affected. The diagnosis is most often made by serologic assays because culture of the organism takes several weeks. Treatment of leptospirosis is typically intravenous penicillin, ceftriaxone, or cefotaxime.

Acute alcoholic hepatitis can produce fevers and malaise, but a more marked increase in liver enzymes would be expected with the aspartate aminotransferase elevated out of proportion to alanine aminotransferase. Disseminated intravascular coagulation in the setting of an infection would demonstrate abnormalities of coagulation, which were not present here. Microscopic polyangiitis is a small to medium vessel vasculitis that could cause pulmonary hemorrhage and acute renal failure. Rarely, the liver can be affected as well. However, the urinalysis does not suggest acute glomerulonephritis because no casts or red blood cells are present. Rat-bite fever causes intermittent fevers, polyarthritis, and a nonspecific rash.

IV-125. The answer is E. (Chap. 172) Tickborne relapsing fever (TBRF) is a spirochetal infection caused by any one of several species of Borrelia. The Borrelia are small spirochetes that are transmitted to humans through the bite of an infected tick. The tick that transmits TBRF is Ornithodoros spp., which feeds on a variety of squirrels and chipmunks that live near freshwater lakes. TBRF is endemic in several areas of the western United States, southern British Columbia, the Mediterranean, Africa, and the plateau regions of Mexico and South and Central America. In the United States, TBRF is rarely reported east of Montana, Colorado, New Mexico, and Texas. The general areas where TBRF is contracted is in the forested and mountainous regions of these states, although it can be contracted in the limestone caves of central Texas. Only 13 counties in the entire United States have had 50% of all cases reported in the United States.

After an incubation period of about 7 days, an individual infected with TBRF will begin to experience fevers that can reach as high as 106.7°F (41.5°C). Symptoms that accompany the fevers include myalgias, chills, nausea, vomiting, abdominal pain, confusion, and arthralgias. The average duration of a first episode is 3 days. If the disease is not recognized and treated, the fever will recur after a period of about 7 days. The duration of fevers is typically shorter with repeated episodes but will continue to relapse about every 7 days until the disease is treated. Diagnosis of TBRF requires detection of the spirochetes in the blood during a febrile episode or serologic conversion. TBRF is typically treated with doxycycline or erythromycin for 7 to 10 days.

The other options should be on the differential diagnosis for an individual with recurrent and relapsing fevers. In addition, this list would also include yellow fever, dengue fever, malaria, rat-bite fever, and infection with echovirus 9 or Bartonella spp. Brucellosis is a bacterial infection most commonly transmitted by ingestion of contaminated milk or cheese, which this patient did not report. Colorado tick fever is a viral infection transmitted by the bite of a Dermacentor andersoni tick that is endemic in the western areas of the United States. The pattern of fever is slightly different from TBRF because the cycle is 2 to 3 days of fever followed by 2 to 3 days of normal temperature. Leptospirosis often has two phases of fever. The first occurs during the acute infection, lasting 7 to 10 days. In some individuals, the fever recurs 3 to 10 days later during the immune phase. The typical route of infection is prolonged contact with infected rodent droppings in wet environments. Lymphocytic choriomeningitis is a viral infection that is most commonly transmitted via contact with urine or droppings from common house mice. This illness usually has two phases as well. During the first phase that occurs 8 to 13 days after exposure, an individual will experience fevers, malaise, and myalgias. In the second phase of illness, symptoms more typical of meningitis occur.

IV-126. The answer is C. (Chap. 174) About 8% of individuals affected by Lyme disease have cardiac involvement during the second stage of disease. Caused by Borrelia burgdorferi, Lyme disease is transmitted by the bite of an infected Ixodes tick. The first phase of the disease represents localized infection and is characterized by the presence of the erythema migrans rash. The second stage of the disease represents disseminated infection. The most common manifestations of this stage are new annular skin lesions, headache, fever, and migratory arthralgias. When cardiac involvement is present, the most common presentation is related to conduction abnormalities, including all categories of heart block. Diffuse cardiac involvement can occur with acute myopericarditis and left ventricular dysfunction. The cardiac involvement typically resolves within a few weeks even without treatment.

Acute myocardial infarction (MI) can cause complete heart block, particularly in the event of an inferior MI. However, this patient has minimal risk factors for cardiac disease, is otherwise healthy, and has no symptoms to suggest this as a cause. Chagas disease is caused Trypanosoma cruzi, an parasite endemic to Mexico and Central and South America. Sarcoidosis is a systemic disease that pathologically demonstrates the diffuse presence of noncaseating granulomas in a variety of tissues. Conduction abnormalities, including complete heart block and ventricular tachycardia, can be the presenting symptoms of the disease. More commonly, sarcoidosis would have pulmonary manifestations. Although sarcoidosis is certainly possible in this patient, it would be a diagnosis of exclusion because his risk factors make Lyme disease more likely. Subacute bacterial endocarditis can also result in complete heart block if the endocarditis progresses to develop a valve ring abscess. The patient with subacute bacterial endocarditis would present with a more acute illness than this patient with fevers, weight loss, and most likely secondary signs of endocarditis such as Osler nodes, splinter hemorrhages, and Janeway lesions.

IV-127. The answer is A. (Chap. 173) Lyme serology tests should be done only in patients with an intermediate pretest probability of having Lyme disease. The presence of erythema migrans in both patient B and patient E is diagnostic of Lyme disease in the correct epidemiologic context. The diagnosis is entirely clinical. Patient C’s clinical course sounds more consistent with systemic lupus erythematosus, and initial laboratory evaluation should focus on this diagnosis. Patients with chronic fatigue, myalgias, and cognitive change are occasionally concerned about Lyme disease as a potential etiology for their symptoms. However, the pretest probability of Lyme is low in these patients, assuming the absence of antecedent erythema migrans, and a positive serology is unlikely to be a true positive test result. Lyme arthritis typically occurs months after the initial infection and occurs in about 60% of untreated patients. The typical attack is large joint, oligoarticular, and intermittent, lasting weeks at a time. Oligoarticular arthritis carries a broad differential diagnosis, including sarcoidosis, spondyloarthropathy, rheumatoid arthritis, psoriatic arthritis, and Lyme disease. Lyme serology is appropriate in this situation. Patients with Lyme arthritis usually have the highest IgG antibody responses seen in the infection.

IV-128 and IV-129. The answers are D and D, respectively. (Chap. 173) This patient’s rash is a classic erythema migrans lesion and is diagnostic for Lyme disease in her geographic region. In the United States, Lyme disease is caused by infection with Borrelia burgdorferi. Partial central clearing, a bright red border, and a target center are very suggestive of this lesion. The fact that multiple lesions exist implies disseminated infection rather than a primary tick bite inoculation in which only one lesion is present. Potential complications of secondary Lyme disease in the United States include migratory arthritis, meningitis, cranial neuritis, mononeuritis multiplex, myelitis, varying degrees of atrioventricular block, and (less commonly) myopericarditis, splenomegaly, and hepatitis. Third-degree or persistent Lyme disease is associated with oligoarticular arthritis of large joints and subtle encephalopathy but not frank dementia. Borrelia garinii infection is seen only in Europe and can cause a more pronounced encephalomyelitis.

Acute Lyme disease involving the skin or joints (or both) is treated with oral doxycycline unless the patient is pregnant or younger than 9 years old. Amoxicillin and macrolides (azithromycin) are less effective therapies. Ceftriaxone is indicated for acute disease in the presence of nervous system involvement (meningitis, facial palsy, encephalopathy, radiculoneuritis) or third-degree heart block. It may also be used for treatment of patients with arthritis who do not respond to oral therapy. First-generation cephalosporins are not active against B. burgdorferi. Although the rash of erythema migrans may look like cellulitis caused by staphylococci or streptococci, there is no proven efficacy of vancomycin for Lyme disease.

IV-130. The answer is E. (Chap. 174) This clinical vignette describes an individual infected with Ehrlichia chaffeensis, the causative agent of human monocytic ehrlichiosis (HME). This rickettsial infection is transmitted through the bite of an infected deer tick and is most common in the southeast, south-central, and mid-Atlantic states. In 2008, the incidence was highest in Arkansas, Oklahoma, and Missouri. This patient is at risk given his occupation, which requires him to spend a significant amount of time outdoors. The time from incubation to symptoms of infection is about 8 days. The most prominent symptoms of HME are nonspecific and include fevers, malaise, headaches, and myalgias. Nausea, vomiting, diarrhea, cough, confusion, and rash are less common. HME can be quite severe with 62% of all individuals requiring hospitalization and a mortality rate of about 3%. In severe cases, septic shock, adult respiratory distress syndrome, and meningoencephalitis can occur. Laboratory findings are helpful in suggesting possible HME. Common findings are lymphopenia, neutropenia, thrombocytopenia, and elevations in aminotransferases. If a bone marrow biopsy is done, the marrow is hypercellular, and noncaseating granulomas can be observed. Diagnosis of HME relies on polymerase chain reaction detection of E. chaffeensis nucleic acids in peripheral blood. Morulae are seen only rarely (<10%) in the cytoplasm of monocytes on peripheral blood smears. Paired sera demonstrating a rise in antibody titers to above 1:64 over a course of about 3 weeks can also be confirmatory. Treatment of HME is oral or intravenous doxycycline that is continued for 3 to 5 days after fever has resolved.

In rare instances, systemic lupus erythematosus could present with a fulminant illness that could include pancytopenia and liver function abnormalities. However, it would be more likely to have a rash and renal involvement, which this patient did not exhibit. Antibody testing for double-stranded DNA and Smith antigens would not be helpful in this case. Because the patient had a normal lumbar puncture result, further testing of the cerebrospinal fluid is unlikely to yield the diagnosis. This testing is most common used to diagnose viral encephalitides or meningitis such as West Nile virus and herpes simplex virus. The presence of noncaseating granulomas on bone marrow biopsy is a nondiagnostic finding. In the appropriate clinical setting, this could be suggestive of sarcoidosis. However, sarcoidosis does not present with a fulminant febrile illness over a matter of days. Moreover, although a chest radiograph may demonstrate hilar adenopathy or lung infiltrate, this, too, is a nondiagnostic finding.

IV-131. The answer is B. (Chap. 174) This patient demonstrates evidence of Rocky Mountain spotted fever (RMSF), which has progressed over the course of several days because of a lack of initial recognition and treatment. RMSF is caused by infection with Rickettsia rickettsii and is transmitted through the bite of an infected dog tick. RMSF has been diagnosed in 47 states and is most commonly diagnosed in the south-central and southeastern states. Symptoms typically begin about 1 week after inoculation. The initial symptoms are vague and are easily misdiagnosed as a viral infection with fever, myalgias, malaise, and headache predominating. Although almost all patients with RMSF develop a rash during the course of the illness, rash is present in only 14% on the first day, and the lack of rash in a patient who is at risk for RMSF should not delay treatment. By day 3, 49% of individuals develop a rash. The rash initially is a macular rash that begins on the wrists and ankles and progresses to involve the extremities and trunk. Over time, hemorrhaging into the macules occurs and has a petechial appearance. As the illness progresses, respiratory failure and central nervous system (CNS) manifestations can develop. Encephalitis, presenting as confusion and lethargy, is present about 25% of the time. Other manifestations can include renal failure, hepatic injury, and anemia. Treatment for RMSF is doxycycline 100 mg twice daily. It can be administered orally or intravenously. Because this patient shows progressive disease with CNS involvement, hospital admission for treatment is warranted to monitor for further decompensation in the patient’s condition. If the patient were more clinically stable, outpatient therapy would be appropriate. Treatment should not be delayed while awaiting confirmatory serologic testing because untreated cases of RMSF are fatal, usually within 8 to 15 days. Treatment with any sulfa drugs should be avoided because these drugs are ineffective and can worsen the disease course. Intravenous ceftriaxone and vancomycin are appropriate agents for bacterial meningitis. Although this could be a consideration in this patient with fever, confusion, and a rash, meningococcemia would present with a more fulminant course, and the patient’s risk factor (hiking in an endemic area) would make RMSF more likely.

IV-132. The answer is D. (Chap. 175) This patient presents with symptoms of atypical pneumonia, and the most common causative organism for atypical pneumonia is Mycoplasma pneumoniae. Pneumonia caused by Mycoplasmaoccurs worldwide without a specific seasonal preference. M. pneumoniae is a highly infectious organism and is spread by respiratory droplets. It is estimated that about 80% of individuals within the same family will experience the infection after one person becomes infected. Outbreaks of M. pneumoniae also occur in institutional settings, including boarding schools and military bases. Clinical manifestations of M. pneumoniae typically are pharyngitis, tracheobronchitis, wheezing, or nonspecific upper respiratory syndrome. Although many commonly believe the organism is associated with otitis media and bullous myringitis, there are little clinical data to support this assertion. Atypical pneumonia occurs in fewer than 15% of individuals infected with M. pneumoniae. The onset of pneumonia typically is gradual with preceding symptoms of upper respiratory infection. Cough is present, and often extensive, but nonproductive. Examination typically demonstrates wheezing or rales in about 80% of patients. The most common radiographic findings are bilateral peribronchial pneumonia with increased interstitial markings. Lobar consolidation is uncommon. Definitive diagnosis requires demonstration of M. pneumoniae nucleic acids on polymerase chain reaction of respiratory secretions or performance of serologic testing. Often, however, the patients are treating empirically without obtaining definitive diagnosis.

Other causes of atypical pneumonia are Chlamydia pneumoniae and Legionella pneumophila. C. pneumoniae more commonly causes pneumonia in school-aged children, although adults can become re-infected. Legionellapneumonia is often associated with outbreaks of disease caused by contaminated water supplies. Individuals with Legionella pneumonia can become quite sick and develop respiratory failure. Adenovirus is a common viral cause of upper respiratory tract infection and has been associated with outbreaks of pneumonia among military recruits.

Streptococcus pneumoniae is the most common cause of community-acquired pneumonia, but it typically presents with lobar or segmental consolidation.

IV-133. The answer is D. (Chap. 175) Mycoplasma pneumoniae is a common cause of pneumonia that is often underdiagnosed based on difficult and time-consuming culture techniques, because it likely causes mild respiratory symptoms, and because it is adequately treated with standard antibiotic regimens for community-acquired pneumonia. It is spread easily person to person, and outbreaks in crowded conditions, such as schools or barracks, are common. Most patients develop a cough without radiographic abnormalities. When radiographic abnormalities are present, there is usually a diffuse bronchopneumonia pattern without any lobar consolidation. Pharyngitis and rhinitis are also common. M. pneumoniae commonly induces the production of cold agglutinins, which in turn can cause an IgM- and complement-mediated intravascular hemolytic anemia. The presence of cold agglutinins is specific for M. pneumoniae infection only in the context of a consistent clinical picture for infection, as in this patient. Cold agglutinins are more common in children. Blood smear shows no abnormality, which is in contrast to IgG or warm-type hemolytic anemia in which spherocytes are seen. Because there is no easy diagnostic test, empirical therapy is often administered.

IV-134. The answer is C. (Chap. 176) This patient likely has pneumonia caused by infection with Chlamydia psittaci. This organisms is a relatively rare cause of pneumonia with only about 50 confirmed cases yearly in the United States. Contrary to common belief, the organism is not limited to psittacine birds (parrots, parakeets, cockatiels, macaws), but any bird can be infected, including poultry. Most infections are seen in owners of pet birds, poultry farmers, or poultry processing workers, and outbreaks of pneumonia have been seen in poultry processing factories. Untreated psittacosis has a mortality rate of as high as 10%. The illness presents with nonspecific symptoms of fevers, chills, myalgias, and severe headache. Gastrointestinal symptoms with hepatosplenomegaly are also common. Severe pneumonia requiring ventilatory support can occur, and other rare manifestations include endocarditis, myocarditis, and neurologic complications. The current diagnostic tool of choice is the microimmunofluorescence test, which is a serologic test. Any titer greater than 1:16 is considered evidence of exposure to psittacosis, and paired acute and convalescent titers showing a fourfold rise in titer are consistent with psittacosis. Complement fixation tests are also used. Treatment of choice for psittacosis is tetracycline 250 mg four times daily for a minimum of 4 weeks. Public health officials should be notified to assess other workers in the factory for disease and limit exposure.

Although this patient has immigrated from an area endemic for tuberculosis, she had a previous negative purified protein derivative result and no known tuberculosis exposures. Her chest radiograph shows diffuse consolidation, which would not be typical for reactivation of tuberculosis. Systemic infection with Staphylococcus aureus from an abscess or endocarditis could present with respiratory failure related to septic emboli. However, her chest imaging is not consistent with this, and she has no risk factors (i.e., intravenous drug use, indwelling intravenous catheter) for development of S. aureus bloodstream infection. Legionella pneumophila is associated with outbreaks of disease related to contaminated water supplies or air conditioning. It should be considered in this patient in light of her ill coworkers. However, hepatosplenomegaly is not consistent with this diagnosis. Influenza A is also a consideration for this patient, but the time of year is not consistent for seasonal influenza. In outbreaks of pandemic influenza, this would be more likely.

IV-135. The answer is D. (Chap. 176) Congenital infection from maternal transmission can lead to severe consequences for the neonate; thus, prenatal care and screening for infection are very important. Chlamydia trachomatis is associated with up to 25% of exposed neonates who develop inclusion conjunctivitis. It can also be associated with pneumonia and otitis media in the newborn. Pneumonia in the newborn has been associated with later development of bronchitis and asthma. Hydrocephalus can be associated with toxoplasmosis. Hutchinson triad, which is Hutchinson teeth (blunted upper incisors), interstitial keratitis, and eighth nerve deafness, is caused by congenital syphilis. Sensorineural deafness can be associated with congenital rubella exposure. Treatment of C. trachomatis infection in infants consists of oral erythromycin.

IV-136. The answer is D. (Chap. 176) Urethritis in men causes dysuria with or without discharge, usually without frequency. The most common causes of urethritis in men include Neisseria gonorrhoeaeChlamydia trachomatis, Mycoplasma genitaliumUreaplasma urealyticumTrichomonas vaginalis, herpes simplex virus, and possibly adenovirus. Until recently, C. trachomatis accounted for 30% to 40% of cases; however, this number may be decreasing. Recent studies suggest that M. genitalium is a common cause of nonchlamy-dial cases. Currently, the initial diagnosis of urethritis in men includes specific tests only for N. gonorrhoeae and C. trachomatis. Tenets of urethral discharge treatment include providing treatment for the most common causes of urethritis with the assumption that the patient may be lost to follow-up. Therefore, prompt empirical treatment for gonorrhea and Chlamydia infections with ceftriaxone and azithromycin should be given on the day of presentation to the clinic to the patient, and recent partners should be contacted for treatment. Azithromycin will also be effective for M. genitalium. If pus can be milked from the urethra, cultures should be sent for definitive diagnosis and to allow for contact tracing by the health department because both of these are reportable diseases. Urine nucleic acid amplification tests are an acceptable substitute in the absence of pus. It is also critical to provide empirical treatment for at-risk sexual contacts. If symptoms do not respond to the initial empirical therapy, patients should be reevaluated for compliance with therapy, reexposure, and T. vaginalis infection.

IV-137. The answer is A. (Chap. 177) Persistent viral infection is speculated to be pathogenically important in up to 20% of human malignancies. Strong associations based on epidemiology, the presence of viral nucleotides in tumor cells, the transformational ability of viruses on human cells, and animal models have been established. Most hepatocellular carcinoma is thought to be related to chronic infection with hepatitis B or C virus. Most cervical cancer is caused by persistent infection with human papilloma virus type 16 or 18. Epstein-Barr virus plays a role in the development of many B-lymphocyte and epithelial cell malignancies such as Hodgkin’s lymphoma, Burkitt’s lymphoma, and nasopharyngeal carcinoma. HTLV-1 is associated with a number of T-cell lymphomas and leukemias. KSHV (Kaposi’s sarcoma associated herpes virus, HHV-8) is associated with Kaposi’s sarcoma, pleural effusion lymphoma, and multicentric Castleman’s disease. Dengue fever virus, a flavovirus, is the cause of dengue fever and has not been associated with human malignancy.

IV-138. The answer is A. (Chap. 178) Compared with the large number of antimicrobials directed against bacterial, antiviral therapies have been fewer, and advances in antiviral therapy have come more slowly. However, in recent years, a large number of antiviral medications have been introduced, and it is generally important to be familiar with the common side effects of these medications. Acyclovir and valacyclovir are most commonly used for the treatment of herpes simplex viruses I and II as well as varicella-zoster virus. Acyclovir is generally a well-tolerated drug, but it can crystallize in the kidneys, leading to acute renal failure if the patient is not properly hydrated. Valacyclovir is an ester of acyclovir that significantly improves the bioavailability of the drug. It is also well tolerated but has been associated with thrombotic thrombocytopenic purpura or hemolytic uremic syndrome when used at high doses. Ganciclovir and foscarnet are medications used to treat cytomegalovirus (CMV) infection. Ganciclovir is primarily given intravenously because the oral bioavailability is less than 10%. Ganciclovir is associated with bone marrow suppression and can cause renal dysfunction. Foscarnet is used for ganciclovir-resistant CMV infections. Renal impairment commonly occurs with its use and causes hypokalemia, hypocalcemia, and hypomagnesemia. Thus, careful monitoring of electrolytes and renal function is warranted with foscarnet use. Amantadine is an antiviral medication used for the treatment of influenza A. It has been demonstrated to have a variety of central nervous system (CNS) side effects, including dizziness, anxiety, insomnia, and difficulty concentrating. Although initially used as an antiviral drug, the CNS effects of amantadine have led to its use in Parkinson’s disease. Interferons are a group of cytokines produced endogenously in response to a variety of pathogens, including viruses and bacteria. Therapeutically, interferons have been studied extensively in the treatment of patients with chronic hepatitis B and C. Interferons lead to a host of systemic effects, including symptoms of a viral syndrome (fevers, chills, fatigue, and myalgias) as well as leukopenia.

IV-139. The answer is E. (Chap. 179) Antibodies to herpes simplex virus-2 (HSV-2) are not routinely detected until puberty, consistent with the typical sexual transmission of the virus. Serosurveys suggest that 15% to 20% of American adults have HSV-2 infection; however, only 10% report a history of genital lesions. Seroprevalence is similar or higher in Central America, South America, and Africa. Recent studies in African obstetric clinics have found seroprevalence rates as high as 70%. HSV-2 infection is believed to be so pervasive in the general population based on ease of transmission, both in symptomatic and asymptomatic states. Therefore, this sexually transmitted disease (STD) is significantly more common in individuals who less frequently engage in high-risk behavior than other STDs. HSV-2 is an independent risk factor for HIV acquisition and transmission. HIV virion is shed from herpetic lesions, thus promoting transmission.

IV-140. The answer is E. (Chap. 179) Primary genital herpes caused by herpes simplex virus-2 (HSV-2) is characterized by fever, headache, malaise, inguinal lymphadenopathy, and diffuse genital lesions of varying stage. The cervix and urethra are usually involved in women. Although both HSV-2 and HSV-1 can involve the genitals, the recurrence rate of HSV-2 is much higher (90% in the first year) than with HSV-1 (55% in the first year). The rate of reactivation for HSV-2 is very high. Acyclovir, valacyclovir, and famciclovir are effective in shortening the duration of symptoms and lesions in genital herpes. Chronic daily therapy can reduce the frequency of recurrences in those with frequent reactivation. Valacyclovir has been shown to reduce transmission of HSV-2 between sexual partners.

IV-141. The answer is C. (Chap. 179) Herpes encephalitis accounts for 10% to 20% of sporadic cases of viral encephalitis in the United States. It most commonly occurs in patients 5 to 30 years and older than 50 years old. HSV-1 accounts for more than 95% of cases, and most adults have clinical or serologic evidence of HSV-1 mucocutaneous infection before onset of central nervous system symptoms. Herpes simplex virus (HSV) encephalitis is characterized by the acute onset of fever and focal neurologic signs, particularly in the temporal lobe. Electroencephalographic (EEG) abnormalities in the temporal lobe are common. The cerebrospinal fluid (CSF) will show elevated protein, lymphocyte leukocytosis with red blood cells, and normal glucose. HSV polymerase chain reaction testing of CSF is highly sensitive and specific for diagnosis. Treatment with acyclovir reduces mortality; however, neurologic sequelae are common, particularly in older patients. Differentiation of HSV encephalitis from other viral forms is difficult. Most experts recommend initiation of empiric acyclovir in any patient with suspected encephalitis pending a confirmed or alternative diagnosis. Tuberculosis meningitis presents typically as a basilar meningitis and not encephalitis. The history, clinical findings, EEG abnormalities, and radiologic findings make fungal meningitis unlikely. CSF oligoclonal bands are typically seen in patient with multiple sclerosis.

IV-142. The answer is C. (Chap. 180) Recently, a varicella-zoster virus vaccine that has 18 times the viral content of the live-attenuated virus vaccine used in children was shown efficacious for shingles in patients older than 60 years of age. The vaccine decreased the incidence of shingles by 51%, the burden of illness by 61%, and the incidence of postherpetic neuralgia by 66%. The Advisory Committee on Immunization Practices has therefore recommended that persons in this age group be offered this vaccine to reduce the frequency of shingles and the severity of postherpetic neuralgia. Because is it a live virus vaccine, it should not be used in immunocompromised patients.

IV-143 and IV-144. The answers are C and E, respectively. (Chap. 181) Epstein-Barr virus (EBV) is the cause of heterophile-positive infectious mononucleosis (IM), which is characterized by fever, sore throat, lymphadenopathy, and atypical lymphocytosis. EBV is also associated with several human tumors, including nasopharyngeal carcinoma, Burkitt’s lymphoma, Hodgkin’s disease, and (in patients with immunodeficiencies) B-cell lymphoma. EBV infection occurs worldwide with more than 90% of adults seropositive. In the developing world, most are infected as young children, and IM is uncommon; in the more developed world, most are infected as adolescents or young adults, and IM is more common. The virus is spread by contaminated saliva. Asymptomatic seropositive individuals shed the virus in saliva. In young children, the EBV infection causes mild disease with sore throat. Adolescents and young adults develop IM as described above plus often splenomegaly in the second to third week of disease. The white blood cell count is usually elevated and peaks at 10,000 to 20,000/μL during the second or third week of illness. Lymphocytosis is usually demonstrable, with greater than 10% atypical lymphocytes. A morbilliform rash may occur in about 5% of patients as part of the acute illness. Most patients treated with ampicillin develop a macular rash as pictured; this rash is not predictive of future adverse reactions to penicillins. Heterophile antibody testing results will be positive in up to 40% of cases of IM in the first week of illness and up to 90% by the third week. If heterophile antibody testing results are negative, the more expensive testing for immunoglobulin M (IgM) antibodies to viral capsid antigen is more sensitive and specific. IgG antibodies to viral capsid antigen will stay present indefinitely after initial infection and are not useful for diagnosing acute disease. Treatment of uncomplicated IM is with rest, supportive measures, and reassurance. Excessive physical activity should be avoided in the first month to avoid splenic trauma. Prednisone is not indicated and may predispose to secondary infection. It has been used at high dose when IM is complicated by airway compromise caused by pharyngeal swelling, autoimmune hemolytic anemia, severe thrombocytopenia, hemophagocytic syndrome, or other severe complications. Controlled trials have shown that acyclovir has no significant impact on the course of uncomplicated IM. One study showed no benefit for combined prednisone plus acyclovir.

IV-145. The answer is D. (Chap. 182) Cytomegalovirus (CMV) retinitis, a common CMV infection in HIV patients, occurs less commonly in solid organ transplant patients. CMV does affect the lung in a majority of transplant patients if either the donor or recipient is CMV-seropositive pretransplant. CMV disease in transplant recipients typically develops 30 to 90 days after transplant. It rarely occurs within 2 weeks of transplantation. CMV very commonly causes a pneumonitis that clinically is difficult to distinguish from acute rejection. Prior CMV infection has been associated with bronchiolitis obliterans syndrome (chronic rejection) in lung transplant recipients. As with HIV, the gastrointestinal tract is commonly involved with CMV infection. Endoscopy with biopsy showing characteristic giant cells, not serum polymerase chain reaction (PCR), is necessary to make this diagnosis. The CMV syndrome is also common in lung transplant patients. Serum CMV PCR should be sent as part of the workup for all nonspecific fevers, worsening lung function, liver function abnormalities, or falling leukocyte counts occurring more than a couple of weeks after transplant.

IV-146. The answer is A. (Chap. 182) When the transplant donor is cytomegalovirus (CMV) immunoglobulin G (IgG) positive and the recipient is negative, there is a very high risk of primary CMV infection in the recipient. However, if the recipient is IgG positive, CMV occurs as a reactivation infection. When both the donor and recipient are seronegative, then the risk of any CMV infection is lowest, but not zero, because contact with an infected host could prompt primary CMV infection. Unlike nearly all other transplant patients, many donor and recipient seronegative patients do not receive chemoprophylaxis with ganciclovir. In patients who are CMV IgG negative and received a CMV IgG negative transplant, transfusions should be from CMV IgG negative donors or white blood cell filtered products administered to reduce the risk of primary CMV infection. It is not clear whether universal prophylaxis or preemptive therapy is the preferable approach in CMV-seropositive immunocompromised hosts. Both ganciclovir and valganciclovir have been used successfully for prophylaxis and preemptive therapy in transplant recipients. A CMV glycoprotein B vaccine reduced infections in a placebo-controlled trial among 464 CMV-seronegative women; this outcome raises the possibility that this experimental vaccine will reduce congenital infections, but further studies must validate this approach.

IV-147. The answer is A. (Chap. 182) Human herpes virus-8 (HHV-8) or Kaposi’s sarcoma–associated herpes virus (KSHV infects B lymphocytes, macrophages, and both endothelial and epithelial cells) appears to be causally related to Kaposi’s sarcoma and a subgroup of AIDS-related B-cell body cavity–based lymphomas (primary effusion lymphomas) and to multicentric Castleman’s disease. HHV-8 infection is more common in parts of Africa than in the United States. Primary HHV-8 infection in immunocompetent children may manifest as fever and maculopapular rash. Among individuals with intact immunity, chronic asymptomatic infection is the rule, and neoplastic disorders generally develop only after subsequent immunocompromise. In patients with AIDS, effective antiretroviral therapy has caused improvement in HHV-8–related disease. The virus is sensitive to ganciclovir, foscarnet, and cidofovir, but clinical benefit has not been demonstrated in trials. Invasive cervical carcinoma has been causally implicated with human papilloma virus infection.

IV-148. The answer is B. (Chap. 183) Molluscum contagiosum is a cutaneous poxvirus infection with a distinctive cutaneous appearance. The rash typically consists of collections of 2- to 5-mm umbilicated papules that can occur anywhere on the body except the palms and soles. It can be accompanied by an eczematous reaction. Molluscum contagiosum is transmitted through close contact, including sexual contact, which causes genital involvement. Unlike other poxvirus lesions, molluscum contagiosum is not associated with inflammation or necrosis. In immunocompetent patients, the disease is usually self-limited; rash will subside within several months. Systemic involvement does not occur.

IV-149. The answer is C. (Chap. 184) Immunocompromised patients occasionally cannot clear parvovirus infection because of a lack of T-cell function. Because parvovirus B19 selectively infects red cell precursors, persistent infection can lead to a prolonged red blood cell aplasia and a persistent drop in hematocrit, with low or absent reticulocytes. Pure red blood cell aplasia has been reported in HIV infection, lymphoproliferative diseases, and after transplantation. Iron studies show adequate iron but decreased utilization. The peripheral smear usually shows no abnormalities other than normocytic anemia and the absence of reticulocytes. Antibody tests are not useful in this setting because immunocompromised patients do not produce adequate antibodies against the virus. Therefore, polymerase chain reaction (PCR) is the most useful diagnostic test. Bone marrow biopsy may be suggestive because it will show no red blood cell precursors, but usually a less invasive PCR test is adequate. Immediate therapy is with red blood cell transfusion followed by intravenous immunoglobulins, which contain adequate titers of antibody against parvovirus B19.

IV-150. The answer is D. (Chap. 184) The most likely diagnosis based on the patient’s antecedent illness with a facial rash is parvovirus infection. Arthropathy is uncommon in childhood parvovirus infection but may cause a diffuse symmetric arthritis in up to 50% of adults. This corresponds to the immune phase of illness when immunoglobulin M antibodies are developed. The arthropathy syndrome is more common in women than men. The distribution of affected joints is typically symmetric, most commonly in the small joints of the hands and less commonly the ankles, knees, and wrists. Occasionally the arthritis persists over months and can mimic rheumatoid arthritis. Rheumatoid factor can be detected in serum. Parvovirus B19V infection may trigger rheumatoid disease in some patients and has been associated with juvenile idiopathic arthritis. Reactive arthritis caused by Chlamydia spp. or a list of other bacterial pathogens tends to affect large joints such as the sacroiliac joints and spine. It is also sometimes accompanied by uveitis and urethritis. The large number of joints involved with a symmetric distribution argues against crystal or septic arthropathy.

IV-151. The answer is E. (Chap. 185) The currently available human papillomavirus (HPV) vaccines dramatically reduce rates of infection and disease produced by the HPV types in the vaccines. These products are directed against virus types that cause anogenital tract disease. Both vaccines consist of virus-like particles without any viral nucleic acid and therefore are not active. To date, one quadrivalent product (Gardasil, Merck) containing HPV types 6, 11, 16, and 18 and one bivalent product (Cervarix, GlaxoSmithKline) containing HPV types 16 and 18 have been licensed in the United States. HPV types 6 and 11 cause 90% of anogenital warts, and types 16 and 18 are responsible for 70% of cervical cancers. Efficacy has varied according to the immunologic and virologic characteristics of study populations at baseline and according to the endpoints evaluated. Among study participants who are shown at baseline not to be infected with a specific virus type contained in the vaccine and who adhere to the study protocol, rates of vaccine efficacy regularly exceed 90%, as measured by both infection and disease caused by that specific virus type. Study participants who are already infected at baseline with a specific virus type contained in the vaccine do not benefit from vaccination against that type but may benefit from vaccination against other virus types contained in the vaccine preparation. Thus, the available HPV vaccines have potent prophylactic effects but no therapeutic effects. The Advisory Committee on Immunization Practices (ACIP) of the Centers for Disease Control and Prevention has recommended that HPV vaccination be routinely offered to girls and young women 9 to 26 years of age. The quadrivalent vaccine has also been licensed in the United States for use in boys and young men; the ACIP has stated that this product may be used to prevent anogenital warts in boys and young men 9 to 26 years of age. Because 30% of cervical cancers are caused by HPV types not contained in the vaccines, no changes in cervical cancer screening programs are currently recommended. Ongoing studies are examining self-testing for HPV to replace many Pap studies in patients with no evidence of cervical infection. Recent studies implicate HPV in some forms of squamous cell carcinoma of the oropharynx. The utility of the current vaccines in preventing these cancers in not yet known.

IV-152. The answer is D. (Chap. 186) This patient presents with symptoms of the common cold with a self-limited illness characterized by rhinorrhea and sore throat. The most common viruses causing the common cold are rhinoviruses, which are implicated in as many as 50% of common colds. Rhinoviruses are small single-stranded RNA viruses of the Picornaviridae family. There are three genetic species of rhinoviruses with 102 serotypes identified. Rhinoviruses grow preferentially at the temperature of the nasal passages (33° to 34°C) rather than the temperature of the lower airways (37°C). Although rhinovirus infections occur year round, there are seasonal peaks of the infection in the early fall and spring in temperate climates. Overall, the rates of rhinovirus infection are highest in infants and young children and decrease with age. The virus is most often introduced into families through young children in preschool or grade school. After the index infection, secondary infections occur in other family members 25% to 70% of the time. Rhinovirus spreads through direct contact with infected secretions, which can occur through respiratory droplets or hand-to-hand contact. It can also be transmitted through large or small particle aerosols. Finally, the virus can be isolated from plastic surfaces from 1 to 3 hours after inoculation, raising the possibility that the virus can also be transmitted through environmental contact.

IV-153. The answer is C. (Chap. 186) Acute viral respiratory illnesses are the most common illness worldwide, and a wide variety of viruses have been implicated as causes. Rhinoviruses are the most common virus causing the common cold and are found in about 50% of cases. The second most commonly isolated viruses are coronaviruses. These viruses are more common in the late fall, winter, and early spring, primarily at times when rhinoviruses are less active. Other causes of common cold in children are adenoviruses, whereas these viruses are uncommon in adults with the exception of outbreaks in individuals living in close quarters such as military recruits. Although human respiratory syncytial virus characteristically causes pneumonia and bronchiolitis in young children, the virus can cause common cold and pharyngitis in adults. Parainfluenza virus is another virus classically associated with croup in children, but it causes common cold in adults. Enteroviruses most often cause an undifferentiated febrile illness.

IV-154. The answer is A. (Chap. 186) The common viruses causing respiratory infections often have specific associated clinical syndromes. Rhinoviruses are primarily responsible for the common cold. Coronaviruses are also commonly associated with colds. However, in 2002 to 2003, there was an outbreak of a coronavirus-associated illness that originated in China and spread to 28 countries in Asia, Europe, and North and South America. This illness was named severe acute respiratory syndrome (SARS) and caused severe lower respiratory illness and acute respiratory distress syndrome. Overall, the case-fatality rate was 9.5%. Human respiratory syncytial virus is the primary agent responsible for lower respiratory disease and bronchiolitis in infants and young children. Another virus primarily associated with childhood illness is parainfluenza virus. This virus is a frequent cause of croup in young children characterized by a febrile illness with a barking cough and stridor. Adenovirus often causes a febrile illness with the common cold and pharyngitis in children. In adults, it is associated with outbreaks of respiratory illness in military recruits. Herpes simplex virus is associated gingivostomatitis in children and pharyngotonsillitis in adults.

IV-155. The answer is C. (Chap. 186) In infants, human respiratory syncytial virus (HRSV) is frequently associated with lower respiratory infections in 25% to 40% of infections. This can present as pneumonia, tracheobronchitis, or bronchiolitis. In cases of lower respiratory infections, tachypnea, wheezing, and hypoxemia are common and can progress to respiratory failure. Treatment is primarily supportive with hydration, suctioning of secretions, and administration of humidified oxygen. Bronchodilators are also used to treat wheezing and bronchospasm. In more severe cases, aerosolized ribavirin has been demonstrated to modestly improve the time to resolution of respiratory illness. The American Academy of Pediatrics states that aerosolized ribavirin may be considered in infants who are seriously ill or who are at high risk of complications, including bronchopulmonary dysplasia and congenital heart disease and those who are immunosuppressed. However, no benefit has been demonstrated with use of standard intravenous immunoglobulin or HRSV-specific immunoglobulin.

IV-156. The answer is E. (Chap. 187) Pandemic strains of influenza emerge through genetic reassortment of RNA segments between viruses that affect different species, including humans, swine, and birds. This process is also called antigenic shift during which a new strain of influenza emerges to which very few people have immunity. Antigenic shift only occurs with influenza A because it is the only influenza that crosses between species. Antigenic drift is the result of point mutations in the hemagglutinin or neuramidase proteins. Antigenic drift occurs frequently and is responsible for the interpandemic influenza outbreaks.

IV-157. The answer is A. (Chap. 187, www.cdc.gov/flu) This patient is presenting with an influenza-like illness during the typical flu season. Hospital infection control practices in this setting are to treat all patients presenting with an influenza-like illness as if they have influenza until proven otherwise. This includes institution of droplet precautions to prevent spread to other individuals as well as performing testing to confirm influenza diagnosis. This is most commonly done via a nasopharyngeal swab but can also be done on throat swab, sputum, nasotracheal aspirates, or bronchoscopic specimens if available. If influenza diagnosis is confirmed, assessment of close household contacts for individuals who may be candidates for chemoprophylaxis against influenza is important, particularly individuals who would be high risk of complications from influenza infection. This group includes children younger than 4 years old, pregnant women, individuals age 65 years or older, individuals with heart or lung disease, individuals with abnormal immune systems, and individuals with chronic metabolic diseases or renal disease.

As far as treatment is concerned, clearly, oxygen should be given to individuals who are hypoxemic. Other appropriate supportive care should also be administered, including intravenous fluids and respiratory therapy support to manage secretions. In general, treatment with antiviral medications has been demonstrated to decrease the duration of symptoms by 1 to 1.5 days when initiated within the first 48 hours after the onset of symptoms. However, the Centers for Disease Control and Prevention (CDC) recommends treatment with antiviral medications as early as possible in patients hospitalized with severe pneumonia. Some evidence indicates that use of antiviral therapy might be effective in decreasing morbidity and mortality in individuals who are hospitalized with severe pneumonia even when given more than 48 hours after the onset of symptoms. The preferred class of antiviral therapy is the neuraminidase inhibitors, which have efficacy against both influenza A and B. In addition, resistance is much lower among this class of medications. This class includes the drugs zanamivir, oseltamivir, and peramivir. Of these, oseltamivir is most commonly used because it is an oral medication with limited side effects. Zanamivir is given by inhalation and can cause bronchoconstriction in individuals with asthma. Peramivir is currently an investigational medication that is administered intravenously.

The adamantine agents include amantadine and rimantadine. These medications have no efficacy against influenza B and also have a high degree of antiviral resistance (>90%) in North America to H3N2 strains of influenza A. It is important to know the resistance patterns to antiviral agents during the local flu season. The CDC currently does not recommend amantadine as first-line therapy for severe influenza. Antibacterial therapy should be reserved for individuals with suspected bacterial complications of influenza.

IV-158. The answer is A. (Chap. 187, www.cdc.gov/flu) The intranasal influenza vaccine contains a cold-adapted, live-attenuated influenza virus. Because of this, its use is more limited than intramuscular inactivated influenza vaccine. It is currently only recommended for healthy individuals between the ages of 2 and 49 years. Contraindications to the use of the live-attenuated intranasal influenza vaccine include pregnancy, chronic pulmonary or cardiovascular conditions, immunosuppressed patients, individuals with a history of Guillain-Barré syndrome, and individuals with a history of severe egg allergy. In addition, individuals who regularly are in contact with immunosuppressed people should not receive the intranasal vaccine, or if it is given, they should avoid contact with severely immunosuppressed patients for 7 days after receipt of the vaccine. In young children, a history of asthma precludes the use of intranasal influenza vaccination because the vaccine can precipitate episodes of wheezing. The Centers for Disease Control and Prevention recommends avoiding the use of the live-attenuated vaccine in individuals whose parents report an episode of wheezing within the past 12 months regardless of whether asthma is a chronic diagnosis. A final caveat in the use of the live-attenuated intranasal influenza vaccine is its use in individuals who are actively taking antiviral medications as chemoprophylaxis against influenza because antiviral medications interfere with the immune response to the live-attenuated influenza vaccine. The current recommendations are that live-attenuated influenza vaccination not be administered for 48 hours after antiviral medications have been stopped and that antiviral medications not be given for 2 weeks after receipt of the live-attenuated influenza vaccine. However, the immune response to the intramuscular inactivated vaccine is not affected by coadministration with antiviral drugs.

IV-159. The answer is D. (Chap. 187) The majority of influenza infections are clinically mild and self-limited. Treatment with over-the-counter cough suppressants and analgesics such as acetaminophen is often adequate. Patients who are younger than the age of 18 years are at risk of developing Reye’s syndrome if they are exposed to salicylates such as aspirin. The neuraminidase inhibitors oseltamivir and zanamivir have activity against influenza A and B. They can be used within 2 days of symptom onset and have been shown to reduce the duration of symptoms by 1 or 2 days. This patient has had symptoms for more than 48 hours, so neither drug is likely to be effective. The patient’s history of asthma is an additional contraindication to zanamivir because this drug can precipitate bronchospasm. The M2 inhibitors amantadine and rimantadine have activity against influenza A only. However, since 2005, more than 90% of A/H3N2 viral isolates demonstrated resistance to amantadine, and these drugs are no longer recommended for use in influenza A.

IV-160. The answer is C. (Chap. 188) Human T-cell lymphotropic virus-I (HTLV-I) is a retrovirus that is a chronic infection like HIV, but it does not cause similar sequelae. It was the first identified human retrovirus. Gradual decline of CD4+ lymphocyte number and function is a feature of HIV but not of HTLV-I. Although many people in endemic areas have serologic evidence of infection, most do not develop disease. The two major complications of HTLV-I are tropical spastic paraparesis and acute T-cell leukemia. Tropical spastic paraparesis is an upper motor neuron disease of insidious onset leading to weakness, lower extremity stiffness, urinary incontinence, and eventually a thoracic myelopathy, leading to a bedridden state in about one-third of patients after 10 years. It is more common in women than men. It can easily be confused with multiple sclerosis; this is why it is important to be able to recall the geographic regions where HTLV-I is endemic when evaluating a myelopathy. Acute T-cell leukemia is a difficult-to-treat leukemia that is specific to chronic HTLV-I infection. HTLV-I is thought to be transmitted in a similar fashion to HIV.

IV-161. The answer is C. (Chap. 189) Clindamycin plus primaquine is a therapeutic, not prophylactic, regimen for mild to moderate disease caused by Pneumocystis infection. Trimethoprim–sulfamethoxazole is usually given as a first-line agent but carries a significant side effect profile that includes hyperkalemia; renal insufficiency; elevation of serum creatinine; granulocytopenia; hemolysis in persons with G6PD insufficiency; and frequent allergic reactions, particularly in those with severe T-cell deficiency. Atovaquone is a common alternative that is given at the same dose for Pneumocystisprophylaxis as for therapy. Gastrointestinal symptoms are common with atovaquone. Aerosolized pentamidine can be given on a monthly basis with a risk of bronchospasm and pancreatitis. Patients who develop Pneumocystis pneumonia while receiving aerosolized pentamidine often have upper lobe–predominant disease. Dapsone is commonly used for Pneumocystis prophylaxis; however, the physician must be aware of the possibility of methemoglobinemia, G6PD-mediated hemolysis, rare hepatotoxicity, and rare hypersensitivity reaction when using this medicine.

IV-162. The answer is C. (Chap. 189) HIV/AIDS continues to have an extraordinary public health impact in the United States. As of January 1, 2010, an estimated 1,108,611 cases of AIDS had been diagnosed in the United States. Approximately 1.1 million individuals in the United States were living with HIV infection, approximately 20% of whom are unaware of their infection. Approximately two-thirds of those living with HIV/AIDS were nonwhite, and nearly half (48%) were men who have sex with men. The annual death rate from HIV/AIDS has fallen steadily in the past 15 years. This trend is attributable to several factors, including improved prophylaxis and treatment of opportunistic infections, the growing experience among health professions in caring for HIV-infected individuals, improved access to health care, and a decrease in new infections because of saturational effects and prevention efforts. However, the most influential factor clearly has been the increased use of potent antiretroviral drugs, generally administered in a combination of three or four agents. After a decrease in the percentage of new HIV infections attributed to male-to-male sexual contact from 1985 to 1999, this percentage has increased. In 2009, this transmission category accounted for 48% of all AIDS diagnoses. The estimated percentage of AIDS diagnoses attributed to injection drug use increased from 20% to 31% from 1985 to 1994 and decreased since that time, accounting for 15% of diagnoses in 2009. The estimated percentage of AIDS diagnoses attributed to heterosexual contact increased from 3% in 1985 to 31% in 2009. HIV infection and AIDS have disproportionately affected minority populations in the United States. Among those diagnosed with HIV (regardless of AIDS status) in 2009, 52% percent were blacks, a group that constitutes only 12% of the U.S. population.

IV-163. The answer is C. (Chap. 189) The quoted risk for HIV transmission via a needle stick is 0.3%. This risk can be reduced to less than 0.1% if the at-risk health care worker is treated with antiretroviral therapy within 24 hours. The risk of transmission is likely highly variable according to a number of factors. Large-bore needle sticks where infected patient blood is visible are higher risk, as are deep tissue puncture to the health care provider. The patient’s degree of virologic control is generally inferred to be critical as well. Patients with viral loads below 1500/mL are considerably less likely to transmit via a needle stick than those with high viral loads. An extension of this point is that during acute and end-stage HIV infection, viral loads are extremely high, and contagion by needle stick is likely to be much higher. In addition, during end-stage disease, virulent viral forms predominate, which may increase the risk to an even greater extent. Each of these variables must be assessed rapidly after an accidental high-risk needle stick. Antiretroviral therapy (ART) is effective at preventing HIV transmission via needle stick if given before viral RNA incorporates into the host genome as proviral DNA. This is thought to occur within about 48 hours, but under the best scenario, ART should be given within 1 hour of a needle stick. Circumstances are often murky, with key information such as viral load, viral resistance history, and even HIV serostatus of the patient variably available; therefore, urgent consultation with an HIV or occupational health specialist is imperative after a needle stick. (Hepatitis B and C transmission must also be considered.)

IV-164. The answer is E. (Chap. 189) Abacavir use is associated with a potentially severe hypersensitivity reaction in about 5% of patients. There is likely a genetic component, with HLA-B*5701 being a significant risk factor for hypersensitivity syndrome. Symptoms, which usually occur within 2 weeks of therapy but can take more than 6 weeks to emerge, include fever, maculopapular rash, fatigue, malaise, gastrointestinal symptoms, and dyspnea. When a diagnosis is suspected, the drug should be stopped and never given again because rechallenge can be fatal. For this reason, both the diagnosis and patient education after the diagnosis is made must be performed thoroughly and carefully. It is important to note that two available combination pills contain abacavir (Epzicom, Trizivir), so patients must know to avoid these as well. Fanconi’s anemia is a rare disorder associated with tenofovir. Zidovudine causes anemia and sometimes granulocytopenia. Stavudine and other nucleoside reverse transcriptase inhibitors are associated with lipoatrophy of the face and legs.

IV-165. The answer is B. (Chap. 189) Cytomegalovirus (CMV) colitis should be considered in AIDS patients with CD4+ lymphocyte counts below 50/μL, fevers, and diarrhea. Diarrhea is often bloody but can be watery. Initial evaluation often involves stool studies to rule out other parasitic or bacterial causes of diarrhea in AIDS patients. A standard panel includes some or all of the following depending on epidemiologic and historical data: Clostridium difficile stool antigen; stool culture; stool Mycobacterium avium intracellulare culture; stool ova and parasite examination; and special stains for Cryptosporidium, Isospora, Cyclospora, and Microsporidium spp. There is no stool or serum test that is useful for the evaluation of CMV colitis in an HIV-infected patient. A positive CMV immunoglobulin G result is merely a marker of past infection. If this test result is negative, then the pretest probability of developing active CMV decreases substantially. Serum CMV polymerase chain reaction (PCR) has gained utility in solid organ and bone marrow transplant patients for following treatment response for invasive CMV infection. However, in HIV-infected patients, CMV viremia correlates imprecisely with colitis. Furthermore, because CMV is a latent-lytic herpesvirus, a positive serum PCR does not imply disease unless drawn in the right clinical context, for which there is none in HIV infection. Colonic histology is sensitive and specific for the diagnosis of CMV colitis, with large-cell inclusion bodies being diagnostic.

IV-166. The answer is C. (Chap. 189) Immune reconstitution syndrome (IRIS) is commonly seen after the initiation of antiretroviral therapy (ART) in patients with AIDS and a concomitant opportunistic infection (OI). It is a syndrome in which either a previously recognized OI worsens after ART despite an initial period of improvement after standard therapy for that particular infection or in which an OI that was not previously recognized is unmasked after ART therapy. The latter scenario occurs presumably as immune cells become reactivated and recognize the presence of a pathogen that disseminated in the absence of adequate T-cell response with the patient remaining subclinical before ART. Many opportunistic pathogens are known to behave this way, but Cryptococcus spp., Mycobacterium tuberculosis, and Mycobacterium avium complex (MAI/MAC) are the most likely to be associated with IRIS. Risk factors for IRIS are a CD4+ lymphocyte count below 50/μL at ART initiation, initiation of ART within 2 months of treatment initiation for the OI, adequate virologic response to ART, and increase in CD4+ lymphocyte count as a result of ART. IRIS can be diagnostically challenging and is very diverse in terms of clinical presentation and severity. Depending on the organ system and pathogen involved, drug-resistant OI and new OI must be considered, sometimes necessitating invasive biopsies and cultures. In this case, the overlap of organ system with the original presentation, low likelihood of MAI drug resistance, and timing of the syndrome favor IRIS. Therapy is with nonsteroidal anti-inflammatory drugs and sometimes glucocorticoids. OI treatment is continued, and all efforts are made to continue ART as well except under the most dire of clinical circumstances.

IV-167. The answer is E. (Chap. 189) The biologic determinants of HIV transmission and acquisition are complex and have been difficult to study. However, several key factors are now known to increase the per-coital rate of HIV transmission, at least for heterosexual couples. In discordant couples, there is a dose-dependent relationship between serum viral load and HIV transmission. In fact, in carefully done studies, there was virtually no transmission between discordant couples when serum viral load was low (<400/mL). It is likely that this is attributable to a fairly tight correlation between serum and genital viral load. A corollary is that during acute HIV or AIDS, the viral load and therefore transmissibility are high. Strong clinical data from randomized trials indicate that circumcised men are less likely to acquire HIV because the interior surface of the foreskin is replete with cellular targets for HIV infection. Nonulcerative sexually transmitted infections cause mucosal breakdown that has been shown to allow for greater acquisition of HIV infection. Herpes simplex virus-2 (HSV-2) carriage (not necessarily requiring active genital ulcer disease) leads to increases in HIV genital shedding as well as HIV-1 target cell migration to the genital mucosa, making both transmission and acquisition of HIV higher in HSV-2–positive persons.

IV-168. The answer is B. (Chap. 189) Centers for Disease Control and Prevention guidelines now state that all adults should receive HIV testing, with the availability of a patient opt-out mechanism rather than informed consent. The basis for this is that about 25% of the 1 million Americans infected with HIV are unaware of their status, there is good available treatment for HIV that serves to extend the lifespan and decrease HIV transmission, and HIV testing is shown to correlate with a decrease in risk-taking behaviors. Cost-benefit analysis has suggested this approach has advantages to current approaches focusing on screening high-risk populations. Pretest counseling is desirable but not always built into the testing process, so physicians should provide some degree of preparation for a positive test result. If the diagnosis is made, support systems should be activated that may include trained nurses, social workers, or community support centers.

IV-169. The answer is A. (Chap. 189) This patient most likely has HIV encephalopathy of moderate severity. Other neurologic conditions associated with HIV may be considered with a broad initial workup, but her reasonably high CD4+ count, lack of focal deficits, and lack of mass lesions on high-resolution brain imaging makes toxoplasmosis, central nervous system (CNS) tuberculoma, progressive multifocal leukoencephalopathy (PML), and CNS lymphoma all less unlikely. Immediate highly active antiretroviral therapy is the treatment of choice for HIV encephalopathy, and she warrants this despite her CD4+ lymphocyte count, placing her in a gray zone according to current guidelines in regards to starting therapy. A lumbar puncture for the Venereal Disease Research Laboratory test is unnecessary because a serum rapid plasma reagin test is a very good screening test for any type of syphilis; JC virus detected in the cerebrospinal fluid would suggest PML, but her pretest probability for this is low because it usually affects patients with low CD4+ T-cell counts. Serum cryptococcal antigen has excellent performance characteristics, but there is little reason to suspect cryptococcal meningitis in the absence of headache or elevated intracerebral pressure.

IV-170. The answer is D. (Chap. 189) Indinavir is the only agent to cause nephrolithiasis. Nucleoside reverse transcriptase inhibitors, particularly stavudine and didanosine (d4T and ddI), are associated with mitochondrial toxicity and pancreatitis. Nevirapine can cause hepatic necrosis in women, particularly with CD4+ lymphocyte counts above 350/μL. Efavirenz, a very commonly used agent, causes dream disturbances that usually, but not always, subside after the first month of therapy. Both indinavir and atazanavir cause a benign indirect hyperbilirubinemia reminiscent of Gilbert’s syndrome.

IV-171. The answer is D. (Chap. 189) Isospora and Cryptosporidium spp. cause very similar clinical disease in AIDS patients that ranges from intermittent, self-resolved watery diarrhea with abdominal cramping and sometimes nausea to a potentially fatal cholera-like presentation in the most immunocompromised hosts. Cryptosporidium spp. may cause biliary disease and can lead to cholangitis. Isosporaspp. is limited to the gut lumen. Cryptosporidiumspp. is not always an opportunistic infection and has led to widespread community outbreaks. Isospora spp. is not seen in immunocompetent hosts. Finally, treatment for Isospora infection is usually successful. In fact, this infection is rarely seen in the developed world because trimethoprim–sulfamethoxazole, which is commonly used for Pneumocystisprophylaxis, tends to eradicate Isospora spp. Cryptosporidiosis, on the other hand, is very difficult to cure, and interventions are controversial. Some clinicians favor nitazoxanide, but cure rates are mediocre, and immune reconstitution with antiretroviral therapy is ultimately critical to cure the gastrointestinal disease.

IV-172. The answer is D. (Chap. 189) Acute HIV should be suspected in any at-risk person who presents with a mono-like illness; it is diagnosed by positive plasma RNA polymerase chain reaction (PCR). Patients typically have not developed sufficient antibodies to the virus yet to develop a positive enzyme immunoassay result, and the diagnosis of HIV is usually missed if this test is sent within the first 2 months of HIV acquisition. It is tempting for clinicians to send an ultrasensitive PCR, but this only decreases specificity (false-positive tests with detection of very low levels of HIV are possible because of cross-contamination in the laboratory) with no other benefit. There is typically a massive amount of HIV virus in the plasma during acute infection, and the ultrasensitive assay is never required for detection at this stage of disease. Ultrasensitive assays are helpful in the context of therapy to ensure that there is not persistence of low-level viremia. CD4+ lymphocyte count decreases during many acute infections, including HIV, and is therefore not diagnostically appropriate. CD4+ lymphocyte counts are useful to risk stratify for opportunistic infection in stable patients with known HIV infection. Resistance tests are sent only when the diagnosis is confirmed.

IV-173. The answer is B. (Chap. 189) Oral hairy leukoplakia is caused by a severe overgrowth of Epstein-Barr virus infection in T-cell–deficient patients. It is not premalignant and is often unrecognized by the patient but is sometimes a cosmetic, symptomatic, and therapeutic nuisance. The white, thickened folds on the side of the tongue can be pruritic or painful and sometimes resolve with acyclovir derivatives or topical podophyllin resin. Ultimate resolution occurs after immune reconstitution with antiretroviral therapy. Oral candidiasis or thrush is a very common, relatively easy-to-treat condition in HIV patients and takes on an appearance of white plaques on the tongue, palate, and buccal mucosa that bleed with blunt removal. Herpes simplex virus (HSV) recurrences or aphthous ulcers present as painful ulcerating lesions. The latter should be considered when oral ulcers persist, do not respond to acyclovir, and do not culture HSV. Kaposi’s sarcoma is uncommon in the oropharynx and takes on a violet hue, suggesting its highly vascularized content.

IV-174. The answer is E. (Chap. 189) Current recommendations are to initiate antiretroviral therapy in patients with the acute HIV syndrome, all pregnant women, patients with an AIDS-defining illness, patients with HIV-associated nephropathy, and patients with asymptomatic disease with CD4+ T-cell counts below 500/μL. Clinical trials are under way to determine the value of even earlier intervention, and some experts would place everyone with HIV infection on antiretroviral therapy. In addition, one may wish to administer a 6-week course of therapy to uninfected individuals immediately after a high-risk exposure to HIV. For patients diagnosed with an opportunistic infection and HIV infection at the same time, one may consider a 2- to 4-week delay in the initiation of antiretroviral therapy during which time treatment is focused on the opportunistic infection. Although not proven, it is postulated that this delay may decrease the severity of any subsequent immune reconstitution inflammatory syndrome by lowering the antigenic burden of the opportunistic infection.

TABLE IV-174 Indications for the Initiation of Antiretroviral Therapy in Patients With HIV Infection


 I. Acute infection syndrome

II. Chronic infection

A. Symptomatic disease (including HIV-associated nephropathy)

B. Asymptomatic disease

  1. CD4+ T-cell count <500/μLa

  2. Pregnancy

III. Postexposure prophylaxis


aThis is an area of controversy. Some experts would treat everyone regardless of CD4+ T-cell count.

Source: Guidelines for the Use of Antiretroviral Agents in HIV-Infected Adults and Adolescents, USPHS.

IV-175. The answer is B. (Chap. 189) Therapy of HIV infection does not lead to eradication or cure of HIV. Treatment decisions must take into account the fact that one is dealing with a chronic infection. Patients initiating antiretroviral therapy must be willing to commit to lifelong treatment and understand the importance of adherence to their prescribed regimen. Treatment interruption regimens are associated with rapid increases in HIV RNA levels, rapid declines in CD4+ T-cell counts, and an increased risk of clinical progression. In clinical trials, there has been an increase in serious adverse events in the patients randomized to intermittent therapy, suggesting that some “non–AIDS-associated” serious adverse events such as heart attack and stroke may be linked to HIV replication. Given that patients can be infected with viruses that harbor drug resistance mutations, it is recommended that a viral genotype be done before the initiation of therapy to optimize the selection of antiretroviral agents. The three options for initial therapy most commonly in use today are three different three-drug regimens. These include (1) tenofovir, emtricitabine, and efavirenz; (2) tenofovir, emtricitabine, and atazanavir (or darunavir); and (3) tenofovir, emtricitabine, and raltegravir. There are no clear data at present on which to base distinctions among these three regimens. After the initiation of therapy, one should expect a rapid, at least 1-log (10-fold) reduction in plasma HIV RNA levels within 1 to 2 months and then a slower decline in plasma HIV RNA levels to less than 50 copies per milliliter within 6 months. There should also be a rise in the CD4+ T-cell count of 100 to 150/μL that is also particularly brisk during the first month of therapy. Subsequently, one should anticipate a CD4+ T-cell count increase of 50 to 100 cells per year until numbers approach normal. Many clinicians believe that failure to achieve these endpoints is an indication for a change in therapy.

IV-176. The answer is B. (Chap. 190) The Norwalk virus is the prototype calicivirus that causes human disease. The calicivirus family, many of which cause gastroenteritis and diarrhea, particularly in children, includes norovirus and sapovirus. Most adults worldwide have antibodies to these viruses. However, they are a major cause of morbidity throughout the world and a frequent cause of nonbacterial diarrhea outbreaks in the United States. They spread via fecal–oral spread and have a low inoculum necessary for disease. In temperate regions, they tend to occur in cold weather months. The incubation period is less than 3 days, typically 24 hours. The onset of disease is rapid. Fever, myalgias, and headache are common. The diarrhea is nonbloody without fecal leukocytes. The disease is self-limited, and therapy is supportive.

IV-177. The answer is B. (Chap. 190) Nearly all children worldwide are infected with rotavirus by age 5 years. In the developing world, it remains a major cause of diarrheal death caused by volume depletion. Repeated infections occur with each subsequent episode of lesser severity. Therefore, severe disease is uncommon in adolescents and adults who may develop disease, particularly after contact with ill children. The disease typically has an abrupt onset with vomiting usually preceding diarrhea. Fever occurs in approximately one-third of cases. Stools usually do not contain blood, mucus, or inflammatory material. The disease is usually self-limited in 3 to 7 days. Because rotavirus is a major cause of childhood hospitalization and morbidity in the United States, vaccination is recommended for all U.S. children. Vaccination has less efficacy in the developing world because of a higher frequency of malnutrition, co-infection, and comorbidities, but is recommended by the World Health Organization for all children worldwide.

IV-178. The answer is B. (Chap. 191) Enteroviruses are responsible for up to 90% of aseptic meningitis in which an etiologic agent can be identified. Symptoms are typically more severe in adults than children. Illness is more frequent in the summer and fall in temperate climates, but other causes of viral meningitis are more common in winter and spring. Cerebro-spinal fluid (CSF) analysis always shows an elevated (although usually <1000 cells/μL) white blood cell count. Early, there may be a neutrophil predominance; however, this typically shifts toward lymphocyte predominance by 24 hours. CSF glucose and protein are usually normal, although the latter can sometimes be elevated. The illness is typically self-limiting, and the prognosis is excellent.

IV-179. The answer is B. (Chap. 191) These lesions are diagnostic of herpangina, which is caused by coxsackievirus A. They are typically round and discrete, which helps differentiate them from thrush caused by Candida spp. Unlike herpes simplex virus stomatitis, herpangina lesions are not associated with gingivitis. Lesions are typically concentrated in the posterior portion of the mouth. Herpangina usually presents with dysphagia, odynophagia, and fever; these lesions can persist for several weeks. The lesions do not ulcerate.

IV-180. The answer is B. (Chap. 191) Enteroviruses are single-strand RNA viruses that multiply in the gastrointestinal (GI) tract but rarely cause GI illness. Typical person-to-person spread occurs via the fecal–oral route; enteroviruses are not known to spread via blood transfusions or insect vectors. Infection is most common among infants and small children; serious illness occurs in neonates, older children, and adults. Most infections with poliovirus are symptomatic or cause a minor illness. Before the implementation of polio vaccines, paralysis was a rare clinical presentation of poliovirus infection and was less frequent in developing countries, likely because of earlier exposure. Paralytic disease caused by polio infection is more common in older adults, pregnant women, and persons exercising strenuously or with trauma at the time of central nervous system symptoms. Exposure to maternal antibodies leads to a lower risk of symptomatic neonatal infection.

IV-181. The answer is D. (Chap. 195) The patient has been bitten by a member of a species known to carry rabies in an area in which rabies is endemic. Based on the animal vector and the facts that the skin was broken and that saliva possibly containing the rabies virus was present, postexposure rabies prophylaxis should be administered. If an animal involved in an unprovoked bite can be captured, it should be killed humanely, and the head should be sent immediately to an appropriate laboratory for rabies examination by the technique of fluorescent antibody staining for viral antigen. If a healthy dog or cat bites a person in an endemic area, the animal should be captured, confined, and observed for 10 days. If the animal remains healthy for this period, the bite is highly unlikely to have transmitted rabies. Postexposure prophylactic therapy includes vigorous cleaning of the wound with a 20% soap solution to remove any virus particles that may be present. Tetanus toxoid and antibiotics should also be administered. Passive immunization with anti-rabies antiserum in the form of human rabies immune globulin (rather than the corresponding equine antiserum because of the risk of serum sickness) is indicated at a dose of 10 units/kg into the wound and 10 units/kg IM into the gluteal region. Second, one should actively immunize with an antirabies vaccine (either human diploid cell vaccine or rabies vaccine absorbed) in five 1-mL doses given intramuscularly, preferably in the deltoid or anterior lateral thigh area. The five doses are given over a 28-day period. The administration of either passive or active immunization without the other modality results in a higher failure rate than does the combination therapy.

IV-182. The answer is B. (Chap. 196) This patient has a typical presentation of dengue fever. All four distinct dengue viruses (dengue 1–4) have the mosquito Aedes aegypti as their principal vector, and all cause a similar clinical syndrome. Thus, lifelong immunity cannot be presumed. In rare cases, second infection with a serotype of dengue virus different from that involved in the primary infection leads to dengue hemorrhagic fever with severe shock. Year-round transmission between latitudes 25°N and 25°S has been established, and seasonal forays of the viruses to points as far north as Philadelphia are thought to have taken place in the United States. Dengue fever is seen throughout Southeast Asia, including Malaysia, Thailand, Vietnam, and Singapore. the Western hemisphere, it may be found in the Caribbean region, including Puerto Rico. With increasing spread of the vector mosquito throughout the tropics and subtropics, large areas of the world have become vulnerable to the introduction of dengue viruses, particularly through air travel by infected humans, and both dengue fever and the related dengue hemorrhagic fever are becoming increasingly common. The A. aegypti mosquito, which is also an efficient vector of the yellow fever and chikungunya viruses, typically breeds near human habitation, using relatively fresh water from sources such as water jars, vases, discarded containers, coconut husks, and old tires. A. aegypti usually inhabits dwellings and bites during the day. After an incubation period of 2 to 7 days, the typical patient experiences the symptoms described above along with the severe myalgia that gave rise to the colloquial designation “break-bone fever.” There is often a macular rash on the first day as well as adenopathy, palatal vesicles, and scleral injection. The illness may last 1 week, with additional symptoms usually including anorexia, nausea or vomiting, marked cutaneous hypersensitivity, and—near the time of defervescence—a maculopapular rash beginning on the trunk and spreading to the extremities and the face. Laboratory findings include leukopenia; thrombocytopenia; and, in many cases, serum aminotransferase elevations. The diagnosis is made by immunoglobulin G enzyme-linked immunosorbent assay (ELISA) or paired serology during recovery or by antigen-detection ELISA or reverse transcription polymerase chain reaction during the acute phase. In endemic regions where specific testing is not easily available, the diagnosis is presumed in cases of a typical clinical presentation and thrombocytopenia. Given the frequency of disease and the potential for hemorrhagic fever, active investigation is pursuing an effective vaccine.

IV-183. The answer is D. (Chap. 198) The classification of fungal infections is typically based on anatomic location of infection and epidemiology of the organism. Additionally, it is important to know the descriptive characteristics of fungi in culture because that information may be useful clinically. Endemic mycoses (e.g., coccidioidomycosis, blastomycosis) are infections that are caused by fungi that are not typically part of human microbial flora. Opportunistic mycoses (e.g., CandidaAspergillus) are infections that are caused by fungi that are commonly part of the human microbial flora. Yeasts (CandidaCryptococcus) are seen microscopically as single cells or rounded organisms. Molds (AspergillusRhizopus) grow as filamentous forms (hyphae) at room temperature and in tissue. Dimorphic is the term to describe fungi that exist as yeasts or large spherical structures in tissue but in filamentous forms in the environment. Blastomycosis, histoplasmosis, paracoccidioidomycosis, coccidioidomycosis, and sporotrichosis are typically dimorphic. Candida may exist as a yeast or filamentous form in tissue infection, with the exception of Candida glabrata, which only exists as a yeast in tissue infection.

IV-184. The answer is A. (Chap. 198) The azole antifungals (fluconazole, itraconazole, voriconazole, and posaconazole) all have oral formulations. The azoles inhibit ergosterol synthesis in the fungal cell wall. Compared with amphotericin B, they are considered fungistatic but do not cause significant renal toxicity. Fluconazole is effective for Candida albicans and coccidioidomycosis. Voriconazole has broader spectrum against Candida spp., including Candida glabrata and Candida krusei. Itraconazole is the drug of choice for mild to moderate histoplasmosis and blastomycosis. Posaconazole is approved in immunocompromised patients as prophylaxis against Candida and Aspergillus infection. Studies of posaconazole have suggested efficacy for zygomycosis, Aspergillus, and cryptococcal infections. It also has activity against fluconazole-resistant Candida. The echinocandins (caspofungin, anidulafungin, micafungin) only have intravenous formulations at this time. They are considered fungicidal for Candida and fungistatic for Aspergillus. Griseofulvin is an oral medication used historically primarily for ringworm infection. Terbinafine is more effective than griseofulvin for ringworm and onychomycosis.

IV-185. The answer is D. (Chap. 198) All patients with Candida fungemia should be treated with systemic antifungals. Fluconazole has been shown to be an effective agent for candidemia with equivalence to amphotericin products and caspofungin. Voriconazole is also active against Candida albicans but has many drug interactions that make it less desirable against this pathogen. However, it has broader activity against Candida spp. including Candida glabrata and Candida krusei. No trials of posaconazole for candidemia have yet been reported. The echinocandins, including micafungin and caspofungin, have broad activity, are fungicidal against Candida spp., and have low toxicity. They are among the safest antifungal agents.

IV-186. The answer is B. (Chap. 198) The definitive diagnosis of an invasive fungal infection generally requires histologic demonstration of fungus invading tissue along with an inflammatory response. However, Coccidioidesserum complement fixation, cryptococcal serum and cerebrospinal fluid antigen, and urine/serum Histoplasma antigen are all tests with good performance characteristics, occasionally allowing for presumptive diagnoses before pathologic tissue sections can be examined or cultures of blood or tissue turn positive. Serum testing for galactomannan is approved for the diagnosis of Aspergillusinfection. However, false-negative test results may occur. Multiple serial tests may decrease the incidence of false-negative test results. There is no approved urine or serologic test for blastomycosis.

IV-187. The answer is D. (Chap. 199) All of these pathogens are typically inhaled and cause pulmonary infection, which may resolve spontaneously or progress to active disease. Resolved infection with blastomycosis, coccidioidomycosis, Cryptococcus spp., and tuberculosis will often leave a radiographic lesion that typically looks like a solitary nodule and may be confused with potential malignancy. Latent tuberculosis is often suggested by the radiographic finding of a calcified lymph node that is typically solitary. Of the listed infections, histoplasmosis is most likely to resolve spontaneously in an immunocompetent individual, leaving multiple mediastinal and splenic calcifications. These represent calcified granulomas formed after an appropriate cellular immunity response involving interleukin-12, tumor necrosis factor-α in combination with functional lymphocytes, macrophages, and epithelial cells. In endemic areas, 50% to 80% of adults have evidence of previous infection without clinical manifestations. In patients with impaired cellular immunity, the infection may disseminate to the bone marrow, spleen, liver, adrenal glands, and mucocutaneous membranes. Unlike tuberculosis, remote Histoplasma infection rarely reactivates.

IV-188. The answer is C. (Chap. 199) This silver stain shows the typical small (2–5 μm) budding yeast of Histoplasma capsulatum on bronchoalveolar lavage (BAL). Patients receiving infliximab and other anti–tumor necrosis factor therapies are at risk of developing opportunistic infection with tuberculosis, Histoplasma, other pathogenic fungi (including pneumocystis), Legionella, and viruses (including cytomegalovirus). These infections typically manifest after approximately 2 months of therapy, although shorter and longer durations are described. Patients with AIDS (CD4 <200 cells/mm3), at extremes of age, and receiving prednisone therapy are also at risk of disseminated histoplasmosis. Disseminated histoplasmosis may present with shock, respiratory failure, pancytopenia, disseminated intravascular coagulation, and multiorgan failure or as a more indolent illness with focal organ dissemination, fever, and systemic symptoms. Culture results of BAL are positive in more than 50% of cases of acute respiratory histoplasmosis. Bone marrow and blood cultures have a high yield in disseminated cases. Histoplasma antigen testing of blood and BAL is also sensitive and specific. There is potential cross-reactivity with blastomycosis, coccidioidomycosis, and paracoccidioidomycosis.

IV-189. The answer is E. (Chap. 199) Patients with severe life-threatening Histoplasma infection should be treated with a lipid formulation of amphotericin B followed by itraconazole (see Table IV-189). In immunosuppressed patients, the degree of immunosuppression should be reduced if possible. Caspofungin (and other echinocandins) is not active against Histoplasma spp. but would be used for infection with Candida or Aspergillus. Ganciclovir is recommended for cytomegalovirus infection. The combination isoniazid, rifampin, pyrazinamide, and ethambutol is recommended therapy for Mycobacterium tuberculosis. Clarithromycin, rifampin, and ethambutol therapy is recommended therapy for Mycobacterium avium complex.

TABLE IV-189 Recommendations for the Treatment of Histoplasmosis

image

IV-190. The answer is C. (Chap. 200) This patient likely has coccidioidomycosis with meningitis caused by HIV infection as evidenced by his history, physical examination, laboratory findings, and diagnostic microbes in the cerebrospinal fluid (CSF). Fresno is in the heart of the San Joaquin Valley, the highest endemic region for coccidioidomycosis. For reasons that are unclear, African American men and Filipino males are at highest risk of developing coccidioidal infection. Coccidioidomycosis becomes disseminated in fewer than 1% of cases, but the meninges as well as the skin, bone, and joints are the most common extrapulmo-nary sites. Defective cellular immunity and immunosuppression increase the likelihood of dissemination and meningitis. Nuchal rigidity is mild when present, but chronic headache and confusion are typical. Untreated infection may lead to hydrocephalus. Untreated meningitis is uniformly fatal. The CSF findings in this case are typical with lymphocyte predominance, markedly low glucose, and high protein. The findings on silver stain are characteristic of spherules with are unique to coccidioidal infection and are diagnostic when found in tissue (often in granulomas) or body fluids. Complement fixation antibodies in the CSF also indicate infection. Traditionally, amphotericin B was used for treatment of meningitis, but azole antifungals may now be used. Studies have shown itraconazole and fluconazole effective for coccidioidal infections. Fluconazole has excellent CSF penetration and is currently the recommended therapy. Itraconazole is likely preferred for bone and joint disease. Therapy should be lifelong because the relapse rate is greater than 80%. Although the clinical presentation and CSF findings are consistent with tuberculosis meningitis, the finding of spherules is diagnostic. Caspofungin is active against Candida and Aspergillus but not coccidioidomycosis. Penicillin G is the preferred therapy for tertiary syphilis.

IV-191. The answer is C. (Chap. 200) Coccidioides immitis is a mold that is found in the soil in the southwestern United States and Mexico. Case clusters of primary disease may appear 10 to 14 days after exposure, and the activities with the highest risk include archaeological excavation, rock hunting, military maneuvers, and construction work. Only 40% of primary pulmonary infections are symptomatic. Symptoms may include those of a hypersensitivity reaction such as erythema nodosum (typically on the lower extremities), erythema multiforme (typically in a necklace distribution), arthritis, or conjunctivitis. Blood eosinophilia is common during acute infection. Although pleurisy is common, significant pleural effusion only occurs in 10% of cases (typically mononuclear with negative culture). Diagnosis can be made by culture of sputum; however, when this organism is suspected, the laboratory needs to be notified because it is a biohazard level 3 fungus. Serologic tests of blood may also be helpful; however, seroconversion of primary disease may take up to 8 weeks. Skin testing is useful only for epidemiologic studies and is not done in clinical practice. Asymptomatic and most cases of focal uncomplicated pneumonia do not require therapy.

IV-192. The answer is E. (Chap. 200) Northern Arizona (i.e., the Grand Canyon region) is not a region of high incidence of coccidioidomycosis. The organism can be cultured from dry top soil in the high desert of Southern Arizona surrounding Phoenix and Tucson. In North America, the areas of greatest endemicity include the San Joaquin valley in California, south central Arizona, and northern Mexico. Endemic foci have also been described in the Texas Rio Grande Valley, some areas of Central America, Columbia, Venezuela, northeastern Brazil, Paraguay, Bolivia, and north central Argentina. Eosinophilia is a common laboratory finding in acute coccidioidomycosis, and erythema nodosum is a common cutaneous clinical feature (particularly on the lower extremities in women). Mediastinal lymphadenopathy is more commonly seen on radiographs for all acute pneumonias caused by endemic mycoses, including Coccidioides spp., rather than caused by bacterial pneumonia. A positive complement fixation test result is one method to definitively diagnose acute infection.

IV-193. The answer is C. (Chap. 201) Blastomycosis is caused by the dimorphic fungus Blastomyces dermatides, which commonly resides in soil and is acquired through inhalation. Pulmonary infection is most common and can be acute or indolent. Extrapulmonary extension via hematogenous spread from the lungs is common with skin lesions and osteomyelitis most common. In patients with AIDS, central nervous system involvement, usually as a brain abscess, has been reported in approximately 40% of cases of blastomycosis. Most cases of blastomycosis are reported from North America with the most common regions being bordering the Mississippi and Ohio River basins, the upper Midwest and Canada bordering the Great Lakes, and a small area of New York and Ontario bordering the St. Lawrence River. Outside of North American, most blastomycosis cases are in Africa. Coccidioidomycosis is endemic in Southern Arizona.

IV-194. The answer is C. (Chap. 201) The constellation of symptoms including chronic pneumonia with ulcerating skin lesions and soil exposure in the upper Midwest in the Great Lakes region is highly suggestive of disseminated blastomycosis infection. Sputum or skin biopsy may show broad-based budding yeast. The definitive diagnosis would be made by growth of the organism from sputum or skin biopsy. Serologic testing is of limited use because of cross-reactivity with other endemic fungi. There is a urine Blastomyces antigen test that appears more sensitive than serum testing. Therapy for blastomycosis in a non–life-threatening condition is with itraconazole. Lipid formulations of amphotericin are indicated in life-threatening disease or central nervous system (CNS) disease (fluconazole can also be used for CNS disease). Blastomycosis may present with solitary pulmonary lesions that may be suggestive of malignancy and should be evaluated as such. The chronic indolent form may also be confused with pulmonary tuberculosis. The differential diagnosis of blastomycosis skin lesions includes pyoderma gangrenosum that may be associated with inflammatory bowel disease. Methicillin-resistant Staphylococcus aureus skin lesions may be nodular then ulcerate but, when associated with hematologic dissemination from the lung, are usually more acute than this indolent presentation.

IV-195. The answer is B. (Chap. 202) The goal of therapy for cryptococcal meningoencephalitis in an HIV-negative patient is cure of the fungal infection, not simply control of symptoms. Thus, intravenous amphotericin plus flucytosine is recommended as induction followed by prolonged therapy with fluconazole. Amphotericin plus flucytosine for 6 to 10 weeks may also be used in non-immunocompromised patients. Patients with immunocompromising conditions may receive the same initial regimen with a more prolonged course of fluconazole to prevent relapse. Although not relevant in this case, isolated pulmonary cryptococcosis in an immunocompetent patient may be treated with fluconazole for 3 to 6 months after clear demonstration that the cerebrospinal fluid (CSF) is not affected. Cryptococcal meningitis often causes increased intracranial pressure (ICP), which is thought to contribute to irreversible brain and cranial nerve damage. ICP should be measured during lumbar puncture, and CSF should be removed as needed to avoid elevated ICP. Shunt placement may be indicated in refractory cases. Newer triazoles, such as voriconazole, are highly active against Cryptococcus, but clinical experience is limited at this time. Neither caspofungin nor micafungin have activity against Cryptococcus. Ceftriaxone and vancomycin are the recommended treatments for bacterial meningitis in an immunocompetent patient younger than 50 years of age and have no role in the therapy of Cryptococcus infection.

IV-196. The answer is A. (Chap. 202) Cryptococcal meningoencephalitis presents with early manifestations of headache, nausea, gait disturbance, confusion, and visual changes. Fever and nuchal rigidity are often mild or absent. Papilledema is present in more than 30% of cases. Asymmetric cranial nerve palsies occur in 25% of cases. Neuroimaging findings are often normal. If there are focal neurologic findings, magnetic resonance imaging may be used to diagnose cryptococcomas in the basal ganglia or caudate nucleus, although they are more common in immunocompetent patients with Cryptococcus neoformansvar. gattii. Imaging does not make the diagnosis. The definitive diagnosis remains cerebrospinal fluid (CSF) culture. However, capsular antigen testing in both the serum and the CSF is very sensitive and can provide a presumptive diagnosis. Approximately 90% of patients, including all with a positive CSF smear, and the majority of AIDS patients have detectable cryptococcal antigen. The result is often negative in patients with isolated pulmonary disease. However, because of a very small false-positive rate in antigen testing, CSF culture remains the definitive diagnostic test. In this condition C. neoformans often can also be cultured from the urine; however, other testing methods are more rapid and useful.

IV-197. The answer is D. (Chap. 203) Reviews of cases reveal consistent conditions and risk factors associated with hematogenous dissemination of Candida. Many refer to the fact that innate immunity is the most important defense mechanism against hematogenous dissemination of the fungus and that neutrophils are the most important component of this defense. Many immunocompetent people have antibodies to Candida spp.; the role of these antibodies in the defense against hematogenous spread is not clear. Therefore, patients the conditions and risk factors listed in the question plus indwelling urinary catheters, parenteral glucocorticoids, neutropenia, cytotoxic chemotherapy, and immunosuppressive agents for organ transplantation all confer risk of disseminated candidiasis. Additionally, low birth weight infants, HIV-infected patients with low CD4 counts, and patients with diabetes are at great risk of local infection with Candida that may disseminate when other predisposing factors are present (e.g., catheters). Women receiving antibiotics are at risk of developing vaginal candidiasis. Patients with pulmonary alveolar proteinosis are at risk of infection with unusual organisms such as Nocardia, atypical mycobacteria, Aspergillus, and pneumocystis but are not at increased risk of disseminated candidiasis in the absence of other risk factors.

IV-198. The answer is D. (Chaps. 198 and 203) This patient presents with the classic skin presentation of disseminated candidiasis. The skin lesions, severe myalgias, joint pains, and fever are typical manifestations of hematogenous spread from either a gastrointestinal or skin source in a patient predisposed by neutropenia and indwelling catheters. The severe myalgias are a characteristic of this syndrome and should be taken seriously as a new complaint in a susceptible host. Blood culture results are likely be positive, but staining of the skin lesions is positive in virtually 100% of cases. Candida is the only fungus that can typically be visualized on tissue Gram stain in the form of pseudohyphae and hyphae. Aspergillus is seen in tissue as clumps of branching (45 degrees) septated hyphae often with angioinvasion and necrosis. Aspergillus may also disseminate in a prolonged neutropenic patient, usually from a lung infection, and cause rapidly progressive skin lesions, usually with a necrotic center. Histoplasma and Blastomyces can be visualized in tissue as budding yeast. Encapsulated yeasts on India ink are indicative of Cryptococcus. Spherules are specific to coccidioidomycosis.

IV-199. The answer is D. (Chap. 203) Candida spp. are susceptible to a number of systemic antifungal agents. Most institutions chose an agent based on their local epidemiology and resistance patterns. Fluconazole is the most commonly used agent for nonneutropenic hemodynamically stable patients unless azole resistance is considered an issue. In a hemodynamically unstable neutropenic patient, more broad-spectrum agents are typically used such as polyenes, echinocandins, or later-generation azoles such as voriconazole. (See Table IV-199.) Lipid formulations of amphotericin, although not approved by the U.S. Food and Drug Administration as primary therapy, are commonly used because they are less toxic than amphotericin B deoxycholate. At present, the vast majority of isolates of Candida albicans are sensitive to fluconazole. Candida glabrata and Candida krusei are more sensitive to polyenes and echinocandins. Flucytosine is not used as sole therapy for Candida. It may be combined with amphotericin for treatment of Candida endophthalmitis and meningitis.

TABLE IV-199 Agents for the Treatment of Disseminated Candidiasis

image

image

IV-200. The answer is B. (Chap. 203) The use of antifungal agents to prevent Candida infections remains controversial, but some general principles have emerged in recent years. Most centers start prophylactic fluconazole to allogeneic stem cell transplant recipients. Many centers also administer them to high-risk liver transplant recipients but not routine living related renal transplant recipients. This prophylaxis should be differentiated from the administration of empiric broad-spectrum antifungal therapy in a patient with prolonged febrile neutropenia. Voriconazole is an appropriate choice for empiric broad-spectrum therapy in an unstable patient with suspected candidemia, but it has not been shown to be superior to any other agent for prophylaxis against Candida in any population. Complicated postoperative surgical patients are at risk of Candida infection, and some centers administer prophylaxis to very high-risk patients. However, the widespread use of Candida prophylaxis in surgical patients is not recommended because the incidence of disseminated candidiasis is low, the cost-benefit ratio is suboptimal, and there is reasonable rationale to believe that this strategy could increase Candida resistance to current medications. Candida prophylaxis for HIV-infected patients is recommended to prevent frequent recurrent oropharyngeal or esophageal infection.

IV-201. The answer is C. (Chap. 203) Isolation of yeast from the bloodstream can virtually never be considered a contaminant. Presentation may be indolent with malaise only or fulminant with overwhelming sepsis in the neutropenic host. All indwelling catheters need to be removed to ensure clearance of infection, and evaluation for endocarditis and endophthalmitis should be strongly considered, particularly in patients with persistently positive cultures or fever. Both of these complications of fungemia often entail surgical intervention for cure. A positive yeast culture in the urine is often difficult to interpret, particularly in patients taking antibiotics and in the intensive care unit. Most frequently, a positive culture result for yeast represents contamination even if the urinalysis suggests bladder inflammation. An attractive option is to remove the Foley catheter and recheck a culture. Antifungals are indicated if the patient appears ill, in the context of renal transplant in which fungal balls can develop in the graft, and often in neutropenic patients. Candida pneumonia is uncommon even in immunocompromised patients. A positive yeast culture of the sputum is usually representative of commensal oral flora and should not be managed as an infection, particularly as in this patient in whom acute bacterial pneumonia is likely.

IV-202. The answer is C. (Chap. 203) Candidemia may lead to seeding of other organs. Among nonneutropenic patients, up to 10% develop retinal lesions; therefore, it is very important to perform thorough funduscopy. Focal seeding can occur within 2 weeks of the onset of candidemia and may occur even if the patient is afebrile or the infection clears. The lesions may be unilateral or bilateral and are typically small white retinal exudates. However, retinal infection may progress to retinal detachment, vitreous abscess, or extension into the anterior chamber of the eye. Patients may be asymptomatic initially but may also report blurring, ocular pain, or scotoma. Abdominal abscess are possible but usually occur in patients recovering from profound neutropenia. Fungal endocarditis is also possible but is more common in patients who use intravenous drugs and may have a murmur on cardiac examination. Fungal pneumonia and pulmonary abscesses are very rare and are not likely in this patient.

IV-203. The answer is A. (Chap. 204) Aspergillus has a worldwide distribution, typically growing in decomposing plant materials. Immunocompetent individuals generally do not develop disease without intense exposure such as during construction or handling of moldy hay, bark, or compost. Nosocomial outbreaks are usually directly related to contaminated air source in the hospital. HEPA filtration is effective in eliminating infection from operating rooms and units with high-risk patients. Contaminated water sources are the typical reservoir of nosocomial Legionella outbreaks. Patient-to-patient spread in waiting rooms has been described for cystic fibrosis patients transmitting Burkholderia infection. Provider-to-patient transmission of methicillin-resistant Staphylococcus aureus and most other bacteria is reduced with effective use of alcohol-based disinfectant; however, in the case of Clostridium difficile, alcohol will not eliminate spores, and effective handwashing with soap and water is necessary.

IV-204. The answer is B. (Chap. 204) Diagnosis of invasive Aspergillus infection is often difficult because early therapy is essential, and approximately 40% of cases are missed clinically and are diagnosed at autopsy. Sputum culture is positive in only 10% to 30% of patients; the yield is higher when fungal media rather than bacterial agar is used. Thus, specifically requesting fungal culture is necessary. The Aspergillus antigen assay relies on galactomannan release during fungal growth. Antigen testing results are positive days before clinical or radiologic abnormalities appear. The test may be falsely positive in patients receiving β-lactam/β-lactamase inhibitor antibiotics. The sensitivity in patients with prolonged neutropenia is likely about 80%. Prior therapeutic or empiric use of antifungal therapy lowers the sensitivity of the serum test. The test can be performed on bronchoalveolar lavage samples. The computed tomography findings in this case are also typical of the “halo sign” often seen in cases of invasive pulmonary aspergillosis. The halo of ground glass infiltrate surrounding an Aspergillus nodule represents hemorrhagic infarction. Other fungi may cause the halo sign, but Aspergillus, because of the tendency to be angioinvasive, is the most common. The other diagnoses in this case are much less likely given the clinical history and the radiologic signs.

IV-205. The answer is E. (Chap. 204) Intravenous voriconazole is currently the preferred therapy for invasive aspergillosis. Caspofungin, posaconazole, and lipid-based formulations of amphotericin are second-line agents. Amphotericin is not active against Aspergillus terreus or Aspergillus nidulans. Fluconazole is active against Candida spp. but not Aspergillus spp. Trimethoprim–sulfamethoxazole is used for therapy against Pneumocystis jiroveci.

IV-206. The answer is A. (Chap. 204) Aspergillus infection has many clinical manifestations. Invasive aspergillosis typically occurs in immunocompromised patients and presents as rapidly progressive pulmonary infiltrates. Infection progresses by direct extension across tissue planes. Cavitation may occur. Allergic bronchopulmonary aspergillosis (ABPA) is a different clinical entity. It often occurs in patients with preexisting asthma or cystic fibrosis. It is characterized by an allergic reaction to Aspergillus spp. Clinically, it is characterized by intermittent wheezing, bilateral pulmonary infiltrates, brownish sputum, and peripheral eosinophilia. Immunoglobulin E may be elevated, suggesting an allergic process, and a specific reaction to Aspergillus spp. that is manifested by serum antibodies or skin testing is common. Although central bronchiectasis and fleeting infiltrates caused by mucus plugging are common radiographic findings in ABPA, the presence of peripheral cavitary lung lesions is not a common feature.

IV-207. The answer is E. (Chap. 204) Allergic bronchopulmonary aspergillosis (ABPA) is not a true infection but rather a hypersensitivity immune response to colonizing Aspergillus spp. It occurs in about 1% of patients with asthma and in up to 15% of patients with cystic fibrosis. Patients typically have wheezing that is difficult to control with usual agents, infiltrates on chest radiographs caused by mucus plugging of airways, a productive cough often with mucus casts, and bronchiectasis. Eosinophilia is common if glucocorticoids have not been administered. The total immunoglobulin E (IgE) is of value if greater than 1000 IU/mL in that it represents a significant allergic response and is very suggestive of ABPA. In the proper clinical context, a positive skin test result for Aspergillus antigen or detection of serum Aspergillus-specific IgG or IgE precipitating antibodies are supportive of the diagnosis. Galactomannan enzyme immunoassay is useful for invasive aspergillosis but has not been validated for ABPA. There is no need to try to culture an organism via bronchoalveolar lavage to make the diagnosis of ABPA. Chest computed tomography, which may reveal bronchiectasis, or pulmonary function testing, which will reveal an obstructive defect, will not be diagnostic.

IV-208. The answer is B. (Chap. 204) The primary risk factor for developing invasive Aspergillus infection is neutropenia and glucocorticoid use (Figure IV-208). Risk is proportional to the degree and length of neutropenia and the dose of glucocorticoid. Stable HIV patients rarely develop invasive aspergillosis. Patients with AIDS are at some risk, typically in the context of prolonged neutropenia or advanced disease. Patients with graft-versus-host disease and uncontrolled leukemia are at particularly elevated risk. The infection is seen in solid organ transplant patients, particularly those requiring high cumulative doses of glucocorticoids for graft rejection. Recent reports describe an increasing incidence of invasive Aspergillus infection in medical intensive care units, particularly in patients with preexisiting lung disease such as pneumonia or chronic obstructive pulmonary disease. Glucocorticoid use does not appear to increase the risk of invasive sinus disease, only lung infection. Anti–tumor necrosis factor therapy also increases the risk of invasive Aspergillus infection.

image

FIGURE IV-208

IV-209. The answer is B. (Chap. 205) Mucormycosis refers to life-threatening infection caused by the Mucorales (formerly known as Zygomycetes) family of fungi. The most common fungus accounting for these infections is Rhizopus oryzae. The mortality rate of these infections approaches 50%. The Mucorales are environmentally ubiquitous; infection requires a defect in the patient’s ability to killing or phagocytic function. The most common predisposing factors are diabetes, glucocorticoid therapy, neutropenia, and iron overload. Free iron supports fungal growth in serum and tissues, enhancing survival and virulence. Deferoxamine therapy predisposes to fatal infection because the chelator acts as a siderophore, directly delivering iron to the fungi. Acidosis also causes dissociation of iron from serum proteins, promoting growth of Mucorales. Patients with diabetic ketoacidosis are at particularly high risk of developing rhinocerebral mucormycosis likely because of the combination of acidosis and phagocytic defects associated with hyperglycemia. Hypoglycemia is not an identified risk factor for mucormycosis.

IV-210. The answer is E. (Chap. 205) This patient has evidence of invasive rhinocerebral mucormycosis with risk factors including acute and chronic hyperglycemia and metabolic acidosis caused by chronic renal insufficiency. With a greater than 50% mortality rate, therapy of rhinocerebral mucormycosis requires early diagnosis, reversal of underlying predisposing conditions, surgical debridement, and immediate antifungal therapy. Insulin and hemodialysis should be initiated to correct hyperglycemia and metabolic acidosis. Amphotericin products remain the treatment of choice for mucormycosis. Liposomal amphotericin has improved central nervous system penetration compared with the lipid complex formations. Surgical debridement is also an important component of early therapy. If untreated, the infection quickly spreads from the ethmoid sinus to the orbit and into the cavernous sinus. Development of contralateral signs suggests cavernous sinus thrombosis and portends a very poor prognosis. Differentiation of mucormycosis from Aspergillus is important because they tend to infect similar hosts and are rapidly fatal. In contrast to mucormycosis species, the hyphae of Aspergillus spp. are septated, are thinner, and branch at acute angles. Voriconazole, the initial therapy for Aspergillus infection, is not indicated in mucormycosis and in fact has been shown to exacerbate mucormycosis in animal models. Echinocandin antifungal agents have activity against Mucorales, and animal data suggest that they may have a role in combination with lipid polyene agents.

IV-211. The answer is E. (Chap. 205) The sites of infection due to Mucorales fungal infection tend to affect patients with specific host defense defects. The most common clinical manifestation of mucormycosis is rhinocerebral. Most cases occur in patients with diabetes or hyperglycemia caused by glucocorticoid therapy (e.g., solid organ transplantation). The initial symptoms usually include facial or orbital pain or numbness, facial suffusion, and soft tissue swelling. The infection usually originates in the ethmoid sinus region and spreads rapidly to the orbit and central nervous system. Painful necrotic lesions may be seen in the mouth. Pulmonary mucormycosis is the second most common manifestation of Mucorales infection. Human stem cell transplantation is a common risk factor for pulmonary mucormycosis. The risk factors and presentation are similar to that of invasive pulmonary Aspergillus infection. Differentiation is important because antifungal therapy differs. The two diseases appear similar on chest computed tomography, although the presence of more than 10 nodules, pleural effusion, or concomitant sinusitis makes mucormycosis more likely. Other sites of involvement with mucormycosis are described but less common. Cutaneous disease may result from external implantation (soil-related trauma or plant penetration) or hematogenous dissemination. Implanted cutaneous disease is also highly invasive; the development of fasciitis has a greater than 70% mortality rate. Rapid surgical debridement is essential. Hematogenous dissemination has a very high mortality rate; involvement of the brain has a near 100% mortality rate. Gastrointestinal mucormycosis is most common in neonates with necrotizing enterocolitis.

IV-212. The answer is E. (Chap. 206) This patient has tinea capitis most likely caused by the dermatophytic mold, Trichophyton spp. The other dermatophytes that less frequently cause cutaneous infection include Microsporum and Epidermophyton spp. They are not part of the normal skin flora but can live in keratinized skin structures. Infections with these organisms are extremely common and are often called ringworm, although the causative organisms are fungi, not worms. They manifest as infection of the head (tinea capitis), feet (tinea pedis), crotch (tinea cruris), and nails (tinea unguium or onychomycosis). Tinea capitis is most common in children ages 3 to 7 years but also occurs in adults. Usually, the typical appearance, as in this case, is diagnostic. Scrapings may be taken from the edge of lesion and stained with KOH to reveal hyphae. Dermatophyte infections often respond to topical therapy. For troublesome infections, itraconazole or terbinafine for 1 to 2 weeks can hasten resolution. Terbinafine is often preferred because of fewer drug interactions.

IV-213. The answer is D. (Chap. 206) Sporothrix schenckii is a thermally dimorphic fungus found in soil, plants, and moss and occurs most commonly in gardeners, farmers, florists, and forestry workers. Sporotrichosis develops after inoculation of the organism into the skin with a contaminated puncture or scratch. The disease typically presents as a fixed cutaneous lesion or with lymphocutaneous spread. The initial lesion typically ulcerates and become verrucous in appearance. The draining lymphatic channels become affected in up to 80% of cases. This presents as painless nodules along the lymphatic channel, which ulcerate. A definitive diagnosis is made by culturing the organism. A biopsy of the lesion may show ovoid or cigar-shaped yeast forms. Treatment for sporotrichosis is systemic therapy. Options include oral itraconazole, saturated solution of potassium iodide, and terbinafine. However, terbinafine has not been approved for this indication in the United States. Topical antifungals are not effective. In cases of serious system disease such as pulmonary sporotrichosis, amphotericin B is the treatment of choice. Caspofungin is not effective against S. schenckii.

IV-214. The answer is C. (Chap. 207) Patients receiving biologic agents, including the tumor necrosis factor antagonists infliximab and etanercept, are at increased risk of multiple infections, including pneumocystis. Pneumocystis is thought to be a worldwide organism with most people exposed before 5 years of age. Airborne transmission has been demonstrated in animal studies, and epidemiologic studies suggest person-person transmission in nosocomial settings. Patients with defects in cell and humoral immunity are at risk for developing pneumonia. Most cases are in HIV-infected patients with CD4 counts less than 200/μL. Others at risk include patients receiving immunosuppressive agents (particularly glucocorticoids) for cancer or organ transplantation, children with immunodeficiency, premature malnourished infants, and patients receiving biologic immunomodulating agents. Pneumocystis pneumonia typically presents in non–HIV-infected patients with several days of dyspnea, fever, and nonproductive cough. Often symptoms develop during or soon after a glucocorticoid taper. Pneumocystis is associated with a reduced diffusing capacity on pulmonary function that typically causes mild hypoxemia and significant oxygen desaturation with exertion. Chest radiography often shows bilateral diffuse infiltrates without pleural effusion. Early in the disease, the radiograph may be unremarkable, but chest computed tomography (CT) will show diffuse ground glass infiltrates as in this case. Patients receiving biologic agents are at risk of pneumonia caused by tuberculosis (the patient was on prophylaxis in this case), Aspergillus spp., and Nocardia spp. Aspergillus spp., Nocardia spp., and septic emboli typically appears as nodules on chest CT. Rheumatoid nodules would be unlikely in the context of improving joint disease.

IV-215. The answer is D. (Chap. 207) Prophylaxis is effective in decreasing the risk of Pneumocystis pneumonia. It is clearly indicated in HIV-infected patients with oropharyngeal candidiasis or CD4 count below 200/μL and in HIV-infected or non–HIV-infected patients with a history of prior Pneumocystis pneumonia. Prophylaxis may be discontinued in HIV-infected patients who respond to therapy after the CD4 count has risen more than 200/μL for more than 3 months. Indications for primary prophylaxis for at-risk non-HIV infected patients without prior pneumocystis pneumonia (e.g., patients receiving induction chemotherapy or high-dose corticosteroids) are less clear. Trimethoprim–sulfamethoxazole remains the drug of choice for primary and secondary prophylaxis. It also provides protection from opportunistic toxoplasmosis and some bacterial infections.

IV-216. The answer is E. (Chap. 207) Pneumocystis jiroveci lung infection is known to worsen after initiation of treatment, likely caused by lysis of organisms and immune response to their intracellular contents. It is thought that adjunct administration of glucocorticoids may reduce inflammation and subsequent lung injury in patients with moderate to severe pneumonia caused by P. jiroveci. Adjunct administration of glucocorticoids in patients with moderate to severe disease as determined by a room air below 70 mmHg or an A–a gradient greater than 35 mmHg has been shown to decrease mortality. Glucocorticoids should be given for a total duration of 3 weeks. Patients often do not improve until many days into therapy and often initially worsen; steroids should be used early in the course of illness rather than waiting for lack of improvement. Pneumothoraces and adult respiratory distress syndrome (ARDS) are common feared complications of Pneumocystis infection. If patients present with ARDS caused by Pneumocystis pneumonia, they meet the criterion for adjunct glucocorticoids because of the severe nature of disease. The use of glucocorticoids as adjunctive therapy in HIV-infected patients with mild disease or in non–HIV-infected patients remains to be evaluated.

IV-217. The answer is A. (Chap. 208) Mefloquine remains the preferred drug for malaria prophylaxis in areas where chloroquine resistance is prevalent. High doses may be used for treatment. Drug resistance has been reported in parts of Africa and Southeast Asia. Mefloquine, similar to quinine and chloroquine, is only active against the asexual erythrocytic stages of malarial infection. Mefloquine is poorly water soluble and is not available parenterally. Oral absorption is enhanced when taken with or after food. Mefloquine is excreted mainly in bile and feces. Dosage adjustment is not necessary in patients with renal failure, and the drug is not removed with hemodialysis. Sleep abnormalities, psychosis, and seizures have been reported with mefloquine administration. Mefloquine should not be prescribed to patients with neuropsychiatric conditions, including depression, generalized anxiety disorder, psychosis, or seizure disorders. If acute anxiety, depression, restlessness, or confusion develops during prophylaxis, the drug should be discontinued. Quinine, quinidine, and beta-blockers may interact with mefloquine to cause significant electrocardiographic abnormalities or cardiac arrest. Halofantrine must not be administered with mefloquine within 3 weeks because of the potential for fatal QTc prolongation. Mefloquine may also alter ritonavir pharmacokinetics.

IV-218. The answer is E. (Chap. 209) Entamoeba histolytica is a common pathogen in areas of the world with poor sanitation and crowding. Transmission is oral–fecal, and the primary manifestation is colitis, often heme positive. Liver abscess is a common complication, occurring after the organism crosses the colonic border and travels through the portal circulation, subsequently lodging in the liver. At the time of presentation with liver abscess, the primary gastrointestinal infection has usually cleared, and organisms cannot be identified in the stool. Suggestive imaging with a positive serologic test result for E. histolytica is diagnostic. When a patient has a diagnostic imaging procedure, a positive amebic serology result is highly sensitive (>94%) and highly specific (>95%) for diagnosis of amebic liver abscess. Treatment for amebic liver abscess is generally with metronidazole. Luminal infection can be treated with paromomycin or iodoquinol. Campylobacter is a major cause of foodborne infectious diarrhea. Although usually self-limited, it may cause serious enteritis and inflammatory diarrhea but not liver abscess.

IV-219. The answer is B. (Chap. 210) The patient presents with signs of an acute infectious illness in an endemic area for malaria. A thick and thin preparation of her peripheral blood is indicated to evaluate for trophozoites, and indeed they are found in this case. Her neurologic findings suggest cerebral malaria, a defining feature of severe malaria. She is treated with intravenous quinidine. Hypoglycemia is a frequent finding in severe malaria; is associated with a poor prognosis; and may be worsened by quinidine or quinine therapy, which promotes pancreatic insulin secretion. Quinine causes fewer arrhythmias and hypotension with infusion than quinidine, but it is often not available in U.S. hospital pharmacies. Malaria itself causes hypoglycemia through failure of hepatic gluconeogenesis as well as increased glucose consumption by the host and parasite. Seizures may be caused by cerebral malaria but are not a complication of quinidine. Nightmares are frequently found with mefloquine, and retinopathy is a complication of prolonged chloroquine dosing.

IV-220. The answer is C. (Chap. 210) Severe malaria is a medical emergency. The patient has infection with Plasmodium vivax, which is less likely than Plasmodium falciparum to be associated with severe disease. Clinical manifestations of severe disease that cannot be managed as an outpatient include the presence of coma or cerebral malaria, two or more seizures over 24 hours, severe academia or anemia, renal failure, pulmonary edema or adult respiratory distress syndrome, hypoglycemia, hypotension or shock, evidence of disseminated intravascular coagulation, hemoglobinuria, extreme weakness, and jaundice with bilirubin above 3 mg/dL if combined with other organ dysfunction. Finally, if more than 5% of the erythrocytes are affected on the peripheral smear in a non-immune patient, severe malaria is present, and outpatient therapy is not advised.

IV-221. The answer is B. (Chap. 210) Artemisinin-containing regimens are now recommended by the World Health Organization as first-line agents for Plasmodium falciparum malaria. In severe P. falciparum malaria, intravenous (IV) artesunate reduced mortality by 35% compared with IV quinine. Artemether and artemotil are given intramuscularly and are not as effective as artesunate. Although safer and more effective than quinine, artesunate is not available in the United States. In the United States, quinidine or quinine is used as a necessary choice. Intravenous quinine is as effective as and safer than IV quinidine. Quinine causes fewer arrhythmias and hypotension with infusion than quinidine, but it is often not available in U.S. hospital pharmacies. Chloroquine is only effective for Plasmodium vivax and in Plasmodium ovale and Plasmodium falciparum infection in certain pockets of the Middle East and Caribbean where resistance has not yet developed. Mefloquine comes only as an oral formulation. It is most commonly used as a prophylactic agent but is also used for treatment of multidrug-resistant malaria.

IV-222. The answer is E. (Chap. 210) Thick and thin smears are a critical part of the evaluation of fever in a person with recent time spent in a Plasmodium-endemic region. Thick smears take a longer time to process but increase sensitivity in the setting of low parasitemia. Thin smears are more likely to allow for precise morphologic evaluation to differentiate among the four different types of Plasmodiuminfection and to allow for prognostic calculation of parasitemia. If clinical suspicion is high, repeat smears should be performed if the results are initially negative. If personnel are not available to rapidly interpret a smear, empirical therapy should be strongly considered to ward off the most severe manifestation of Plasmodium falciparum infection. Antibody-based diagnostic tests that are sensitive and specific for P. falciparum infection have been introduced. The results will remain positive for weeks after infection and do not allow quantification of parasitemia.

IV-223. The answer is C. (Chap. 211) The patient is seen in an endemic area for Babesia microti, which includes Nantucket, Martha’s Vineyard, Block Island, Shelter Island, Long Island, southeastern coastal Massachusetts, Connecticut, and Rhode Island. Her flulike symptoms and tick bite make this disease very likely. Patients generally present with these symptoms or occasionally neck stiffness, sore, throat abdominal pain, and weight loss. Physical examination findings are typically normal with the exception of fever. The presence of erythema chronicum migrans suggests concurrent Lyme disease because a rash is not a feature of babesiosis. Although thick or thin preparation typically demonstrates the ring form of this protozoan, if these are negative, the 18S rRNA may be demonstrated by polymerase chain reaction. The ring forms are distinguished from Plasmodium falciparum by the absence of central brownish deposit seen in malarial disease. Babesia duncani is typically found on the West Coast of the United States, and Babesia divergens have been reported sporadically in Washington state, Missouri, and Kentucky. Therapy for severe Babesia microti disease in adults is clindamycin with additional quinine. Red blood cell exchange transfusion may be considered for B. microti but is not recommended as it is with B. divergens.

IV-224. The answer is C. (Chap. 212) Most cases of leishmaniasis occur on the Indian subcontinent and Sudan. The most commonly used technique for diagnosis of visceral leishmaniasis (kala azar) is a rapid immunochromatographic test for recombinant antigen rK39 from Leishmania infantum. This is widely available, rapid, and safe, requiring only a fingerprick of blood with results available in approximately 15 minutes. Although splenic aspiration with demonstration of amastigotes in tissue smear is the gold standard for the diagnosis of visceral leishmaniasis and culture may increase the sensitivity, the test is invasive and may be dangerous in inexperienced hands. Polymerase chain reaction for the leishmaniasis nucleic acid is only available at specialized laboratories and is not routinely used clinically. Leishmaniasis is not diagnosed via stool analysis.

IV-225. The answer is C. (Chap. 213) Trypanosoma cruzi is the causative agent of Chagas disease or American trypanosomiasis, which only occurs in the Americas. The protozoa is transmitted to mammalian hosts by the reduviid bugs, which become infected by sucking blood from animals or humans with circulating protozoa. The infective form of T. cruzi is excreted in the feces and infects humans through contact with breaks in the skin, mucous membranes, or conjunctiva. Infection has also been transmitted from blood transfusion, organ transplant, and ingestion of contaminated food or drink. Acute Chagas disease is typically a mild febrile illness followed by a chronic phase characterized by subpatent parasitemia, antibodies to T. cruzi, and no symptoms. About 10% to 30% of patients with chronic Chagas disease develop symptoms, usually related to cardiac or gastrointestinal lesions. Deer flies are the transmission vector of Loa loa (filariasis).

IV-226. The answer is D. (Chap. 213) This patient most likely has chronic Chagas disease with cardiac involvement and biventricular systolic dysfunction. Chagas disease is a health problem in rural Mexico, Central America, and South America. Most acute cases occur in children, but the epidemiology is uncertain because most cases go undiagnosed. The heart is the organ most often involved in chronic Chagas disease with biventricular systolic dysfunction and conduction abnormalities [right bundle branch block and left anterior hemiblock (LAH)]. Apical aneurysms and mural thrombi may occur. Chronic Chagas disease is diagnosed by demonstration of specific immunoglobulin G antibodies to Trypanosoma cruzi antigens. False-positive results may occur in patients with other parasitic infections or autoimmune disease. The World Health Organization recommends a positive test be confirmed with a separate assay. Polymerase chain reaction (PCR) to detect T. cruzi DNA in chronically infected patients has not been shown to be superior to serology, and no commercially available PCR tests are available. Given the patient’s demographics, lack of coronary artery disease risk factors, and indolent symptoms, acute myocardial infarction, ischemic cardiomyopathy, and hypertensive cardiomyopathy are less likely diagnoses. Right heart catheterization with placement of a Swan-Ganz catheter could quantify left and right heart pressures and cardiac output. Constrictive pericarditis could also be evaluated, but this diagnosis is less likely with the presence of signs of left heart failure.

IV-227. The answer is A. (Chap. 213) Current consensus is that all Trypanosoma cruzi–infected patients up to 18 years old and all newly infected adults be treated for acute Chagas disease. Unfortunately, the only available drugs, benznidazole and nifurtimox, lack efficacy and have notable side effects. In acute Chagas disease, nifurtimox reduces the duration of symptoms and parasitemia and decreases the acute mortality Rate. However, only approximately 70% of acute infections are cured by a full course of treatment. Benznidazole is at least as effective as nifurtimox and is generally the treatment of choice in Latin America. The role of therapy in patient with indeterminate or chronic asymptomatic Chagas disease is controversial. Some experts recommend that therapy be offered. In contrast, randomized studies have shown benefit of treatment in children. The current antifungal azoles, including voriconazole, do not have adequate efficacy against T. cruzi, although newer agents in this class show promise in animal studies. Serologic confirmation with T. cruzi immunoglobulin G testing is used to diagnose chronic, not acute, Chagas disease. Malaria is endemic to Honduras below 1000 m elevation, and primaquine is effective therapy in those cases. Thin and thick smears evaluated by experts should not confuse Plasmodium spp. with T. cruzi.

IV-228. The answer is A. (Chap. 213) Human African trypanosomiasis (HAT) or sleeping sickness is caused by the protozoan Trypanosoma brucei complex. HAT remains a major public health problem in Africa despite its near-eradication in the 1960s. Although HAT only occurs in sub-Saharan Africa, it is important to distinguish between the West African (T.b. gambiense) and East African (T.b. rhodesiense) forms. Tsetse flies are the transmission vector for both forms. Humans are the major reservoir of West African trypanosomiasis, and it occurs in rural areas, rarely affecting tourists. Antelope and cattle are the reservoirs for T.b. rhodesiense, and infection has been reported in safari tourists. A primary lesion (trypanosomal chancre) typically appears 1 week after the bite of an infected tsetse fly. This is followed by a systemic illness with fever and lymphadenopathy (stage 1 disease). Myocarditis may occur, which can be fatal. Central nervous system (CNS) involvement follows (stage 2 disease) with cerebrospinal fluid (CSF) pleocytosis, elevated protein, and elevated pressure. During this stage, trypanosomes may be found in CSF. T.b. rhodesiense tends to be more aggressive with CNS disease developing earlier than T.b. gambiense. Symptoms during stage 2 disease include progressive somnolence and indifference sometimes alternating with insomnia and nighttime restlessness. If untreated, symptoms progress to coma and death. Diagnosis requires demonstration of the protozoa from blood, CSF, lymph node material, bone marrow, or chancre fluid. There are serologic tests for T.b. gambiense, but they lack the sensitivity or specificity for treatment decisions. There are not yet commercially available polymerase chain reaction tests. All patients with HAT should have a lumbar puncture to evaluate for CNS involvement, which will determine therapy. Suramin is effective for stage 1 East African HAT (T.b. rhodesiense). Pentamidine is first-line treatment for stage 1 West African HAT. When the CSF is involved, eflornithine is used for West African HAT and melarsoprol for East African HAT. Melarsoprol is an arsenical that is highly toxic, with a risk of encephalopathy.

IV-229. The answer is C. (Chap. 214) The magnetic resonance imaging (MRI) scan shows the classic lesions of encephalitis caused by Toxoplasma gondii in a patient with advanced immunosuppression caused by HIV infection. Cats are the definitive host for the sexual phase of Toxoplasma, and oocysts are shed in their feces. In the United States, up to 30% of 19-year-old young adults and up to 67% of adults older than 50 years of age have serologic evidence of Toxoplasma exposure. Patients with HIV infection are at risk of reactivation of latent toxoplasmosis with resultant encephalitis when the CD4 T-cell count falls below 100/μL. Patients receiving immunosuppressive medication for lymphoproliferative disease or solid organ transplant are also at risk for reactivation of latent disease. Although the central nervous system (CNS) is the most common site of symptomatic reactivation disease, the lymph nodes, lung, heart, eyes, and gastrointestinal tract may be involved. Toxoplasma usually causes encephalitis, not meningitis; therefore, cerebrospinal fluid (CSF) findings may be unremarkable or have modest elevations of cell count and protein (with normal glucose). The treatment of choice for CNS toxoplasmosis is pyrimethamine plus sulfadiazine. Trimethoprim–sulfamethoxazole is an acceptable alternative. The differential diagnosis of encephalitis in patients with AIDS includes lymphoma, metastatic tumor, brain abscess, progressive multifocal leukoencephalopathy, fungal infection, and mycobacterial infection. In this case, given the classic MRI findings, toxoplasmosis is most likely.

IV-230. The answer is C. (Chap. 215) Of the listed protozoa, only Giardia infection can be diagnosed with stool ova and parasite examination. Stool antigen immunoassay can be used to diagnose Giardiaand Cryptosporidium spp. Fecal acid-fast testing may be used to diagnose CryptosporidiumIsospora, and Cyclospora spp. Microsporidia require special fecal stains or tissue biopsy for diagnosis.

IV-231. The answer is D. (Chap. 215) Trichomoniasis is transmitted via sexual contact with an infected partner. Many men are asymptomatic but may have symptoms of urethritis, epididymitis, or prostatitis. Most women have symptoms of infection that include vaginal itching, dyspareunia, and malodorous discharge. These symptoms do not distinguish Trichomonas infection from other forms of vaginitis, such as bacterial vaginosis. Trichomoniasis is not a self-limited infection and should be treated for symptomatic and public health reasons. Wet-mount examination for motile trichomonads has a sensitivity of 50% to 60% in routine examination. Direct immunofluorescent antibody staining of secretions is more sensitive and can also be performed immediately. Culture is not widely available and takes 3 to 7 days. Treatment should consist of metronidazole either as a single 2-g dose or 500-mg doses twice daily for 7 days; all sexual partners should be treated. Trichomoniasis resistant to metronidazole has been reported and is managed with increased doses of metronidazole or with tinidazole.

IV-232. The answer is E. (Chap. 215) Giardiasis is diagnosed by detection of parasite antigens in the feces or by visualizing cysts or trophozoites in feces or small intestine. There is no reliable serum test for this disease. Because a wide variety of pathogens are responsible for diarrheal illness, some degree of diagnostic testing beyond the history and physical examination is required for definitive diagnosis. Colonoscopy does not have a role in diagnosing Giardia infection. Giardiasis can persist in symptomatic patients and should be treated. Severe symptoms such as malabsorption, weight loss, growth retardation, and dehydration may occur in prolonged cases. Additionally, extraintestinal manifestations such as urticarial, anterior uveitis, and arthritis have been associated with potential giardiasis. A single oral 2-g dose of tinidazole is reportedly more effective than a 5-day course of metronidazole with cure rates above 90% for both. Paromomycin, an oral poorly absorbed aminoglycoside, can be used for symptomatic patients during pregnancy, but its efficacy for eradicating infection is not known. Clindamycin and albendazole do not have a role in treatment of giardiasis. Refractory disease with persistent infection can be treated with a longer duration of metronidazole.

IV-233. The answer is D. (Chap. 215) Cryptosporidium typically causes a self-limited diarrheal illness in immunocompetent patients but may cause severe debilitating disease in patients with severe immunodeficiency, such as advanced HIV infection. Outbreaks in immunocompetent hosts are caused by ingestion of oocysts. Infectious oocysts are excreted in human feces, causing human-to-human transmission. Waterborne transmission of oocysts accounts for disease in travelers and common-source outbreaks. Oocysts resist killing by routine chlorination of drinking and recreational water sources. Infection may be asymptomatic in immunocompetent and immunosuppressed hosts. Diarrhea is typically watery and nonbloody and may be associated with abdominal pain, nausea, fever, and anorexia. In immunocompetent hosts, symptoms usually subside in 1 to 2 weeks without therapy. In advanced AIDS with CD4 counts below 100/μL, severe symptoms may develop, leading to significant electrolyte and volume loss. Nitazoxanide is approved for treatment of Cryptosporidium but to date has not been shown to be effective in HIV-infected patients. The best available therapy for these patients is antiretroviral therapy to reduce immune suppression. Tinidazole and metronidazole are used to treat giardiasis and trichomoniasis, not cryptosporidiosis.

IV-234. The answer is D. (Chap. 216) There are roughly 12 cases of trichinellosis reported each year in the United States. Because most infections are asymptomatic, this may be an underestimate. Recent outbreaks in North American have been related to ingestion of wild game, particularly bear. Heavy infections can cause enteritis; periorbital edema; myositis; and, infrequently, death. This infection, caused by ingesting Trichinella cysts, occurs when infected meat from pigs or other carnivorous animals is eaten. Laws that prevent feeding pigs uncooked garbage have been an important public health measure in reducing Trichinella infection in this country. Person-to-person spread has not been described. The majority of infections are mild and resolve spontaneously.

IV-235. The answer is E. (Chap. 216) Trichinellosis occurs when infected meat products are eaten, most frequently pork. The organism can also be transmitted through the ingestion of meat from dogs, horses, and bears. Recent outbreaks in the United States and Canada have been related to consumption of wild game, particularly bear meat. During the first week of infection, diarrhea, nausea, and vomiting are prominent features. As the parasites migrate from the gastrointestinal (GI) tract, fever and eosinophilia are often present. Larvae encyst after 2 to 3 weeks in muscle tissue, leading to myositis and weakness. Myocarditis and maculopapular rash are less common features of this illness. In pork, larvae are killed by cooking until the meat is no longer pink or by freezing at –15°C for 3 weeks. However, arctic Trichinella nativa larvae in walrus or bear meat are resistant to freezing. Giardia and Campylobacterare organisms that are frequently acquired by drinking contaminated water; neither produces this pattern of disease. Although both cause GI symptoms (and Campylobacter causes fever), neither causes eosinophilia or myositis. Taenia solium, or pork tapeworm, shares a similar pathogenesis to Trichinellaspp. but does not cause myositis. Cytomegalovirus has varied presentations but none that lead to this presentation.

IV-236 and IV-237. The answers are D and B, respectively. (Chap. 216) Visceral larva migrans, caused in this case by the canine roundworm Toxocara canis, most commonly affects young children who are exposed to canine stool. Toxocara eggs are ingested and begin their life cycle in the small intestine. They migrate to many tissues in the body. Particularly characteristic of this illness are hepatosplenomegaly and profound eosinophilia, at times close to 90% of the total white blood cell count. Staphylococci will not typically cause eosinophilia. Trichinellosis, caused by ingesting meat from carnivorous animals that has been infected with Trichinella cysts, does not cause hepatosplenomegaly and is uncommon without eating a suspicious meal. Giardiasis is characterized by profuse diarrhea and abdominal pain without systemic features or eosinophilia. Cysticercosis typically causes myalgias and can spread to the brain, where it is often asymptomatic but can lead to seizures. The vast majority of Toxocara infections are self-limited and resolve without therapy. Rarely, severe symptoms may develop with deaths caused by central nervous system, myocardial, or respiratory disease. Severe myocardial involvement manifests as acute myocarditis. In these patients, glucocorticoids are administered to reduce the inflammatory complications. Antihelminthic drugs such as albendazole, mebendazole, or praziquantel have not been shown conclusively to alter the course of visceral larval migrans. Metronidazole is used for infections caused by Trichomonas, not tissue nematodes.

IV-238. The answer is A. (Chap. 216) Angiostrongylus cantonensis, the rat lungworm, is the most common cause of human eosinophilic meningitis. The infection principally occurs in Southeast Asia and the Pacific Basin, although cases have also been described in Cuba, Australia, Japan, and China. Infective larvae are excreted in rat feces and ingested by land snails and slugs. Humans acquire infection by ingesting the mollusks, vegetables contaminated by mollusk slime, or seafood (crabs, freshwater shrimp) that consumed the mollusks. The larvae migrate to the brain, where they initiate a marked eosinophilic inflammatory response with hemorrhage. Clinical symptoms develop 2 to 35 days after ingestion of larvae, and the initial presentation typically includes headache (indolent or acute), fever, nausea, vomiting, and meningismus. The cerebrospinal fluid (CSF) findings are as in this case with an eosinophil percentage greater than 20%. A. cantonensis larvae are only rarely demonstrated in the CSF. The diagnosis usually relies on the presence of eosinophilic meningitis and compatible epidemiology. There is no specific chemotherapy for A. cantonensis meningitis. Supportive care includes repeat removal of CSF to control intracranial pressure. Glucocorticoids may reduce inflammation. In most cases, cerebral angiostrongyliasis has a self-limited course with complete recovery. Gnathostoma spinigerum is a less common cause of eosinophilic meningoencephalitis. It also causes migratory cutaneous swellings or eye infections. It is also endemic in Southeast Asia and China and is usually transmitted by eating undercooked fish or poultry (som fak in Thailand and sashimi in Japan). Trichinella murrelli and Trichinella nativa cause trichinosis in North America and the Arctic, respectively. Trichinella cara is the cause of larval migrans.

IV-239. The answer is B. (Chap. 217) Strongyloides is the only helminth that can replicate in the human host, allowing autoinfection. Humans acquire Strongyloides when larvae in fecally contaminated soil penetrate the skin or mucous membranes. The larvae migrate to the lungs via the bloodstream; break through the alveolar spaces; ascend the respiratory airways; and are swallowed to reach the small intestine, where they mature into adult worms. Adult worms may penetrate the mucosa of the small intestine. Strongyloides is endemic in Southeast Asia, sub-Saharan Africa, Brazil, and the Southern United States. Many patients with Strongyloides are asymptomatic or have mild gastrointestinal symptoms or the characteristic cutaneous eruption, larval currens, as described in this case. Small bowel obstruction may occur with early heavy infection. Eosinophilia is common with all clinical manifestations. In patients with impaired immunity, particularly glucocorticoid therapy, hyperinfection or dissemination may occur. This may lead to colitis, enteritis, meningitis, peritonitis, and acute renal failure. Bacteremia or gram-negative sepsis may develop because of bacterial translocation through disrupted enteric mucosa. Because of the risk of hyperinfection, all patients with Strongyloides infection, even asymptomatic carriers, should be treated with ivermectin, which is more effective than albendazole. Fluconazole is used to treat candidal infections. Mebendazole is used to treat trichuriasis, enterobiasis (pinworm), ascariasis, and hookworm. Mefloquine is used for malaria prophylaxis.

IV-240. The answer is B. (Chap. 217) Ascaris lumbricoides is the longest nematode (15–40 cm) parasite of humans. It resides in tropical and subtropical regions. In the United States, it is found mostly in the rural Southeast. Transmission is through fecally contaminated soil. Most commonly, the worm burden is low, and it causes no symptoms. Clinical disease is related to larval migration to the lungs or to adult worms in the gastrointestinal tract. The most common complications occur because of a high gastrointestinal adult worm burden, leading to small bowel obstruction (most often in children with a narrow-caliber small bowel lumen) or migration leading to obstructive complications such as cholangitis, pancreatitis, or appendicitis. Rarely, adult worms can migrate to the esophagus and be orally expelled. During the lung phase of larval migration (9–12 days after egg ingestion), patients may develop a nonproductive cough, fever, eosinophilia, and pleuritic chest pain. Eosinophilic pneumonia syndrome (Löffler’s syndrome) is characterized by symptoms and lung infiltrates. Meningitis is not a known complication of ascariasis but can occur with disseminated strongyloidiasis in an immunocompromised host.

IV-241. The answer is A. (Chap. 217) Ascariasis should always be treated, even in asymptomatic cases, to prevent serious intestinal complications. Albendazole, mebendazole, and ivermectin are effective. These agents should not be administered to pregnant women. Pyrantel is safe in pregnancy. Metronidazole is used for anaerobic bacterial and Trichomonas infections. Fluconazole is mostly used to treat Candida infections. Diethyl-carbamazine (DEC) is first-line therapy for active lymphatic filariasis. Vancomycin has no effect on nematodes.

IV-242. The answer is E. (Chap. 217) This patient’s most likely diagnosis is anisakiasis. This is a nematode infection in which humans are an accidental host. It occurs hours to days after ingesting eggs that previously settled into the muscles of fish. The main risk factor for infection is eating raw fish. Presentation mimics an acute abdomen. History is critical because upper endoscopy is both diagnostic and curative. The implicated nematodes burrow into the mucosa of the stomach, causing intense pain, and must be manually removed by endoscope or, on rare occasion, surgery. There is no medical agent known to cure anisakiasis.

IV-243. The answer is E. (Chap. 218) This patient likely has filariasis with acute lymphadenitis caused by Wuchereria bancrofti. It is endemic throughout the tropics and subtropics, including Asia, the Pacific Islands, Africa, parts of South America, and the Caribbean. W. bancrofti is the most widely distributed human filarial parasite and is transmitted by infected mosquitoes. Lymphatic infection is common and may be acute or chronic. Chronic lower extremity lymphatic infection causes elephantiasis. Definitive diagnosis requires demonstration of the parasite. Microfilariae may be found in blood, hydrocele, or other body fluid collections by direct microscopic examination. Enzyme-linked immunosorbent assays for circulating antigens are available commercially and have sensitivity of greater than 93% with excellent specificity. Polymerase chain reaction–based assays have been developed that may be as effective. In cases of acute lymphadenitis, ultrasound examination with Doppler may actually reveal motile worms in dilated lymphatics. Live worms have a distinctive movement pattern (filarial dance sign). Worms may be visualized in the spermatic cords of up to 80% of men infected with W. bancrofti. Stool ova and parasite examination is not useful for demonstration of W. bancrofti.

IV-244. The answer is B. (Chap. 218) Diethylcarbamazine (DEC), which has macro- and microfilaricidal properties, is the first-line treatment for acute filarial lymphadenitis. Albendazole, doxycycline, and ivermectin are also used to treat microfilarial infections (not macrofilarial). There is growing consensus that virtually all patients with Wuchereria bancrofti infection should be treated, even if asymptomatic, to prevent lymphatic damage. Many of these patients have microfilarial infection with subclinical hematuria, proteinuria, and so on. Albendazole and doxycycline have demonstrated macrofilaricidal efficacy. Combinations of DEC with albendazole, ivermectin, and doxycycline have efficacy in eradication programs. The World Health Organization established a global program to eliminate lymphatic filariasis in 1997 using a single annual dose of DEC plus either albendazole (non-African regions) or ivermectin (Africa). Praziquantel is used for treatment of schistosomiasis.

IV-245. The answer is B. (Chap. 218) This patient has loiasis caused by the African eye worm Loa loa. It is endemic to the rain forests of Central and West Africa. Microfilaria circulate periodically in blood with macrofilaria living in subcutaneous tissues including the subconjunctiva. Loiasis is often asymptomatic in indigenous regions with recognition, as in this case, only with visualized macrofilarial migration. Angioedema and swelling may occur in affected areas. Diethylcarbamazine (DEC) is effective treatment for the macrofilarial and microfilarial stages of disease. Multiple courses may be necessary. Albendazole and ivermectin are effective in reducing microfilarial loads but are not approved by the U.S. Food and Drug Administration. There are reports of deaths in patients with heavy loads of microfilaria receiving ivermectin. Terbinafine is the treatment for ringworm. Voriconazole is an antifungal with no activity against worms.

IV-246. The answer is A. (Chap. 219) Human schistosomiasis is caused by five species of parasitic trematodes. Whereas Schistosoma mansoni, Schistosoma japonicum, Schistosoma mekongi, and Schistosoma intercalatum are intestinal species, Schistosoma haematobium is a uni-nary species. There are reportedly up to 300 million individuals infected with SchistosomaFigure IV-246 shows the global distribution. S. haematobium is not seen in South America. All forms of schistosomiasis are initiated by penetration of infective cercariae released from infected snails into fresh water. After entering the skin, the schistosome migrates via venous or lymphatic vessels to either the intestinal or urinary venous system, depending on the species. Acute skin infection causes dermatitis (swimmer’s itch) within 2 to 3 days. Katayama fever, acute schistosomal serum sickness–related to migration, may develop in 4 to 8 weeks. Eosinophilia is common in acute infection. This has become a more common global health problem because travelers are exposed while swimming or boating in infected fresh water bodies. Chronic schistosomiasis depends on the species and the location of infection. The intestinal species are responsible for portal hypertension. S. haematobium causes urinary symptoms and a higher risk of urinary tract carcinoma. Immunologic tests are available to diagnose schistosomiasis, and in some cases, stool or urine examination results may be positive.

image

FIGURE IV-246 Global distribution of schistosomiasis. A. S. mansoni infection (lightest shade) is endemic in Africa, the Middle East, South America, and a few Caribbean countries. S. intercalatum infection (darkest shade) is endemic in sporadic foci in West and Central Africa. B. S. haematobium infection (medium shade) is endemic in Africa and the Middle East. The major endemic countries for S. japonicuminfection (darkest shade) are China, the Philippines, and Indonesia. S. mekongi infection (black) is endemic in sporadic foci in Southeast Asia.

IV-247. The answer is D. (Chap. 219) This patient has Katayama fever caused by infection with Schistosoma mansoni. Approximately 4 to 8 weeks after exposure, the parasite migrates through the portal and pulmonary circulations. This phase of the illness may be asymptomatic but in some cases evokes a hypersensitivity response and a serum sickness–type illness. Eosinophilia is usual. Because there is not a large enteric burden of parasites during this phase of the illness, stool study results may not be positive, and serology may be helpful, particularly in patients from nonendemic areas. Praziquantel is the treatment of choice because Katayama fever may progress to include neurologic complications. Praziquantel remains the treatment for most helminthic infections, including schistosomiasis. Chloroquine is used for treatment of malaria; mebendazole for ascariasis, hookworm, trichinosis, and visceral larval migrans; metronidazole for amebiasis, giardiasis, and trichomoniasis; and thiabendazole for Strongyloidesspp.

IV-248. The answer is B. (Chap. 219) Schistosoma mansoni infection of the liver causes cirrhosis from vascular obstruction resulting from periportal fibrosis but relatively little hepatocellular injury. Hepatosplenomegaly, hypersplenism, and esophageal varices develop quite commonly, and schistosomiasis is usually associated with eosinophilia. Spider nevi, gynecomastia, jaundice, and ascites are observed less commonly than they are in alcoholic and postnecrotic fibrosis.

IV-249. The answer is B. (Chap. 220) This patient has a new onset of seizures caused by neurocysticercosis from infection with Taenia solium (pork tapeworm). The computed tomography (CT) scan shows a parenchymal cysticercus with enhancement of the cyst and an internal scolex (arrow). The cyst represents larval oncospheres that have migrated to the central nervous system (CNS). Infections that cause human cystercicosis result from ingestion of T. solium eggs, usually from close contact with a tapeworm carrier who developed intestinal infection for ingestion of undercooked pork. Autoinfection may occur if an individual ingests tapeworm eggs excreted in their own feces. Cysticerci may be found anywhere in the body, but clinical manifestations usually arise from lesions in the CNS, cerebrospinal fluid (CSF), skeletal muscle, subcutaneous tissue, or eye. Neurologic manifestations are most common, including generalized or focal seizures from surrounding inflammation, hydrocephalus from CSF outflow occlusion, or arachnoiditis. As shown in Table IV-249, neuroradiologic demonstration of a cystic lesion containing a characteristic scolex is absolute criteria for diagnosis of cysticercosis. Intestinal infection may be detected by fecal examination for eggs. More sensitive enzyme-linked immunosorbent assay, polymerase chain reaction, and serologic testing is not currently commercially available. Treatment of neurocysticercosis after neurologic stabilization is with albendazole or praziquantel. Studies have shown faster resolution of clinical and radiologic findings compared with placebo. Initiation of therapy may be associated with worsening symptoms caused by inflammation that is treated with glucocorticoids. Intestinal T. solium infection is treated with a single dose of praziquantel. CNS cystic lesions (but without the visualized scolex) are typical of toxoplasmosis in patients with advanced HIV infection and are treated with pyrimethamine and sulfadiazine. However, in this case, the patient was documented HIV antibody negative, and the CT lesion was typical for cysticercosis. Viral testing for HIV would not be helpful because toxoplasmosis is seen in advanced cases, not acute infection. Echocardiography would be indicated for suspected staphylococcal (or other bacterial) endocarditis with systemic embolization.

TABLE IV-249 Diagnostic Criteria for Human Cysticercosisa


1. Absolute criteria

a. Demonstration of cysticerci by histologic or microscopic examination of biopsy material

b. Visualization of the parasite in the eye by funduscopy

c. Neuroradiologic demonstration of cystic lesions containing a characteristic scolex

2. Major criteria

a. Neuroradiologic lesions suggestive of neurocysticercosis

b. Demonstration of antibodies to cysticerci in serum by enzyme-linked immunoelectrotransfer blot

c. Resolution of intracranial cystic lesions spontaneously or after therapy with albendazole or praziquantel alone

3. Minor criteria

a. Lesions compatible with neurocysticercosis detected by neuroimaging studies

b. Clinical manifestations suggestive of neurocysticercosis

c. Demonstration of antibodies to cysticerci or cysticercal antigen in cerebrospinal fluid by ELISA

d. Evidence of cysticercosis outside the central nervous system (e.g., cigar-shaped soft tissue calcifications)

4. Epidemiologic criteria

a. Residence in a cysticercosis-endemic area

b. Frequent travel to a cysticercosis-endemic area

c. Household contact with an individual infected with Taenia solium


aDiagnosis is confirmed by either one absolute criterion or a combination of two major criteria, one minor criterion, and one epidemiologic criterion. A probable diagnosis is supported by the fulfillment of (1) one major criterion plus two minor criteria, (2) one major criterion plus one minor criterion and one epidemiologic criterion, or (3) three minor criteria plus one epidemiologic criterion.

Abbreviation: ELISA, enzyme-linked immunosorbent assay.

Source: Modified from Del Brutto OH et al: Proposed diagnostic criteria for neurocysticercosis. Neurology 57:177, 2001.

IV-250. The answer is B. (Chap. 220) Echinococcosis is usually caused by infection of Echinococcus granulosus complex or Echinococcus multilocularis transmitted to humans via dog feces. E. granulosusis found on all continents with high prevalence in China, central Asia, Middle East, Mediterranean region, Eastern Africa, and parts of South America. E. multi-locularis, which causes multiloculated invasive lung lesions, is found in alpine, sub-Arctic, or Arctic regions, including Canada, the United States, China, Europe, and central Asia. Echinococcal cysts, most commonly in the liver followed by the lung, are typically slowly enlarging and cause symptoms because of space-occupying effects. Cysts are often incidentally discovered on radiologic studies. Compression or leakage into the biliary system may cause symptoms typical for cholelithiasis or cholecystitis. Echinococcal cysts may be characterized by ultrasonography. Demonstration of daughter cysts within a larger cyst is pathognomonic. Serodiagnosis may be helpful in questionable cases for diagnosis of E. granulosus. Patients with liver cysts typically have positive serology in more than 90% (but not 100%) of cases. Up to 50% of patients with lung cysts may be seronegative. Biopsy is generally not recommended for cysts close to the liver edge because of the risk of leakage. Small cysts may respond to medical therapy with albendazole or praziquantel. Percutaneous aspiration-injection-resaspiration (PAIR) therapy is recommended for most noncomplex nonsuperficial cysts. Surgical resection is recommended for complex cysts, superficial cysts with risk of leakage, and cysts involving the biliary system. Albendazole therapy is generally administered before and after PAIR or surgical therapy.

 



If you find an error or have any questions, please email us at admin@doctorlib.info. Thank you!